Vous êtes sur la page 1sur 282

!

Your way to Top Business Schools _____________


1















QUANTITATIVE SECTION
HANDBOOK













Course: GMAT

Student______________________________

Third Edition
February 2011
www.mbastrategy.ru | www.mbastrategy.ua





!

Your way to Top Business Schools _____________
2


!

Your way to Top Business Schools _____________
3

Contents
Introduction ................................................................................................................................................ 5
Lesson 1 .................................................................................................................................................. 6
Numbers .................................................................................................................................................. 6
Divisibility ...............................................................................................................................................10
Decimals and Fractions ...........................................................................................................................14
Digits ......................................................................................................................................................22
Powers and Roots ...................................................................................................................................25
Home assignment ...................................................................................................................................29
Test1. Number properties, fractions, digits, powers and roots, ratios, percent, divisibility ...................39
Lesson 2 ..................................................................................................................................................42
Progressions ...........................................................................................................................................42
Statistics .................................................................................................................................................46
Home assignment ...................................................................................................................................52
Test 2 Progression and statistics .........................................................................................................56
Lesson 3 .................................................................................................................................................60
Factorization ...........................................................................................................................................60
LCM and GCD ..........................................................................................................................................65
Division with Remainder .........................................................................................................................69
Home assignment...................................................................................................................................73
Test 3 Prime factorization, LCM, GCD, division with remainder...........................................................78
Lesson 4 Algebra ....................................................................................................................................81
Functions. Substitution. Symbolism. ........................................................................................................81
Simplifying algebraic expressions ............................................................................................................84
Linear equations. Equations with the absolute value. Systems of linear equations. .................................86
Quadratic equations ...............................................................................................................................90
Exponential equations ............................................................................................................................94
Inequalities. Inequalities with the absolute value. Exponential inequalities. ............................................95
Home assignment. ..................................................................................................................................98
Test 4 algebra...................................................................................................................................106
Lesson 5 Geometry ..............................................................................................................................110
Triangles ...............................................................................................................................................112
Quadrilaterals .......................................................................................................................................119
Polygons ...............................................................................................................................................123

!

Your way to Top Business Schools _____________
4

Circle ....................................................................................................................................................124
Volume geometry .................................................................................................................................127
Coordinate geometry ............................................................................................................................130
Home assignment .................................................................................................................................133
Test 5 Geometry ..............................................................................................................................144
Lesson 6 Word Problems ......................................................................................................................152
Systems of Linear Equations ..................................................................................................................152
Rate problems ......................................................................................................................................157
Work problems .....................................................................................................................................159
Mixture Problems .................................................................................................................................161
Home assignment. ................................................................................................................................164
Test 6 Word problems ......................................................................................................................176
Lesson 7 ...............................................................................................................................................182
Combinatorics .......................................................................................................................................182
Probability ............................................................................................................................................190
Home assignment. ................................................................................................................................197
Test 7 Combinatorics, probability .....................................................................................................210
Practice test 1 .......................................................................................................................................216
Practice test 2 ......................................................................................................................................227
Practice test 3 ......................................................................................................................................239
Practice test 4 ......................................................................................................................................250
Key Glossary .............................................................................................................................................262
List of Mistakes .........................................................................................................................................271
Appendix ..................................................................................................................................................281



!

Your way to Top Business Schools _____________
5

Introduction

The quantitative section of the GMAT contains 37 multiple choice-questions in a 75-minute period.

The math topics include arithmetic, basic algebra and geometry (no proofs). The test writers
carefully choose questions to eliminate biases toward candidates with specific majors: all test
takers will be on a level playing field. The section is designed to test your ability to solve
problems, rather than your mathematical knowledge. Questions lean heavily toward word
problems and applying mathematical formulas in typical real-world applications. The handbook
covers all themes with formulas, definitions, main types of questions and algorithms for solving.

The questions include two formats:
a) standard multiple choice
b) data sufficiency

Instructions for Data Sufficiency Questions:
Directions: Each of the following Data Sufficiency problems contains a question followed by two
statements, numbered (1) and (2). You need not solve the problem; rather you must decide whether
the information given is sufficient to solve the problem.

The correct answer to a question is
A if statement (1) ALONE is sufficient to answer the question but statement (2) alone is not
sufficient;
B if statement (2) ALONE is sufficient to answer the question but statement (1) alone is not
sufficient;
C if the two statements TAKEN TOGETHER are sufficient to answer the question, but NEITHER
statement ALONE is sufficient;
D if EACH statement ALONE is sufficient to answer the question;
E if the two statements TAKEN TOGETHER are still NOT sufficient to answer the question.

Drawings: The drawings are drawn to scale according to the information given in the question, but
may conflict with the information given in statements (1) and (2).
You can assume that a line that appears straight is straight and that angle measures cannot be
zero.
You can assume that the relative positions of points, angles, and objects are as shown. All
drawings lie in a plane unless stated otherwise.

!

Your way to Top Business Schools _____________
6

Lesson 1
Numbers

Numbers are classified according to their type. The first type is the one you have known since
elementary school:

Natural numbers/whole positive numbers are the numbers used for counting: 1, 2, 3, 4, 5,
Adding 0 and the negatives of the naturals, we obtain

Integers - numbers from the set {, -2, -1, 0, 1, 2, 3, 4, 5, }
Note: The integer 0 is neither positive nor negative.

1. (OG) The value of -3 (-10) is how much greater than the value of -10 (-3)?
Explanation:
Since -3 (-10) = -3 + 10 = 7 and -10 (-3) = -10 + 3 = -7, the first expression is greater than the
second one by 7 (-7) = 14.

2. (OG) If x = -(2 - 5), then what is the value of x?
Explanation:
Since 2 5 = -3, x = -(-3) = 3.

Integers




Even (divisible by 2) Odd (not divisible by two)
{, -4, -2, 0, 2, 4, } {, -5, -3, -1, 1, 3, 5, }
N=2k N=2k+1
Note: The integer 0 is even: 0=20.


Integers




Positive (greater than 0) Negative (less than 0)
{1, 2, 3, } 0 {, -5, -4, -3, -2 -1}


!

Your way to Top Business Schools _____________
7


Properties of even/odd integers
even even = even
even ouu = even
ouu ouu = ouu
even +even = even
ouu +ouu = even
ouu +even = ouu

3. (OG) If n is a member of the set {33, 36, 38, 39, 41, 42}, what is the value of n?
(1) n is even
(2) n is a multiple of 3

Explanation:
From statement (1) we know that n belongs to the subset {36, 38, 42}. Thus, we cannot uniquely
determine the value of n. Therefore (1) is not sufficient to answer the stem. Then, the answer must
be B, C, or E. Statement (2) alone is again insufficient since it states that n belongs to the subset
{33, 36, 39, 42}. The answer must be C or E. Statements (1) and (2) together are insufficient since
from (1) and (2) we can derive that n belongs to {36, 42}. The answer is E.

The numbers 2, 1, 0, 1, 2, 3, 4, 5 are consecutive integers. Consecutive integers can be
represented by n, n + 1, n + 2, n + 3, . . . , where n is an integer.
The numbers 0, 2, 4, 6, 8 are consecutive even integers. Consecutive even integers can be
represented by 2n, 2n + 2, 2n + 4, . . . , where n is an integer.
The numbers 1, 3, 5, 7, 9 are consecutive odd integers. Consecutive odd integers can be
represented by 2n + 1, 2n + 3, 2n + 5, . . . , where n is an integer.

4. If m and n are consecutive integers, is m greater than n?
(1) m 1 and n + 1 are consecutive integers
(2) m is an even number

Explanation:
Since m and n are consecutive, one of the following is true: m = n + 1 or m = n 1. In the first case
m > n, while in the second case m < n. The stem asks which one is true. Statement (1) actually says
that m = n + 1. Otherwise m = n 1, and {m 1, n + 1} (in fact, {n-2, n+1}), are not consecutive
integers. Thus, statement (1) is sufficient to answer the stem question. Given this, the answer must
be A or D. Statement (2) states that m is even, but this fact cannot help us to determine whether m =
n + 1 or m = n 1 because nothing has been said about n. Thus, statement (2) alone is insufficient
to answer the stem question. The answer is A (it cannot be D since (2) is insufficient).

5. How many integers n are there such that r < n < s?
(1) s r = 5
(2) r and s are not integers

Explanation:
We shouldnt forget that there are non-integer numbers on the number line. For example, if r = 0.5
and s = 5.5, s r = 5 there are 5 integers between s and r. But if r = 0, s = 5, s r = 5 and there are
only 4 integers between s and r. Thus, statement (1) alone is insufficient to answer the question
x eve odd
eve eve even
odd eve odd
+ even odd
eve even odd
odd Odd even

!

Your way to Top Business Schools _____________
8

asked in the stem. Then, the answer must be B, C, or E. Statement (2) says that s and r are not
integers. But it tells nothing about the difference between s and r and is not sufficient. Thus, the
answer must be C or E. Statements (1) and (2) combined are sufficient to answer the stem question
since it is known that the difference between s and r is 5 and s is not integer. Thus, the answer is C.

6. (OG) A certain company currently has how many employees?
(1) If 3 additional employees are hired by the company, and all of the present employees remain,
there will be at least 20 employees in the company
(2) If no additional employees are hired by the company, and 3 of the present employees resign,
there will be fewer than 15 employees in the company

Explanation:
Let E be the number of employees in the company. From statement (1) we know that E + 3 20. (1)
is not sufficient to answer the stem. Given this, the answer must be B, C, or E. Statement (2) alone
is again insufficient since it states that E 3 < 15, but this fact cannot help us to determine E. Thus,
statement (2) alone is insufficient to answer the stem question. The answer must be C or E.
Statements (1) and (2) together are sufficient since from inequalities E + 3 20, E 3 < 15 we get
17 E < 18, and E can be uniquely determined: E = 17. Thus, the answer is C.

7. (OG) If x, y, and z are three integers, are they consecutive integers?
(1) z x = 2
(2) x < y < z

Explanation:
From statement (1) we know that z x = 2. Nothing is said about y, thus, we cannot make any
inference concerning y. (1) is not sufficient to answer the stem. Then, the answer must be B, C, or
E. Statement (2) alone is again insufficient since it states that x < y < z, but we still dont know
whether z y = 1 and y x = 1. Thus, statement (2) alone is insufficient to answer the stem
question. The answer must be C or E. Statements (1) and (2) together are sufficient since from x < y
< z, z - x = 2 we get z y 1, y x 1, and z x 2. Thus, z y = 1 and y - x = 1. The answer is
C.

A prime number is a positive integer that has exactly two different positive divisors: 1 and itself.
For example, 2, 3, 5, 7, 11, and 13 are prime numbers, but 15 is not, since 15 has four different
positive divisors, 1, 3, 5, and 15.
Note: The number 1 is not a prime number since it has only one positive divisor.
Note: 2 is the only even prime. Indeed, if a prime greater than 2 were even, it would have at least
three different divisors: 1, itself and 2.

8. The sum of prime numbers that are greater than 60 but less than 70 is
(A) 67
(B) 128
(C) 191
(D) 197
(E) 260

Explanation:

!

Your way to Top Business Schools _____________
9

Note that 62, 64, 66, and 68 are divisible by 2; 63, 66, and 69 are divisible by 3; and 65 is divisible
by 5. The only prime numbers between 60 and 70 are 61 and 67, and 61 + 67 = 128. The correct
answer is B.

9. (OG) If positive integers x and y are not both odd, which of the following must be even?
(A) xy
(B) x+y
(C) x-y
(D) x+y+1
(E) 2(x+y)-1

Explanation:
If integers are not both odd, at least one of them is even. xy must be even since x and y are not both
odd. A is the answer. B, C, D, E are wrong since it is unknown whether x and y are both even or
not.

10. (OG) If n is a positive integer, is n odd?
(1) 3n is odd
(2) n + 3 is even

Explanation:
(1) alone is sufficient since the product of integers is odd only when all factors are odd. Odd = 3
n=3 ouu. n is odd.
(2) is also sufficient, since if n + 3 is even. The sum of two integers is even if summands are both
even or both odd. 3 is odd, and n must be odd. D is the answer.

!

Your way to Top Business Schools _____________
10

Divisibility
The fact, that 9 when divided by 3 yields an integer can be expressed in several ways:

9 is divisible by 3
9 is a multiple of 3
3 is a divisor of 9
3 is a factor of 9.

There are three groups of criterions of divisibility:
- Divisibility is determined by last digits of a number (divisibility by 2, 4, 5, 8, 10, );
- Divisibility is determined by algebraic sum of all digits (divisibility by 3, 9 and 11);
- Divisibility is determined by mix of some criterions from first two groups (divisibility by 6, 12, 15,
18, 22).

Lets consider an integer N=a,bcd.

Divisor (N is a
multiple of)
Criterion Examples
2 The last digit d is divisible by 2 84 is a multiple of 2 since 4 is a




multiple of 2.

333 is NOT a multiple of 2 since 3 is
NOT a multiple of 2.
4 Two last digits cd form the integer
divisible by 4
3,124 is a multiple of 4 since 24 is a
multiple of 4.

1,333,334 is NOT a multiple of 4 since
34 is NOT a multiple of 4
8 Three last digits bcd form the
integer divisible by 8
88,863,024 is a multiple of 8 since 24 is
a multiple of 8.

17,723,001 is NOT a multiple of 8
since 1 is NOT a multiple of 8.

In general, an integer is a multiple of 2
h
if its k last digits form the integer divided by 2
h
.
If a number is not divisible by 2
k
, the remainder when the number is divided by 2
k
is the same as
the remainder when the last k digits form the integer divided by 2
k
.

!

Your way to Top Business Schools _____________
11


Examples:
The number 2,346 is not divisible by 4, since the number formed by its last two digits, 46, is not
divisible by 4. When 2,346 is divided by 4, the remainder is 2, since when 46 is divided by 4 the
remainder is 2.
The number 172,045 is not divisible by 8, since the number formed by its last three digits, 45, is not
divisible by 8. When 172,045 is divided by 8, the remainder is 5, since when 45 is divided by 8 the
remainder is 5.

Divisor (N is a
multiple of)
Criterion Examples
3 The sum of all digits a+b+c+d is
divisible by 3
177 is a multiple of 3 since 1+7+7=15
is a multiple of 3.

162,346 is NOT a multiple of 3 since
1+6+2+3+4+6=22 is NOT a multiple of
3.
9 The sum of all digits a+b+c+d is
divisible by 9
1,737 is a multiple of 9 since
1+7+3+7=18 is a multiple of 9.

1,333,334 is NOT a multiple of 9 since
1+3+3+3+3+3+4=20 is NOT a multiple
of 9.

In general, an integer is a multiple of 3
h
if the sum of its digits is divisible by 3
h
, h = 1, 2.
If a number is not divisible by S
k
, the remainder when the number is divided by S
k
is the same as
the remainder when the sum of its digits is divided by S
k
, k = 1,2.

Examples:
The number 3,248 is not divisible by 3, since the sum of its digits, 17, is not divisible by 3. When
3,248 is divided by 3, the remainder is 2, since when 17 is divided by 3 the remainder is 2.
The number 172,345 is not divisible by 9, since the sum of its digits, 22, is not divisible by 9. When
172,345 is divided by 9, the remainder is 4, since when 22 is divided by 9 the remainder is 4.

Divisor (N is a
multiple of)
Criterion Examples
5 The last digit d is 5 or 0 95 is a multiple of 5 since the last digit
is 5.

813 is NOT a multiple of 5 since the
last digit is neither 5 nor 0.

!

Your way to Top Business Schools _____________
12

10 The last digit d is 0 7,120 is a multiple of 10 since the last
digit is 0.

1,333,334 is NOT a multiple of 10
since the last digit is NOT 0.
11 The sum of digits on odd places
minus the sum of digits on even
places ((a+c)-(b+d)) is divisible
by 11

824,472 is a multiple of 11 since
(8+4+7)-(2+4+2)=11 is a multiple of
11.

17,723,001 is NOT a multiple of 11
since (1+7+3+0)-(7+2+0+1)=1 is NOT
a multiple of 11.

If a number is not divisible by 5, the remainder when the number is divided by 5 is the same as the
remainder when the last digit is divided by 5.
If a number is not divisible by 10, the remainder when the number is divided by 10 is the same as
the remainder when the last digit is divided by 10.

Examples:
The number 145,632 is not divisible by 5, since its last digit, 2, is neither 5 nor 0. When 145,632 is
divided by 5, the remainder is 2, since 2 divided by 5 is 0 with a remainder of 2.
The number 7,332,899 is not divisible by 10, since its last digit, 9, is not 0. When 7,332,899 is
divided by 10, the remainder is 9, since 9 divided by 10 is 0 with a remainder of 9.

Divisibility by other integers can be checked using combinations of criterions listed above.

Examples:
Is 9,558 divisible by 6?
Since 6=23, we will use both divisibility tests for 2 and 3. Thus, lets check whether the last digit
of the integer in the question is even and whether the sum of its digits is divisible by 3.
The last digit, 8, is even. The sum of the digits, 9+5+5+8=27, is divisible by 3. Hence, 9,558 is
divisible by 6.
The number 724,560 is divisible by 12, since the number formed by its last two digits, 60, is
divisible by 4, and the sum of its digits is 30, which is a multiple of 3.
The number 7,145,580 is divisible by 15, since its units digit is 0 and the sum of its digits, 30, is
divisible by 3.

1. Which one of the following is divisible by both 2 and 3?
(A) 1005
(B) 1296
(C) 1351
(D) 1406
(E) 1414

!

Your way to Top Business Schools _____________
13


Explanation:
Obviously, A and C cannot be correct answers, since they are odd numbers. Among B, D and E we
choose B, because 1+2+9+6=18, which is divisible by 3. Hence, 1296 itself is divisible by 3.

2. Is a 3-digit integer divisible by 3?
(1) The hundreds' digit of the number is equal to the tens' digit of the number.
(2) The units' digit of the number is 6 greater than the tens' digit of the number

Explanation:
A number is divisible by 3 if the sum of its digits is divisible by 3. So 3-digit integer abc will be
divisible by 3 if a+b+c is a multiple of 3. From the first statement we know, that a=b, therefore
a +b + c = 2b + c, which may or may not be a multiple of 3. Hence, the first statement is not
sufficient.
From the second statement we know, that c=b+6. Therefore, a + b + c = a +2b + 6, which again
may or may not be a multiple of 3. Hence, statement 2 is not sufficient.
Finally, having a=b and c=b+6 we obtain a +b +c = b +b +b +6 = Sb + 6 = S(b + 2) which
is divisible by 3. The correct answer is C.

3. In the number 11,0AB, A and B represent the tens and units digits, respectively. If 11,0AB is
divisible by 55, what is the greatest possible value of B A?
(A) 0
(B) 5
(C) 10
(D) 15
(E) 25

Explanation:
11,0AB is divisible by 55, hence, it is divisible by 5 and by 11. Divisibility by 11 means, that
(1+0+B)-(1+A) = B-A is divisible by 11. As A and B are digits, the only possible value for B-A is
0. Hence, A=B. Additionally, 11,0AB is divisible by 5, which means that B is 0 or 5. As we are
asked about the greatest possible value, we choose B=A=5. AB=25. The best answer is E.

!

Your way to Top Business Schools _____________
14

Decimals and Fractions

GMAT math goes beyond an understanding of the properties of integers or whole numbers. Lets
consider the numbers that fall in between the integers. They can be expressed by
Fractions or decimals.

Fractions
In a fraction
n
d
n is the numerator and d is the denominator.
Note: The denominator of a fraction can never be 0, because division by zero is not defined.

Simplifying a fraction
The process of simplifying (or cancelling) is governed by one simple rule:
MULTIPLYING or DIVIDING both the numerator and the denominator by the same number does
not change the value of the fraction.

Example:
2u
1S
=
2u S
1S S
=
4
S

12
6u
=
12 12
6u 12
=
1
S

1. (OG) (8
2
)(3
3
)(2
4
)/96
2
=
(A) 3
(B) 6
(C) 9
(D) 12
(E) 18

Explanation:
Since 96 = 2
5
3
1
, 96
2
= 2
10
3
2
, and 8
2
3
3
2
4
= 2
10
3
3
, (8
2
)(3
3
)(2
4
)/96
2
= 2
10
3
3
/(2
10
3
2
) = 3. The
answer is A.

2. (GC) If a/b = 9/11, what is the value of ab?
(1) a and b are positive integers
(2) GCD(a, b) = 3
Here GCD(a, b) denotes greatest common divisor of two integers a and b.

Explanation:
Clearly, (1) alone is not sufficient to answer the stem question. Thus, the answer must be B, C, or E.
Statement (2) implies that both a and b are positive integers (otherwise, GCD(a, b) cannot be
defined) and these positive integers have greatest common divisor 3. Therefore, a = 3k, b = 3m,
where k and m are mutually prime (i.e., their GCD equals to 1 and they do not have common
divisors), and k/m = 9/11, or, equivalently, k = 9 and m = 11. Thus, a = 27, b = 33 and ab = 2733 =
900 9 = 891.

Adding and Subtracting fractions

!

Your way to Top Business Schools _____________
15

In order to add or subtract fractions you must
a. Find a common denominator a number, divisible by each denominator of given fractions.
b. Change each fraction so, that it is expressed using this common denominator.
c. Add or subtract the numerators only.

Example:
3
8
+
7
12
A common denominator is 24. Thus,
3
8
=
9
24
and
7
12
=
14
24

9
24
+
14
24
Express each fraction using the common denominator 24.
9
24
+
14
24
=
23
24
Finally, add the numerators to find the answer.

Example:
11
15

7
30
A common denominator is 30. Thus,
11
15
=
22
30
and
7
30
stays the same.
22
30

7
30
Express each fraction using the common denominator 30.
22
30

7
30
=
15
30
Subtract the numerators.
15
30
=
1
2
Simplify
15
30
to find the answer.

3. (OG) 1 3/7 1/3 =
(A) 3/5
(B) 6/7
(C) 2/21
(D) 5/21
(E) 16/21

Explanation:
Since 3/7 + 1/3 = (9 + 7)/21 = 16/21, 1 3/7 1/3 = 5/21. The answer is D.

4. (OG) A straight pipe 1 yard in length was marked off in fourths and also in thirds. If the pipe was
then cut into separate pieces at each of these markings, which of the following gives all the different
lengths of the pieces, in fractions of a yard?
(A) 1/6 and 1/4 only
(B) 1/4 and 1/3 only
(C) 1/6, 1/4 and 1/3 only
(D) 1/12, 1/6 and 1/4
(E) 1/12, 1/6 and 1/3

Explanation:
1/4 is necessarily one of the answers. Thus, answer choices A, C, and D are under consideration.
Choice C is wrong since no pieces of length 1/3 exist (length of every piece does not exceed 1/4, the
minimal marking). Piece 1/12 necessarily belongs to the answer choice set (it corresponds to [1/4,
1/3] markings). The answer is D.

Multiplication of fractions
To multiply fractions, first multiply the numerators together, multiply the denominators together.

!

Your way to Top Business Schools _____________
16

Example:
8
12

15
32
=
815
1232
=
4253
43216
=
5
16


Dividing fractions
In order to divide fractions we should first learn what a reciprocal is.
The reciprocal to
3
4
is
4
3

The reciprocal to
7
9
is
9
7

The reciprocal to 5=
5
1
is
1
5

To check if you have found the reciprocal of a number use the rule:
The product of a number and its reciprocal always equals 1.

In order to divide fractions
a. Change the divisor into its reciprocal. (The divisor is the second number.)
b. Multiply the fractions.

Example:
S
4

7
11
=
S
4

11
7
=
S 11
4 7
=
SS
28

Ratio and Proportion
The ratio of the number a to the number b (b 0) is
a
b
.
A ratio may be expressed or represented in several ways. For example, the ratio of 2 to 3 can be
written as 2 to 3, 2:3, or
2
3
. The order of the terms of a ratio is important.

Example:
The ratio of the number of months with exactly 30 days to the number with exactly 31 days is
4
7
not
7
4
.
A proportion is a statement that two ratios are equal.

Example:
2
3
=
8
12
is a proportion.
One way to solve a proportion involving an unknown is to cross multiply, obtaining a new equality.

Example:
To solve for n in the proportion
2
S
=
n
12
,
cross multiply, obtaining 24 = Sn; then divide both sides by 3, to get n = 8.

!

Your way to Top Business Schools _____________
17

5. (OG) The ratio of two quantities is 3 to 4. If each of the quantities is increased by 5, what is the
ratio of these two new quantities?
(A) 3/4
(B) 8/9
(C) 18/19
(D) 23/24
(E) It cannot be determined from the information given.

Explanation:
Let the quantities be P and Q. From the stem it can be inferred that P/Q = 3/4. This information is
insufficient to determine the value of (P + 5)/(Q + 5). Thus, the answer is E.

6. (OG) What is the ratio of 3/4 to the product 4(3/4)?
(A) 1/4
(B) 1/3
(C) 4/9
(D) 9/4
(E) 4

Explanation:
(3/4)/(4(3/4)) = 3/(43) = 1/4. A is the answer.
7. (OG) In which of the following pairs are the two numbers reciprocals to each other?
I. 3 and 1/3
II. 1/17 and -1/17
III. 3 and -3








Explanation:
It is evident that 31/3 = 1, therefore, 3 and 1/3 are reciprocals to each other. 1/17 and -1/17 is not a
pair of reciprocals, as is pair {3, -3}. The answer is A.

Comparing fractions
Example:
Compare these pairs of fractions:
a)
5
18
anu
11
18

b)
3
7
anu
3
9

c)
5
18
anu
3
4


In a) we notice, that the fractions have the same denominator 18.
(A) I only
(B) II only
(C) I and II
(D) I and III
(E) II and III

!

Your way to Top Business Schools _____________
18

Among the fractions with equal denominators the greatest will be the one with the greatest
numerator.
Thus, in our case,
5
18
<
11
18

In b) we notice that the fractions have the same numerator 3.
Among the fractions with equal numerators the greatest will be the one with the least numerator.
Thus, in our case,
3
7
>
3
9

In c) neither the denominators, nor the numerators are equal. After we find the common
denominator for these two fractions we will be able to reduce the problem to case a).
The least common denominator for the given fractions is 36. Thus,
S
18
=
1u
S6
anu
S
4
=
27
S6

Finally,
10
36
<
27
36
and
5
18
<
3
4
.

8. (OG) Which of the following is greater than 2/3?
(A) 33/50
(B) 8/11
(C) 3/5
(D) 13/27
(E) 5/8

Explanation:
33/50 < 2/3 since 99 < 100; 8/11 > 2/3 as 24 > 22; 3/5 < 2/3 as 9 < 10; 13/27 < 2/3 is equivalent to
39 < 54; 5/8 < 2/3 since 15 < 16. Moreover, 33/50 is closest to 2/3 among numbers from the answer
choices that are less than 2/3. B is the answer.

9. (GC) Which one of the five factors is largest?
(A) 25038876541/25038876543
(B) 25038876543/25038876545

(C) 25038876545/25038876547
(D) 25038876547/25038876549
(E) 25038876549/25038876551

Explanation:
We can see that in any fraction the difference between denominator and numerator is 2, so each
fraction can be represented as x/(x+2).
Evidently, 1/3 < 2/4=1/2. In general x/(x + 2) < (x + 2)/(x + 4) if x > 0. Thus, A < B < C , D < E. E
is the answer.

Decimals
Any number can be expressed as a decimal.
Example:
5 = 5.0

!

Your way to Top Business Schools _____________
19

S
S
= u.6
1
S
= u.SSSS = u. (S)
Note: On the GMAT it is the decimal point that determines the place value of the digit, not comma.
Example: Here are some examples of expressing decimals and fractions through each other.
u.S21 =
S21
1uuu

1.S6 =
1S6
1uu

S
8
= S 8 = u.S7S

All decimals are divided into three groups:
- Terminating (they have only a finite number of non-zero digits)
- Periodic (their digits are periodically repeated)
- Infinite

The fraction expressed as a decimal will be the terminating decimal if it can be presented as
a
2
n
5
m
,
where a is an integer, m=0,1,2,3... , and n=0,1,2,3... .
The fraction will be converted into the periodic decimal if its denominator equals, for example 3, 6,
9, 11, 99.

10. (OG) 3.003/2.002 =


Explanation:
3.003/2.002 = (31.001)/(21.001) = 3/2 = 1.5. E is the answer.

11. (OG) Any decimal that has only a finite number of non-zero digits is a terminating decimal. If r and
s are positive integers and the ratio r/s is expressed as a decimal, is r/s a terminating decimal?
(1) 90 < r < 100
(2) s = 4

Explanation:
Any fraction r/s, where r and s are mutually prime integers and s = 2
k
5
m
, can be expressed as a
terminating decimal. This fact is related to the properties of decimal notation. Thus, the answer is B.

12. (OG) If each of the following fractions were written as a repeating decimal, which would have the
longest sequence of different digits?
(A) 2/11
(B) 1/3
(A) 1.05
(B) 1.50015
(C) 1.501
(D) 1.5015
(E) 1.5

!

Your way to Top Business Schools _____________
20

(C) 41/99
(D) 2/3
(E) 23/37

Explanation:
If we divide numerator by denominator in each fraction, will find that
2/11 = 0.(18), 1/3 = 0.(3), 41/99 = 0.(41), 2/3 = 0.(6), 23/37 = 0.(621). Thus, the answer is E.

13. (GC) S = 0.abc, where a, b and c are any decimal digits. Is S > 2/3?
(1) a + b > 14
(2) a + c > 15

Explanation:
From statement (1) we know that a + b 15 and if a > 6 (1) is sufficient to answer the question
positively. Otherwise, a = 6 and b must be equal to 9. And again, from (1) it can be inferred that S >
2/3. Thus, the answer must be A or D. From statement (2) we have a + b 16 or a 16 b 7.
Thus, (2) is sufficient for S to be greater than 2/3. The answer is D.

Percent
Percent means per hundred or number out of 100. A percent can be represented as a fraction with a
denominator of 100, or as a decimal.

Example:
37% =
37
100
= u.S7
To nd a certain percent of a number, multiply the number by the percent expressed as a decimal or
fraction.

Example:
2u% of 9u = u.2 9u = 18
7S% of 2uu =
7S
1uu
2uu =
S
4
2uu = 1Su

14. (OG PS 33) What is 45% of 7/12 of 240?
(A) 63
(B) 90
(C) 108
(D) 140
(E) 311

Explanation:
45%7/12240 = 45%140 = 45/100140 = 457/5 = 97 = 63. Thus, the answer is A.

15. (OG) What is 15 percent of x?
(1) 18 is 6% of x
(2) 2/3 of x is 200

!

Your way to Top Business Schools _____________
21

Explanation:
From statement (1) we know that 18 = 6%x, x = 18100/6 = 300, 15%x = 45. Therefore, the
answer must be A or D. From statement (2) we can infer that 2/3x = 200 and x = 300, and,
similarly, 15%x = 45. The answer is D.

!

Your way to Top Business Schools _____________
22

Digits
A digit is an integer from 0 to 9 inclusive: {0, 1, 2, 3, 4, 5, 6, 7, 8, 9}.

Every digit in a number has a particular place value depending on its location within the number.
Example: S27 = S 1uu +2 1u + 7 1. Thus, in the number 527 the digit 7 is in the ones
(units) place, the digit 2 is in the tens place, and the digit 5 is in the hundreds place. The name of
each location corresponds to the value of the place. Indeed,
7 is worth seven units, or 7
2 is worth two tens, or 20
5 is worth five hundreds, or 500.

You should memorize the names of all the places value. Lets consider and analyze all the digits in
the number
736,589,325,231.3489.
7

H
U
N
D
R
E
D

B
I
L
L
I
O
N
S
3

T
E
N





B
I
L
L
I
O
N
S
6

O
N
E





B
I
L
L
I
O
N
S
5

H
U
N
D
R
E
D

M
I
L
L
I
O
N
S
8

T
E
N





M
I
L
L
I
O
N
S
9

O
N
E





M
I
L
L
I
O
N
S
3

H
U
N
D
R
E
D

T
H
O
U
S
A
N
D
S
2

T
E
N





T
H
O
U
S
A
N
D
S
5

O
N
E





T
H
O
U
S
A
N
D
S
2

H
U
N
D
R
E
D
S

3

T
E
N
S
1

U
N
I
T
S
.

D
E
C
I
M
A
L

P
O
I
N
T
3

T
E
N
T
H
S
4

H
U
N
D
R
E
D
T
H
S
8

T
H
O
U
S
A
N
D
T
H
S
9

T
E
N



T
H
O
U
S
A
N
D
T
H
S
Generally,
abcu. ef = 1uuua +1uub +1uc +u +
1
1u
e +
1
1uu
f
Note: Notation ab can have two different meanings: a product of a and b or a two-digit number.
If it is given that a and b are integers, than you deal with a product. If a and b are said to be digits
treat ab as a two-digit number.

Consider the following problem:
A and B are both two-digit numbers. If A and B contain the same digits, but in reverse order, what
integer must be a factor of (A+B)?

!

Your way to Top Business Schools _____________
23

To solve this problem, assign two variables to be A and B: x and y. Using your knowledge of place
value, you can express A as 10x+y, where x is the digit in the tens place and y is the digit in the unit
place. B, therefore, can be expressed as 10y+x. Hence, the sum of A and B can be expressed as
follows:
A +B = 1ux +y + 1uy +x = 11x + 11y = 11(x +y).
Clearly, 11 must be a factor of A+B.

1. (OG) If a and b represent single digits in the correctly worked computation below, what is the value
of a +b?
4 a 7
b 2 3
+ 1 6 2
1 2 2 2

Explanation:
Perform this computation. Since 2 + 6 + 1+a = 12, a = 3. Since 4 + b + 1 + 1 = 12, b = 6. Thus, a +
b = 9.

2. (OG) If the two-digit integers M and N are positive and have the same digits, but in reverse order,
which of the following CANNOT be the sum of M and N?
(A) 181
(B) 165
(C) 121
(D) 99
(E) 55

Explanation:
If M = XY = 10X + Y (where X and Y are digits), then N = YX = 10Y + X and M + N = 11(X +
Y). Thus, M + N is divisible by 11. Among answer choices given, only A does not satisfy this
condition. Dont forget about those divisibility shortcuts for 2, 3, 5, 9, 10, and 11!

3. (OG) The product of the two-digit numbers below is the three-digit number ada, where a, b, and d
are three different nonzero digits. If ab < 10, what is the two-digit number ab?
ab
ba
ada

(A) 11
(B) 12
(C) 13
(D) 21
(E) 31

Explanation:
Since ab < 10 and ab=a, solving last equation gives b=1 or a=0. Only b=1 remains as a root,
because a is not equal to 0. Using this information, ab = 21 or 31, the shortest way to check is to
multiply 21 by 12 or 31 by 13: 2112=252. D is the answer.

!

Your way to Top Business Schools _____________
24


4. (GC) If 10
50
74 is written as an integer in decimal notation, what is the sum of the digits in that
integer?
(A) 412
(B) 423
(C) 433
(D) 440
(E) 461

Explanation:
First solution: 10
50
74 = 9999....99926, where the last integer contains 48 9s. Thus, the sum of its
digits equals to 948 + 2 + 6 = 440. The answer is D.

Second solution: The remainder (by 9) of the sum of digits is equal to the remainder (by 9) of the
original number. The remainder of 10
50
74 by 9 is equal to 1 2 = -1 and the only answer choice
with the same remainder is D.
Knowing the place value is important because the GMAT occasionally requires you to round a
number to a specific place value.

Example:
What is 3.592 rounded to the nearest tenths?
First, find the digit located in the specified place value. The digit 5 is in the tenths place.
Second, look at the right-digit-neighbour (the digit immediately to the right) of the digit in question.
In this case we have 9.
If the right-digit-neighbour if 5 or greater, round the digit in question UP. Otherwise, the digit in
question remains the same.
In this case since 9 is greater than 5, the digit in question (5) must be rounded up to 6. Thus, 3.592
rounded to the nearest tenth equals 3.6.

Note: all the digits to the right of the right-digit-neighbour are irrelevant when rounding. In the
example the digit 2 is not important.

5. (OG) If d = 2.0453 and d* is the decimal obtained by rounding d to the nearest hundredth, what is
the value of d* - d?
(A) -0.0053
(B) -0.0003
(C) 0.0007
(D) 0.0047
(E) 0.0153

Explanation:
d* = 2.05, d* - d = 0.0047. Thus, the answer is D.


!

Your way to Top Business Schools _____________
25

Powers and Roots
Real number r is said to be a square root of real number x if i
2
= x.

Note: only non-negative real numbers have square roots; 0 has only one square root, while any
positive real number has two square roots (one negative and one positive).
Positive root of a real number is sometimes called arithmetic square root.

Examples: 4 = 2, S6 = 6, 144 = 12, 441 = 21, etc.
You should be pretty familiar with following properties of powers and roots:
a
n
a
m
= a
m+n

a
n
: a
m
= a
n-m

(a
m
)
n
= a
mn

a
n
b
n
= (
a
b
)
n

a
-n
=
1
a
n

a
1
n
= a
n

a
n
m
= a
n
m

a
n
m
= (a
n
)
1
m
= a
n
m
= (a
1
m
)
n
= ( a
m
)
n


Note:
- a
1
= a
- a
0
= 1, a u
- u
0
unuefineu
- 1
n
= 1
- a
2
= |a|
- u
n
= u, n > u

1. (OG) (16 2u + 8 S2) =
(A) 420
(B) 24
(C) 25
(D) 420 + 82
(E) 32

Explanation:
Since 1620 + 832 = 576 = 24
2
, (16 2u + 8 S2) = 24. The answer is B. But it is easier to
split off expression inside : 1620 + 832 = 16(20 + 16) = 1636 = 4
2
6
2
= (46)
2
.

2. (OG) How many bits of computer memory will be required to store the integer x, where x = -
810,000, if each digit requires 4 bits of memory and the sign of x requires 1 bit?
(A) 24
(B) 25
(C) 17
(D) 13

!

Your way to Top Business Schools _____________
26

(E) 12
Explanation:
810,000 = 8110000 = 9
2
100
2
= 900
2
. Therefore, x = 81u,uuu = 9uu
2
= 9uu and it
takes 4 bits for each of the digits and one bit for sign -: 34 + 1 = 13 bits to store the information.
Thus, the answer is D.

3. (OG) 16 + 16 =
(A) 42
(B) 82
(C) 162
(D) 8
(E) 16

Explanation:
16 + 16 = 162 = 2
4
2. Therefore, (16 + 16) = (2
4
2) = 2
2
2 = 42. A is the answer.
Following problems will give you deeper insight into conditions under which roots are integers.

4. (OG) If d is a positive integer, is u an integer?
(1) d is the square of an integer
(2) u is the square of an integer

Explanation:
Statement (2) alone is sufficient: the square of an integer is always an integer. (1) is sufficient too,
since there exist some positive p such that d = p
2
and d = p. The answer is D.

5. (OG) Is the positive square root of x an integer?
(1) x = n
4
and n is an integer
(2) x = 16

Explanation:
Clearly, (1) alone is sufficient: x = n
2
and n
2
is an integer. (1) is sufficient too, since 16 = 4. The
answer is D.

6. (OG) If x is an integer, is x an integer?
(1) 4x is an integer
(2) Sx is not an integer

Explanation:
(1) is sufficient to answer the stem question: 4x = 2x is an integer and thus x must be integer.
(2) is not sufficient (corresponding examples are x = 1 and x = 3). A is the answer.
You should remember approximations of roots:
2 1. 41, 3 1. 73, 5 2. 2, 7 2.

7. (OG) The value of89
3
is between
(A) -9 and -10
(B) -8 and -9
(C) -4 and -5

!

Your way to Top Business Schools _____________
27

(D) -3 and -4
(E) undefined

Explanation:
Since4 = 64
3
> 89
3
> 12S
3
= S, C is the answer.

8. (OG) The expression 61.24(0.998)
2
/4uS is approximately equal to
(A) 4
(B) 3
(C) 2
(D) 5
(E) 6

Explanation:
When you are asked to find approximate solution it is easier to round factors than round the result.
We know that 0.998 is close to 1, 4uS is close to 20, and the result is close to 3. The answer is B.

9. (OG ) Of the following numbers which one is third greatest?
(A) 22 - 1
(B) 2 + 1
(C) 1 - 2
(D) 2 - 1
(E) 2

Explanation:
It is evident that 1 < 2 < 2. Having this, we can prove that 22 1 > 2 + 1 > 2 > 2 1 > 1 -
2. Also we can use here approximation of 2 1.41. The answer is E.

Formulas of short multiplication:
a
2
b
2
= (a b)(a + b)
(a + b)
2
= a
2
+2ab +b
2

(a b)
2
= a
2
2ab +b
2

a
3
+ b
3
= (a +b)(a
2
ab +b
2
)
a
3
b
3
= (a b)(a
2
+ ab +b
2
)
a
3
+b
3
+ c
3
+ u
3
= (a + b + c +u)()
(a +b)
3
= a
3
+Sa
2
b +Sab
2
+ b
3

(a b)
3
= a
3
Sa
2
b +Sab
2
b
3


10. (OG) (1 - S)(1 + S) =
(A) -4
(B) 2
(C) 6
(D) 2S
(E) -2S

Explanation:
If we use first formula of short multiplication then (1 - S)(1 + S) = 1
2
(S)
2
=-4. A is the
answer.

!

Your way to Top Business Schools _____________
28

11. (GC) 1/(S - 2) =
(A) S + 2
(B) S - 2
(C) 1
(D) 1- S
(E) 1 2

Explanation:
Here roots are in the denominator, and if we need rational numbers in denominator formula of short
multiplication will be useful: (S 2)(S + 2) = S 2 = 1. Lets multiply numerator and
denominator by (S +2):
1
3-2
=
3+2
(3-2)(3+2)
=
3+2
3-2
= S +2. A is the answer.

12. (OG) (7 + 7)
2
=
(A) 28
(B) 14
(C) 49
(D) 98
(E) 7

Explanation:
7 + 7 = 27 and (27)
2
= 4 7 = 28. Or we can use the formula: (7 + 7)
2
= 7
2
+
277 +7
2
= 7 + 2 7 + 7 = 28
A is the answer.
More complex problems may appear on the GMAT. Consider following example:

13.

6 +2S =
(A)
S
(B)
S 1
(C)
S+ 1
(D)
3 - S
(E) 3

Explanation:
The key to such problems is to find that 6 +2S = (1 +S) +2 1 S = 1
2
+2 1 S +
S
2
= (1 +S)
2
. Having this, we can find that C is the answer.

!

Your way to Top Business Schools _____________
29

Home assignment
I. Numbers.

1. If x is an integer, is y an integer?
(1) The average (arithmetic mean) of x, y, and y 2 is x
(2) The average (arithmetic mean) of x and y is NOT an integer

2. If n is a positive integer, what is the maximum possible number of prime numbers in the
following sequences: n + 1, n + 2, n + 3, n + 4, n + 5, and n + 6?
(A) 2
(B) 3
(C) 4
(D) 5
(E) 6

3. If n is an integer greater than 1, and n is not a prime number, then which of the following
must be true?
(A) n is the sum of three prime numbers
(B) n is the difference between 2 even numbers
(C) n is the difference between one even number and one odd number
(D) n is the product of one even number and one odd number
(E) n is the product of prime numbers

4. If x, and z are positive integers such that
0 < x < y < z and x is even, is odd, and z is prime, which of the following is a possible value of
x + + z ?
(A) 4
(B) 5
(C) 11
(D) 15
(E) 18

5. If a and b are integers such that a + b = 5, which of the following must be true?
I. The product of a and b is odd.
II. If a is odd, b is even.
III. If a is negative, b is positive.
(A) I only
(B) II only
(C) I and II only
(D) II and III only
(E) I, II, and III

6. If the sum of five consecutive positive integers is a, then the sum of the next five
consecutive integers in terms of a is
(A) a + 1
(B) a + 5
(C) + 25

!

Your way to Top Business Schools _____________
30

(D) 2a
(E) 5a

7. If S is a series of x consecutive odd integers, and z is the largest integer in S, what is the
smallest integer in S?
(A) z 2x
(B) z x + 1
(C) z 2(x 1)
(D) z x + 2
(E) z
x
2


8. (OG) If p is an even integer and q is an odd integer, which of the following must be an odd
integer?
(A) p/q
(B) pq
(C) 2p+q
(D) 2(p+q)
(E) 3p/q

9. (OG) If r and s are integers and rs + r is odd, which of the following must be even?
(A) r
(B) s
(C) r+s
(D) rs-r
(E) r*r+s

10. (OG) If m is an integer, is m odd?
(1) m/2 is not an even integer
(2) m - 3 is an even integer

11. (OG) Is x an integer?
(1) x/2 is an integer
(2) 2x is an integer

12. (GC) Is (x
2
+
2
) a square of an integer?
(1) x is even, is odd
(2) x and are odd

13. (GC) Are integers a and b both even?
(1) ab is evenly divisible by 4
(2) ab = 0

14. If n is an even number, which of the following must be even?
I.
4
4 4 + n

II.
4
2 4
2
n n +

III.
4
4 4
2
n n +

!

Your way to Top Business Schools _____________
31

(A) I only
(B) II only
(C) III only
(D) I and III only
(E) II and III only

15. If x is a positive integer, is x a prime number?
(1) x + 1 is a prime number.
(2) x 5 is a prime number

II. Divisibility

1. (GC ) Let N be the greatest integer multiple of 8, no two of those digits are the same. What
is the remainder when N is divided by 1000?
(A) 120
(B) 12
(C) 320
(D) 240
(E) 210

2. (GC ) What is the sum of the least and the greatest positive four-digit multiples of 4 that can
be written each using the digits 1, 2, 3, and 4 exactly once?
(A) 5555
(B) 5658
(C) 5636
(D) 4312
(E) 1324

3. & represents the tens digit in integer A=1543&2. What is &?
(1) A is divisible by 9
(2) A is divisible by 4

4. Which of the following is the smallest number that can be divisible by 6?
(A) 102,000
(B) 101,010
(C) 110,100
(D) 100,010
(E) 110,010

5. If l23 n is divisible by 990, what is the least possible value of n?
(A) 10
(B) 11
(C) 12
(D) 18
(E) 22

!

Your way to Top Business Schools _____________
32

6. If x and y are positive integers, is x y divisible by 4?
(1) xy is divisible by 16.
(2) x is divisible by 4.

III. Decimals and fractions

1. (OG) Which of the following fractions has greatest value?
(A) 6/(2
2
5
2
)
(B) 1/(2
3
5
2
)
(C) 28/(2
2
5
3
)
(D) 62/(2
3
5
3
)
(E) 122/(2
4
5
3
)

2. (OG) If the numbers 17/24, 1/2, 3/8, 3/4, and 9/16 were ordered from greatest to least, the
middle number of the resulting sequence would be
(A) 17/24
(B) 1/2
(C) 3/8
(D) 3/4
(E)

9/16
3. (OG) On the number line segment [0, 1] has been divided by tick marks into fifths and into
sevenths What is the least possible distance between any two of the tick marks?
(A) 1/70
(B) 1/35
(C) 2/35
(D) 1/12
(E) 1/7

4. (OG)If x is equal to one of the numbers 1/4, 3/8, or 2/5, what is the value of x?
(1) 1/4 < x < 1/2
(2) 1/3 < x < 3/5

5. (OG) In the fraction x/y, where x and y are positive integers, what is the value of y?
(1) The least common denominator of x/y and 1/3 is 6
(2) x = 1

6. (GC) Find the value of (p-1)
3
if p = 15/18 + 5/16 - 20/24.
(A) [
11
16

3

(B) [
11
16

3

(C) [
20
48

3

(D) [
20
48

3

(E) [
7
24

!

Your way to Top Business Schools _____________
33

7. (OG) What number when multiplied by 4/7 yields 6/7 as the result?
(A) 2/7
(B) 2/3
(C) 3/2
(D) 24/7
(E) 7/2

8. (OG) If the successive marks on the line below are equally spaced and if x and y are the
numbers designating the end points of intervals as shown, what is the value of y?

(1) x = 1/2
(2) y x = 2/3

9. (OG) The number 2 0.5 is how many times the number 1 0.5?
(A) 3
(B) 6
(C) 1.5
(D) 0.5
(E) 2.5

10. (OG) What is the value of (-1.51.2 4.50.4)/30?
(A) 0
(B) -0.12
(C) 0.12
(D) 1.2
(E) -0.012

11. (OG) What is the value of (0.3)
5
/(0.3)
3
?
(A) 0.03
(B) 0.09
(C) 0.27
(D) 0.003
(E) 0.009

12. (OG) 30.072/0.54 =
(A) 0.04
(B) 0.3
(C) 0.4
(D) 0.8
(E) 4

13. (OG) If 1 < d < 2, is the tenths digit of the decimal representation of d equal to 9?
(1) d + 0.01 < 2
(2) d + 0.05 > 2

!

Your way to Top Business Schools _____________
34

14. (OG) What is the decimal equivalent of (1/5)
5
?
(A) 0.00032
(B) 0.0016
(C) 0.00625
(D) 0.008
(E) 0.03125

15. (OG) 3/100 + 5/1000 + 7/100000 =
(A) 0.357
(B) 0.3507
(C) 0.35007
(D) 0.0357
(E) 0.03507

16. (OG) Which of the following fractions is equal to 0.0625?
(A) 5/8
(B) 3/8
(C) 1/16
(D) 1/18
(E) 3/80

17. (OG) 31/125 =
(A) 0.248
(B) 0.252
(C) 0.284
(D) 0.312
(E) 0.320

18. (OG) What percent of 30 is 12?
(A) 2.5%
(B) 3.6%
(C) 25%
(D) 40%
(E) 250%

19. (OG) If 18 is 15% of 30% of a certain number, what is the number?
(A) 9
(B) 36
(C) 40
(D) 81
(E) 400

20. (OG) If x > 0, x/25 + x/50 is what percent of x?
(A) 6%

!

Your way to Top Business Schools _____________
35

(B) 25%
(C) 37.5%
(D) 60%
(E) 75%

21. (OG) If r and s are positive integers, r is what percent of s?
(1) r = 3/4s
(2) r + s = 75/100

22. (OG) The manager of a theatre noted that for every 10 admission tickets sold, the theater sell 3
bags of popcorn at $2.25 each, 4 sodas at $1.50 each, and 2 candy bars at $1.00 each. To the nearest
cent, what is the average (arithmetic mean) amount of these snack sales per ticket sold?
(A) $1.48
(B) $1.58
(C) $1.60
(D) $1.64
(E) $1.70

23. If a and b are integers, is
b
a
a terminate decimal?
(1) a = 5
r
, where r is an integer.
(2) b = 2
s
, where s is an integer.

IV. Digits

1. (GC) The sum of the digits of a standard two-digit numeral is 9. If the digits are reversed, the
numeral represents a number that is 45 less than original number. What is the original numeral?
(A) 72
(B) 27
(C) 54
(D) 63
(E) 81

2. (GC) A person travels in a car with uniform speed. He observes a mile-stone, which has 2 digits.
After one hour he observes another mile-stone with same digits reversed. After another hour he
observes another mile-stone with same 2 digits containing a 0. Find the speed of the car. (Mile-
stones tell you how far you are from your destination OR how far one has travelled from the
starting point).
(A) 40
(B) 45
(C) 50
(D) 55
(E) 60

3. (GC) A, B, C are digits where AB is not zero. What's B?
(1) AB+BA = AAC
(2) A=1

!

Your way to Top Business Schools _____________
36


4. (GC) M is a positive integer less than 50. The units digit of M is 5. What is the value of M?
(1) Thousands digit of M
2
is 1
(2) Hundreds digit of M
2
is 2

5. (GC) S = 0.abc, where a, b, and c are any decimal digits. Is S > 2/3?
(1) a + b > 14
(2) a + c > 15

6. (GC) x, y, z and w are distinct decimal digits. Is 0.xy+0.zw > 1?
(1) The smallest among the 4 digits is 4
(2) The product of the decimals 0.xy and 0.zw is greater than 1/2

7. (OG) If Q is an integer between 10 and 100, what is the value of Q?
(1) One of Qs digits is 3 more than the other, and the sum of its digits is 9
(2) Q < 50

8. (OG) A cashier mentally reversed the digits of one customers correct amount of change and thus
gave the customer an incorrect amount of change. If the cash register contained 45 cents more
than it should have as a result of this error, which of the following could have been the correct
amount of change in cents?
(A) 14
(B) 45
(C) 54
(D) 65
(E) 83

9. (OG) If x = 0.rstu, where r, s, t, and u each represent a nonzero digit of x, what is the value of x?
(1) r = 3s = 2t = 6u
(2) The product of r and u is equal to the product of s and t

10. (OG) What is the value of the two-digit integer x?
(1) The sum of the two digits is 3
(2) x is divisible by 3

11. (OG) If 1 < d < 2, is the tenths digit of the decimal representation of d equal to 9?
(1) d + 0.01 < 2
(2) d + 0.05 > 2

12. (OG) If a and b each represent single digits in the decimal 3.2ab6, what does digit a represent?
(1) When the decimal is rounded to the nearest tenth, 3.2 is the result
(2) When the decimal is rounded to the nearest hundredth, 3.24 is the result

13. (OG) If digit h is the hundredths digit in the decimal d = 0.2h6, what is the value of d rounded to
the nearest tenth?
(1) d < 1/4
(2) h < 5


!

Your way to Top Business Schools _____________
37


V. Powers and roots

1. (GC ) What is the value of x +24 - x?
(1) x is an integer
(2) x +24 is an integer

2. (OG ) Of the following, the closest approximation to _
5.98601.5
15.79
is
(A) 5
(B) 15
(C) 20
(D) 25
(E) 225

3. (OG) 4SS is between
(A) 21 and 22
(B) 22 and 23
(C) 23 and 24
(D) 24 and 25
(E) 25 and 26

4. (OG) What is the value of the positive integer n?
(1) n
4
< 25
(2) n n
2

5. (OG) (2 + 26)/2 =
(A)
6
(B)
26
(C)
1 + 6
(D)
1 + 26
(E)
2 + 6

6. (OG) (2 + 1)( 2 - 1)( S + 1)( S 1) =
(A) 2
(B) 3
(C)
26
(D) 5
(E) 6

7. (OG) (S + 2)( S - 2) =
(A)
S - 4
(B)
6 - 4

!

Your way to Top Business Schools _____________
38

(C) -1
(D) 1
(E) 2

8. 16 2u +8 S2 =
(A) 42u
(B) 24
(C) 25
(D) 42u +82
(E) 32

9. (GC) If x and y are integers, is k the square of an integer?
(1) k = x
2
y
2

(2) k = 4

10. (GC 6344) Find the value of

S
48
3

(A) S
8

(B) S
16

(C) S
24

(D) S
43

(E) S
6


11. Which of the following is not equal to the square of integer?
(A)

1
(B) 4
(C) 18/2
(D) 41-25
(E) 36

12. Which of the following ratios is most nearly equal to the ratio 1 +S to 2?
(A) 8 to 5
(B) 6 to 5
(C) 5 to 4
(D) 2 to 1
(E) 1 to 1

13. What is the smallest integer n for which 2S
n
> S
12
?
(A) 6
(B) 7
(C) 8
(D) 9
(E) 10

!

Your way to Top Business Schools _____________
39

Test1. Number properties, fractions, digits, powers and roots, ratios, percent,
divisibility

1. If , then N is between
(A) 0 and
(B) and
(C) and
(D) and
(E) and 2
2. When x is which of the following numbers,
x
|
.
|

\
|

3
1
has the largest value?
(A) 5
(B) 4
(C) 0
(D) 4
(E) 5

3. What is the sum of digits of number 10
28
28?
(A) 227
(B) 228
(C) 236
(D) 237
(E) 243

4. If m, n, p, and q are consecutive positive even numbers, m < n < p < q, in terms of m, what is
the sum of four numbers?
(A) 4m + 3
(B) 4m + 6
(C) 4m + 8
(D) 4m + 12
(E) 4m + 16

5. There are 135 employees in Company X. If the number of employees who birth in June is the
greatest, what is the least possible number of the employees in the company who birth in June?
(A) 10
(B) 11
(C) 12
(D) 13
(E) 14
3 2
3
1
3
1
3
1
|
.
|

\
|
+ |
.
|

\
|
+ = N
9
1
9
1
3
1
3
1
9
8
9
8
3
4
3
4

!

Your way to Top Business Schools _____________
40


6. For positive integer x, if x
2
has 4 digits, which of the following must be true?
I. x must be a 2-digit integer
II. 2x must be a 3-digit integer
III. 3x must be a 3-digit integer
(A) I only
(B) II only
(C) III only
(D) I and II only
(E) I and III only

7. In a certain positive two-digit integer, the ratio of the units digit to the tens digit is 2 to 3, what
is the integer?
(1) The tens digit is 3 more than the units digit.
(2) The product of the two digits is 54.

8. For numbers x, y, z, and w, if x is 50% greater than y, and z is 20% greater than w, is what
percent greater or less than ?
(A) 20 % greater
(B) 20 % less
(C) 25 % greater
(D) 25 % less
(E) 50 % greater

9. If x
n
= l, where x and n are both integers, what is the value of the x?
(1) n is a multiple of 5.
(2) n is an odd number.

10. The ratio of number of teachers to number of students in school X is equal to that of school Y.
What is the ratio of number of students in school X to number of students in school Y?
(1) The number of the students in school X is 2,000 more than the number of the students in
school Y.
(2) The ratio of number of teachers to number of students in school Y is 1:20.

11. If a and b are positive integers, what is the value of ab?
(1) ab is divisible by 6.
(2) a and b are prime numbers.

12. If a and b are positive integers, is a even?
(1) ab + b is an even number.
(2) ab + a is an even number.

13. Is 49 in the set S?
(1) All numbers in set S are multiple of 7.
(2) All numbers in set S are square numbers.
y
x
w
z

!

Your way to Top Business Schools _____________
41


14. If z and x are integers with absolute values greater than 1, is z
x
less than 1?
(1) x < 0.
(2) z
z
< 1.

15. If the product of digits of a 2-digit positive integer n is 20, what is the value of n?
(1) n is greater than 50.
(2) n is an even number.

16. Integer n is greater than 20 and less than 80, is n odd?
(1) The two digits of n both are prime numbers.
(2) The sum of two digits is prime number.

17. If m and n are positive integers and m + n is prime, is m odd?
(1) n is an odd number.
(2) n is greater than 2.

18. If x < y < z, is xyz > 0?
(1) xy > 0.
(2) xz > 0.

19. If m and n are integers, is an infinite decimal?
(1) m is a prime factor of 100.
(2) n is a prime factor of 50.


Answers:
1. C
2. B
3. E
4. D
5. D
6. A
7. D
8. C
9. B
10. E











11. C
12. E
13. E
14. A
15. D
16. E
17. C
18. E
19. B
n
m

!

Your way to Top Business Schools _____________
42

Lesson 2
Progressions
An arithmetic progression (sequence) is a sequence in which each term after the first is equal to
the sum of the preceding term and a constant. This constant is called progression difference.
If given any two members of arithmetic progression any other member can be uniquely determined.

Examples: 1, 3, 5, 7, 9, 11 is an example; 5, 4, 3, 2 could serve as another example of arithmetic
progression.
Sample problems:
1. (OG) If p, q, r, s, t is an arithmetic progression, which of the following must be also arithmetic
progression?
I. 2p, 2q, 2r, 2s, 2t
II. p 3, q 3, r 3, s 3, t - 3
III. p
2
, q
2
, r
2
, s
2
, t
2

(A) I only
(B) II only
(C) III only
(D) I and II
(E) II and III

Explanation:
Difference between any two consequent numbers from I is constant. Thus, I is an example of
arithmetic progression. The same is true for II. III is not necessarily true (it is easy to see given the
example: p = 1, t = 5). Thus, D is the answer.

Sometimes it is necessary to find length of progression. Length of progression can be easily found
if you consider
n =
(x
n
x
1
)
d
+ 1.
Here d denotes difference and x
i
denotes i-th member of progression.

Sample problems:
2. (OG) How many multiples of 4 are there between 12 and 96, inclusive?
(A) 21
(B) 22
(C) 23
(D) 24
(E) 25

Explanation:
It is easy to see that multiples of 4 constitute arithmetic progression with difference d = 4. Thus, n =
(96 - 12)/4 + 1 = 22. B is the answer.

3. (OG) How many integers n are there such that 1 < 5n + 5 < 25?
(A) 5
(B) 4
(C) 3

!

Your way to Top Business Schools _____________
43

(D) 2
(E) 1

Explanation:
1 < 5n + 5 < 25,
-4 < 5n < 20,
-0.8 < n < 4,
n = 0..3, there are 4 possible values for n. B is the answer.

4. (GC) How many integers between 324700 and 458600 have tens digit 1 and units digit 3?
(A) 10300
(B) 10030
(C) 1353
(D) 1352
(E) 1339

Explanation:
For example, consequent members of this progression are 324713 and 324813, and we can find out
that numbers with given tens and units digits constitute arithmetic progression with difference d =
100.The first member of this progression is 324713, the last member is 458513. Thus, the
progression has (458,513 324,713)/100 + 1 = 4585 3247 + 1 = 1339 members.

Total sum of progression is equal to
S
n
=
x
1
+x
n
2
n, S
n
=
2x
1
+d(n-1)
2
n

Sample problems:
5. (OG ) If x is equal to the sum of the even integers from 40 to 60, inclusive, and y is the number of
even integers from 40 to 60, inclusive, what is the value of x + y?
(A) 550
(B) 551
(C) 560
(D) 561
(E) 572

Explanation:
Even integers from 40 to 60, inclusive, constitute arithmetic progression with difference d = 2. Then
y = (60 40)/2 + 1 = 11. X = (40 + 60)11/2 = 550. Thus, x + y = 561.

6. (OG) What is the sum of the integers in the table below?
1 2 3 4 5 6 7
-2 -4 -6 -8 -10 -12 -14
3 6 9 12 15 18 21
-4 -8 -12 -16 -20 -24 -28
5 10 15 20 25 30 35
-6 -12 -18 -24 -30 -36 -42
7 14 21 28 35 42 49

(A) 28
(B) 112

!

Your way to Top Business Schools _____________
44

(C) 336
(D) 448
(E) 784

Explanation:
Integers in each row constitute arithmetic progression. Thus, we can easily find the sum in first row:
S
1
= (7 + 1)7/2 = 28. If we look at two consecutive rows then second one is product of first row
and integer 2, the third row is a product of first row and integer 3, etcetera. The sums in rows can be
found: S
2
= -2S
1
, S
3
= 3S
1
, S
4
= -4S
1
, S
5
= 5S
1
, S
6
= -6S
1
, S
7
= 7S
1
. The sum of all integers
in the table is equal to S
1
(1 2 + 3 4 + 5 6 + 7) = S
1
4 = 112. The best answer is B.

Consecutive integers can also be viewed as an example of arithmetic progression. GMAT questions
often ask about their sums.

Sample problems:
7. (OG) If an increasing sequence of 10 consecutive integers, the sum of the first 5 integers is 560.
What is the sum of the last 5 integers in the sequence?
(A) 585
(B) 580
(C) 575
(D) 570
(E) 565

Explanation:
Let the first integer be x, then the last integer in the sequence is x + 9. Thus, the sum of the first 5
integers is equal to (x + x + 4)5/2 = 5x + 10. The sum of the last 5 integers is equal to (x + 5 + x +
9)5/2 = 5x + 35. These two sums differ by 25, and the second sum is greater than the first one.
Thus, the answer is 565 + 25 = 585. The best choice is A.

8. (OG) If the sum of n consecutive integers is 0, which of the following must be true?
I. n is an even number
II. n is an odd number
III. The average (arithmetic mean) of the n integers is 0.
(A) I only
(B) II only
(C) III only
(D) I and III
(E) II and III

Explanation:
If the sum of consecutive numbers is 0, then for each positive x in the sequence must be negative
x. For example, if 2 in sequence then -2 is also there. There are k positive numbers, k negative
numbers, and there should be zero: n = k +k + 1 = 2k +1 odd number.
The arithmetic average of some set is equal to the sum of its elements divided by the number of
elements in this set. Therefore we can conclude that the average is equal to 0 if and only if the sum
is equal to 0. Given this, III is also true. E is the answer.

Geometric progression is a sequence of numbers where each term after the first is found by
multiplying the previous one by a fixed non-zero number called the common ratio.

!

Your way to Top Business Schools _____________
45

Example: the sequence 2, 6, 18, 54, ... is a geometric progression with common ratio 3. Similarly
10, 5, 2.5, 1.25, ... is a geometric sequence with common ratio 1/2.

The n-th term of a geometric sequence with initial value a and common ratio r is given by
a
n
= a
1
r
n-1

Such a geometric sequence also follows the recursive relation
a
n
= r a
n-1
, n 1
The sum of first n progression elements is equal to
S
n
=
a
1
(r
n
1)
r 1

Sample problems:

9. A certain series is defined by the following recursive rule: S
n
= k S
n-1
, where k is a constant. If
the first term of the series is 64 and the 25
th
term is 192, what is the 9
th
term?
(A) 2
(B) S
(C) 64S
(D) 64S
3

(E) 64 S
24


Explanation:
Using recursive rule, S
25
= S
1
k
24
, 192 = 64 k
24
,we can find value of k: k
24
= S, k
83
=
(k
8
)
3
= S , k
8
= S
3

To find the 9
th
term we need only k
8
: S
9
= S
1
k
8
, S
9
= 64 S
3
. the answer is D.

!

Your way to Top Business Schools _____________
46

Statistics
Mean (arithmetic average) of the set {x
1
, x
2
, x
3
, ..., x
n
} is equal to (x
1
+ x
2
+ x
3
+ ... + x
n
)/n.
Example: the average of 6, 4, 7, 10, and 4 is
6+4+7+10+4
5
=
31
5
= 6.2

To calculate the median of n numbers, rst order the numbers from least to greatest; if n is odd, the
median is dened as the middle number, whereas if n is even, the median is dened as the average
of the two middle numbers.
Example: In order to find the median of the set {6,4,7,10,4}, put them in order from least to the
greatest: {4,4,6,7,10}, and take the central number: 6. Thus the median is 6.
For the numbers 4, 6, 6, 8, 9, 12, the median is (6+8)/2=7.

Range of the set {x
1
, x
2
, x
3
, ..., x
n
} is equal to the difference between its greatest and lowest
elements.
iange u, anu iange = u, if

1. all the elements equal to each othei {a, a, a ]
2. the set consists of only one element {a]


Mode is a most frequent element in the set.
Example: in the set {1,2,3,4,3,5,7,2,1,2} mode is equal to 2.

Arithmetic progression has its mean equal to its median.
Arithmetic progression has its range equal to (n 1)d.

Sample problems:
10. (OG) If m is the average of the first 10 positive multiples of 5 and if M is the median of the first 10
positive multiples of 5, what is the value of M m?
(A) -5
(B) 0
(C) 5
(D) 25
(E) 27.5


Explanation:
Multiples of 5 constitute numbers is an arithmetic progression, for which mean and median are
equal. Thus their difference is equal to 0. The answer is B.

11. (OG) For a certain set on n numbers, where n > 1, is the average equal to the median?
(1) If the n numbers in the set are listed in increasing order, then the difference between any pair of
successive numbers in the set is 2
(2) The range of n numbers in the set is 2(n 1)

Explanation:
Statement (1) essentially says that this set of numbers is arithmetic progression with difference d =
2. For arithmetic progression we have mean equal to median. Therefore, (1) alone is sufficient and
the answer must be A or D. Second statement gives no clue about question asked in the stem and
thus is insufficient. The answer is A.
Following problems will give you deeper insight into calculation of averages (means).

!

Your way to Top Business Schools _____________
47


Sample problems:
12. (OG) If the average of four numbers K, 2K + 3, 3K 5, and 5K + 1 is 63, what is the value of K?
(A) 11
(B) 63/4
(C) 22
(D) 23
(E) 253/10

Explanation:
Since (K + 2K + 3 + 3K 5 + 5K + 1)/4 = 63, (11K 1)/4 = 63, 11K = 253, K = 23. D is the
answer.

13. (OG) If S = {0, 4, 5, 2, 11, 8}, how much greater than the median of the numbers in S is the mean
of the numbers in S?
(A) 0.5
(B) 1.0
(C) 1.5
(D) 2.0
(E) 2.5

Explanation:
Clearly, mean of S is equal to (4 + 5 + 2 + 11 + 8)/6 = 30/6 = 5. To find the median we should order
the numbers in the set: {1, 2, 4, 5, 8, 11}. Median of S is equal to (4 + 5)/2 = 4.5. The difference
between mean and median is 5 0.5 = 0.5. The answer is A.

Sometimes questions concerning averages may ask to find one specific missing member of the set.
The information given usually includes values of other members and the average of the whole set.

14. (OG) If Jills average score for three games of bowling was 168, what was her lowest score?
(1) Jills highest score was 204
(2) The sum of two Jills highest scores was 364

Explanation:
(1) is insufficient to answer the stem question since the other two scores are unknown.
(2) is alone sufficient to answer since the lowest score is 3168 364 = 504 364 = 140. B is the
answer.

15. (OG) The average of 6, 8, and 10 equals the average of 7, 9, and
(A) 5
(B) 7
(C) 8
(D) 9
(E) 11

Explanation:
The average of 6, 8, and 10 is 8, and if x is missing number, (7 + 9 + x)/3 = 8 and x = 8. C is the
answer.
Average (mean) value of the set which consists of m xs and n ys can be expressed as

!

Your way to Top Business Schools _____________
48

(mx + n y)
(m + n)
.
This feature resembles discrete mixtures. Following problems illustrate this property.

Sample problems:
16. (OG) If x books cost $5 each and y books cost $8 each, then the average cost, $ per book, is
(A) (5x + 8y)/(x + y)
(B) (5x + 8y)/(xy)
(C) (5x + 8y)/13
(D) 40xy/(x + y)
(E) 40xy/13


Explanation:
Total cost of the books is 5x + 8y, and total number of books is x + y. Thus, A is the answer.
Another clear shortcut is the formula mentioned above.

Similar problems concerning medians often appear on the GMAT. Let us start with simplest ones.

17. (OG) The temperatures in degrees Celsius recorded at 6 in the morning in various parts of a certain
country were 10
0
, 5
0
, -2
0
, -1
0
, -5
0
, and 15
0
. What is the median of these temperatures?
(A) -2
0
C
(B) -1
0
C
(C) 2
0
C
(D) 3
0
C
(E) 5
0
C

Explanation:
Since the set contains even number of elements, the median is (-1 + 5)/2 = 2
0
C. The answer is C.
More complex problems require more rigorous analysis.

Sample problems:

18. (OG) If the median of the numbers in the list {3, 6, 8, 19} equals to the median of {x, 3, 6, 8, 19},
what is the value of x?
(A) 6
(B) 7
(C) 8
(D) 9
(E) 10

Explanation:
It is evident that the median of the first set equals to 7 (
6+8
2
= 7). Median of the second set is x, 6, or
8. Thus, x = 7. The answer is B.

19. (OG) If n is an integer between 1 and 10, inclusive, then the median of the set {2, 4, 6, 8, n, 3, 5, 7,
9} must be
(A) either 4 or 5
(B) either 5 or 6

!

Your way to Top Business Schools _____________
49

(C) either 6 or 7
(D) n
(E) 5.5

Explanation:
Note that if n = 5, the median is equal to 5, and if n = 6, the median is equal to 6. Otherwise n < 5 or
n > 6 and the median is anyway in the set {5, 6}. Thus, B is the answer.

20. (OG) Which of the following numbers is greater than three-fourths of the numbers but less than
one-fourth of the numbers from the set {38, 69, 22, 73, 31, 47, 13, 82}
(A) 56
(B) 68
(C) 69
(D) 71
(E) 73

Explanation:
The ranking of the numbers is as follows: {13, 22, 31, 38, 47, 69, 73, 82}. Thus, answer choice
should be greater than 6 numbers - three-fourths of set and less than 2 numbers - one-fourth of
numbers - strictly between 69 and 73. Of the choices given, only 71 satisfies these conditions. D is
the answer.

As we have already said, range equals to the difference between largest and lowest members of the
set. To see how this topic is tested on the GMAT, consider following examples:

Sample problems:
21. (OG) S is a set containing 9 different numbers, T is a set containing 8 different numbers, all of
which are members of S. Which of the following CANNOT be true?
(A) The mean of S is equal to the mean of T
(B) The median of S is equal to the median of T
(C) The range of S is equal to the range of T
(D) The mean of S is greater than the mean of T
(E) The range of S is less than the range of T

Explanation:
A can be true if S = {-4, -3, -2, -1, 0, 1, 2, 3, 4} and T = {-4, -3, -2, -1, 1, 2, 3, 4}.
The same is true for B and C.
D can be true if S = {-4, -3, -2, -1, 0, 1, 2, 3, 4} and T = {-4, -3, -2, -1, 0, 1, 2, 3}.
E CANNOT be true anyway since range can only shrink when some elements are taken out from
the set. Thus, E is the answer.

22. (OG) If S is a set of four integers w, x, y, and z, is the range of the numbers in S greater than 2?
(1) w z > 2
(2) z is the least number in S

Explanation:
(1) is sufficient to answer the question from the stem, since range is equal to the maximum
difference between any two elements of the set. If there are some members from the set S that have
difference greater than 2, then the range is greater than 2.

!

Your way to Top Business Schools _____________
50

(2) alone is not sufficient to answer the stem question since no information about any other
members is given. A is the answer.

If x
*
is the mean of some finite set S = {x
1
, ..., x
n
}, then
s =
_
(x
1
x

)
2
+(x
2
x

)
2
+. . . +(x
n
x

)
2
n

is called standard deviation of S.

Properties:
1. Su u,
anu su = u, if
1. all the elements equal to each othei {a, a, a ]
2. the set consists of only one element {a]

2. The value of standard deviation is independent from other characteristics, such as mean, median or
mode. To obtain the standard deviation we need to know all the values of set members.
3. The value of standard deviation does not characterize the values of set members, but only their
mutual deviations.
4. If the same constant a is added to each number of the set {x
1
,x
2
,x
3
,,x
n
} with standard deviation s,
the deviation of the new set {x
1
+a, x
2
+a, , x
n
+a} will still be s.
5. If the each number of the set {x
1
,x
2
,x
3
,,x
n
} with standard deviation s is multiplied by the same
number b, the deviation of the new set {bx
1
, bx
2
, , bx
n
} will equal bs.

Sample problems:
23. (OG) I = {72, 73, 74, 75, 76}
II = {74, 74, 74, 74, 74}
III = {62, 74, 74, 74, 89}
The data sets I, II, and III are ordered from greatest standard deviation to least standard deviation in
which of the following?
(A) I, II, and III
(B) I, III, and II
(C) II, III, and I
(D) III, I, and II
(E) III, II, and I

Explanation:
Standard deviation of the second set is 0. Even from this fact we can infer that the answer is B or D.
Lets compare standard deviations I and III. It can be assumed that standard deviation of I is less
than standard deviation of III (elements in I are closer to their mean m = 74, than elements in III). In
fact one can calculate s
2
(I) = (-2)
2
+ (-1)
2
+ 0
2
+ 1
2
+ 2
2
= 10, while s
2
(III) = (-12.6)
2
+ (...)
2
+ (...)
2
+
(...)
2
+ (...)
2
>> s
2
(I).
The answer is D.

Neither median nor mean are measures of standard deviation. Conversely, standard deviation is not
a measure of mean. Following problems illustrate these theses.

Sample problems:
24. (GC) Is mean of set A greater than mean of set B?

!

Your way to Top Business Schools _____________
51

(1) The median of set A is greater than the median of set B
(2) The standard deviation of set A is greater than the standard deviation of set B

Explanation:
(1) and (2) together are insufficient to answer the stem question: if A = {4, 7, 10} and B = {3, 5, 7}
both (1) and (2) are satisfied and the mean of A is greater than the mean of B; if A = {1, 7, 7} and B
= {6, 6, 6} both (1) and (2) are again satisfied and the mean of A is less than the mean of B. Thus, E
is the answer.

!

Your way to Top Business Schools _____________
52

Home assignment
1. (OG) A certain bakery has 6 employees. It pays annual salaries of $14,000 to each of 2 employees,
$16,000 to 1 employee, and $17,000 to each of the remaining 3 employees. The average annual
salary of these employees is closest to which of the following?
(A) $15,200
(B) $15,500
(C) $15,800
(D) $16,000
(E) $16,400
2. (OG) What is the value of the sum of a list of n odd integers?
(1) n = 8
(2) The square of the number of integers on the list is 64
3. (OG) Is the average of x and y greater than 20?
(1) The average of 2x and 2y is 48
(2) x = 3y
4. (OG) What is the average of j and k?
(1) The average of j + 2 and k + 4 is 11
(2) The average of j, k, and 14 is 10
5. (GC) What is the average of a, b, and c?
(1) (a + b) + (c + d) = 17
(2) d = 5
6. (GC ) Are the integers a, b, and c consecutive?
(1) b a = c - b
(2) The average of a, b, and c equals to b
7. (OG) The average of six numbers is 8.5. When one number is discarded, the average of the
remaining numbers becomes 7.2. What is the discarded number?
(A) 7.8
(B) 9.8
(C) 10.0
(D) 12.4
(E) 15.0

!

Your way to Top Business Schools _____________
53

8. (OG) On 3 sales John has received commissions of $240, $80, and $110, and he has one additional
sale pending. If John is to receive an average commission of $150 on the 4 sales, then the fourth
commission must be
(A) $164
(B) $170
(C) $175
(D) $182
(E) $185
9. (OG) If the average of 4 numbers is 50, how many of the numbers are greater than 50?
(1) None of the four numbers is equal to 50
(2) Two of the numbers are equal to 25
10. (OG) Will the first 10 volumes of a 20-volume encyclopedia fit upright in the bookrack of width x?
(1) x = 50 cm
(2) Twelve of the volumes have an average thickness of 5 cm
11. (OG) If x, y, and z are numbers, is z = 18?
(1) The average of x, y, and z equals to 6
(2) x = -y
12. (GC) If the average height of three people is 68 inches, is the shortest person more than 60 inches
tall?
(1) The height of the tallest person is 72
(2) One of the persons is 70 inches tall
13. (OG) If a basketball team scores an average of x points per game for n games and then scores y in
its next game, what is the teams average score for the n + 1 games?
(A) (nx + y)/(n + 1)
(B) x + y/(n + 1)
(C) x + y/n
(D) n(x + y)/(n + 1)
(E) (x + ny)/(n + 1)
14. (OG) Last year department store X had a sales total for December that was 4 times the average of
the monthly sales totals for January through November. The sales total for December was what
fraction of the sales total for the year?
(A) 1/4
(B) 4/15

!

Your way to Top Business Schools _____________
54

(C) 1/3
(D) 4/11
(E) 4/5
15. (GC ) Two groups in the class have passed the test. If the average grade for the first group was 68.2,
and the average grade for the second group was 73.5, what was the average grade in two groups?
(1) The first group had 20 more students than the second one
(2) The first group has 3 times more students than the second one
16. (GC) In a certain company, the average wages of employees in Town A is X, the average wages of
employees in Town B is Y. If both types of employees are added together, is the new average salary
smaller than (X + Y)/2?
(1) There are more employees in Town A than B
(2) Y X = 4200
17. (OG) The average of the set of numbers {3, k, 2, 8, m, 3} is 4. If k and m are integers and k m,
what is the median of the set?
(A) 2.0
(B) 2.5
(C) 3.0
(D) 3.5
(E) 4.0
18. (OG) If p, q, x, y, and z are different positive integers, which of the five integers is the median?
(1) p + x < q
(2) y < z
19. (GC) Two numbers are removed from the list {1, 3, 5, 7, 9, 11, 13, 15, 17, 19}. What is the standard
deviation of the remaining 8 numbers?
(1) The median of the eight numbers is 10
(2) The mean of the eight numbers is the same as the original mean
20. (GC) What is the value of k?
(1) The median of the set {n, k, 2, 8, 12} is 7
(2) k > n
21. (GC) Is n > 5?
(1) The median of the set {-2, 1, 5, 8, n} is greater than 1
(2) The median of the set {-2, 1, 5, 8, n} is less than 4

!

Your way to Top Business Schools _____________
55

22. (GC ) What is the median of the set with 9 positive integers?
(1) The mean of 4 smallest ones is 150
(2) The mean of 4 largest ones is 500
23. (GC) A secondary school teacher conducted Math test to 2 groups of students. Is the range for the
first group same as the second group?
(1) The average score was 60 for each group
(2) The number of students in each group taking the test was 30, and the lowest score was 48
24. (GC) A set S has a range 24.7 and following operations are performed on S: 6 is added to all
elements of the set and further all the elements of the set S are divided by 10. What will be the new
range of the set?
(A) 24.7
(B) 2.47
(C) 3.07
(D) 3.47
(E) 30.7
25. (OG) Which of the following CANNOT be the median of the three positive integers x, y, and z?
(A) X
(B) Z
(C) x + z
(D) (x + z)/2
(E) (x + z)/3

!

Your way to Top Business Schools _____________
56

Test 2 Progression and statistics
1. If sequence x
1
, x
2
, x
3
, , x
n
is such that x
n+1
is 5 more than x
n
for n > 0, does x
99
have a units digit
of 9?
(1) The 100 term plus 1 is a multiple of 5.
(2) The first term is 4.

2. On a given day, 50 students in a certain class took a math test. Some of the students took the test on
test paper A, and the remaining of the 50 students took the test on test paper B. What is the average
score on the test for the 50 students?
(1) The average score for the students who take the test A was 75.
(2) The average score for the students who take the test was 77.

3. What is the average of n x , n y, n z, and n w?
(1) The average of x, , z and w is 4n.
(2) n = 120.

4. If the range of the six numbers 4, 3, 14, 7, 10, and x is 12, what is the difference between the
greatest possible value of x and least possible value of x?
(A) 2
(B) 7
(C) 12
(D) 13
(E) 15

5. If the average of positive integers x, y, and z is 10, what is the greatest possible value of z?
(A) 8
(B) 10
(C) 20
(D) 28
(E) 30

6. Of the five numbers in a sequence, the first term is 10,000, and each of the following terms is 20%
of the previous term. What is the value range of the five numbers?
(A) 9,375
(B) 9,750
(C) 9,975
(D) 9,984
(E) 10,736

7. The infinite sequence a
1
, a
2
, , a
n
, is such that a
1
= 1, a
2
= 2, and , what is a
5

a
4
?
(A) 4
(B) 12
(C) 28
2
1

=
n
a
n n
a a

!

Your way to Top Business Schools _____________
57

(D) 60
(E) 124

8. Which of the following is terminating decimal?
I.
II.
III.
(A) I only
(B) II only
(C) III only
(D) I and II only
(E) II and III only

9. 2
33
+ 2
34
2
35
=
(A) 32
33
(B) 2
33
(C) 2
33
(D) 22
33
(E) 32
33

10. If 60 students took a test and the median score was 80, which of the following must be true?
I. At least 29 scores are greater than 80
II. At least 30 scores are equal to or greater than 80
III. At most 30 scores are equal to or less than 80
(A) I only
(B) II only
(C) III only
(D) I and II only
(E) I and III only

11. In the first week of the year, Nancy saved $1, in each of the next 51 weeks, she saved $1 more than
she had saved in the previous week. What was the total amount that Nancy saved during the 52
weeks?
(A) $ 1,326
(B) $ 1,352
(C) $ 1,378
(D) $ 2,652
(E) $ 27,560

12. Is 3 in set M?
(1) 13 is in the set M.
(2) If x is in the set, then x 2 is also in the set M.

120
5
625
33
512
37

!

Your way to Top Business Schools _____________
58

13. In a certain group of people, the average weight of the males is 170 pounds and of the females, 120
pounds. What is the average weight of the people in the group?
(1) The group contains twice as many females as males.
(2) The group contains 15 more females than males.

14. If p and q are prime numbers, where p is no more than q, is sum of p and q a prime number?
(1) p is not equal to q.
(2) p is greater than 2.

15. If the terms of a arithmetic sequence are 2, 4, 6, 8, 10, ..., x
n
, what is the value of n?
(1) The sum of n terms is 2,550.
(2) The average of the n terms is 51.

16. If x and are integers and xy
2
is a positive odd integer, which of the following must be true?
I. xy is positive
II. xy is odd
III. x + is even
(A) I only
(B) II only
(C) III only
(D) I and II only
(E) II and III only

17. Is mx < m + x?
(1) 0 < x < 1.
(2) m is a positive integer.

18. What is the range of numbers in Set S?
(1) The standard deviation of the numbers is 3.5.
(2) The average of all numbers is 7.

19. The average of positive integers x, y, and z is 12. If x < y < z,

and the median of three numbers is 10
greater than the smallest number, what is the greatest possible value of z?
(A) 11
(B) 15
(C) 22
(D) 24
(E) 33

20. If 8 < m < 120, how many integer m are possible for is a square of integer?
(A) 2
(B) 3
(C) 4
(D) 5
(E) 6
3
m

!

Your way to Top Business Schools _____________
59


21. If the median of five different positive integers is 20, which of the following is the least possible
sum of these five positive integers?
(A) 62
(B) 66
(C) 69
(D) 72
(E) 100

22. Is the standard deviation of the scores of students in class X greater than the standard deviation of
the scores of students in class Y?
(1) The median score of students in class Y is greater than the median score of students in class X.
(2) The average (arithmetic mean) score of students in class Y is greater than the average score of
students in class X.


Answers:
1. B
2. E
3. C
4. D
5. D
6. D
7. B
8. E
9. B
10. B
11. C




















12. C
13. A
14. B
15. D
16. E
17. C
18. E
19. D
20. D
21. B
22. E

!

Your way to Top Business Schools _____________
60

Lesson 3
Factorization
Positive integer that has exactly two different positive factors is called prime.

Examples: 7 is an example of prime number since it has only two positive integer factors: 1 and 7. 1
is not prime since it has only one positive integer factor: 1.
Every positive integer can be uniquely factored into product of powers of primes.
Example: Lets find all prime factors of 132. Obviously, the least prime it can be
divided by is 2. We write 2 in the right column and the result of division (66) in the
left one. As we see, that 66 is also divisible by 2, we repeat the previous step.
33 can by evenly divided by 3 with 11 as a result. 11 is prime itself.
Thus, 1S2 = 2
2
S 11.

Consider one more example:

72 = 2
3
S
2



In general, in order to find all prime factors of a certain number N, first find out whether it is
divisible by the least primes you know, e.g. 2, 3 or 5. If it is not divisible by any of them, try 7, 11
and so on. Each time when you find a prime factor, divide N by it and continue with the newly
obtained number until you get 1. As a result, the integer N will be expressed as a product of n
primes p
1
, p
2
, , p
n
, each in the respective power s
1
, s
2
, , s
n
.
N = p
1
s
1
p
2
s
2
p
n
s
n


1. (OG) If the product of the integers x, y, z, and w is 770, and if 1<w<x<y<z, what is the value of w+
z?
(A) 10
() 13
() 16
(D) 18
(E) 21

Explanation:
The prime factorization of 770 is 2x5x7x11. Since 1<w<x<x<y<z, the values for the variables must
be w = 2, x = 5, = 7, and z = 11, so w + z = 13. Thus, is the answer.

More complex problems may ask for the total number of factors of a given integer.
132 2
66 2
33 3
11 11
1
72 2
36 2
18 2
9 3
3 3
1

!

Your way to Top Business Schools _____________
61


Example: 8 = 2
3
. The only prime factor of 8 is 2, but the total number of its positive different
factors is 4: they are 1, 2, 4 and 8.
In most cases such procedure is too complicated and takes a lot of time. There is an easy way:
for
N = p
1
s
1
p
2
s
2
p
n
s
n

the total number of positive factors (Q) can be found using the formula
Q = (s
1
+1) (s
2
+ 1) (s
n
+1)

Example:
8 = 2
3
, hence 8 has 3+1=4 positive factors.
72 = 2
3
S
2
, and 72 has (S +1) (2 +1) = 4 S = 12 positive factors.
1S2 = 2
2
S 11, and 132 has (2 + 1) (1 + 1) (1 + 1) = S 2 2 = 12 positive factors.

2. (OG) How many different positive integers are factors of 441?
(A) 6
(B) 9
(C) 4
(D) 12
(E) 8

Explanation:
441 = 3x147 = 3x3x49 = 3x3x7x7 = 3
2
x7
2
. Substituting this into the formula, we have (2 + 1)x(2 +
1) = 9 factors. Thus, B is the answer

3. (OG) If n is an integer, then n is divisible by how many positive integers?
(1) n is the product of two different prime numbers
(2) n and 2
3
are each divisible by the same number of positive integers

Explanation:
(1) is sufficient to answer the stem question since n = p
1
p
2
, it has (1 + 1)x(1 + 1) - 4 factors.
(2) is also sufficient since the number of integer factors of 2
3
is (3 + 1) = 4. Thus, D is the answer.

4. (OG) Which of the following is the least positive integer that is divisible by 2, 3, 4, 5, 6, 7, 8, and 9?
(A) 15,120
(B) 3,024
(C) 2,520
(D) 1,890
(E) 1,680

Explanation:
The maximum power of 2 that is presented among {2, 3, 4, 5, 6, 7, 8, 9} is 3. The maximum power
of 3 is 2, the maximum power of 5 is 1, the maximum power of 7 is 1. Thus, 2
3
x3
2
x5
1
x7
1
=
8x9x5x7 = 2,520. is the answer.

If positive integer N is expressed as a product of primes
N = p
1
s
1
p
2
s
2
p
n
s
n

!

Your way to Top Business Schools _____________
62

then N
2
is expressed as follows:
N
2
= p
1
2s
1
p
2
2s
2
p
n
2s
n
,
In other words, the powers of primes are doubled. Hence, if N is a positive integer, the prime
factorization of N
2
contains only even powers of prime factors.
In general, for positive integer N
- If N
2
is uivisible by a
2
, then N is uivisible by a
- If N
2
is uivisible by the piime p, then N
2
is uivisible by p
2
anu hence,
N is uivisible by p.

5. (OG) If n is a positive integer and n
2
is divisible by 72, then the largest positive integer that must
divide n is
(A) 6
() 12
() 24
(D) 36
(E) 48

Explanation:
If n
2
is divisible by 72, then n must be divisible by 3 and 4 (given the formula from the property).
Thus, n must be divisible by 12. is the answer.

6. (OG) If x, y, and z are positive integers such that x is a factor of y, and x is a multiple of z, which of
the following is NOT necessarily an integer?
(A) (x+z)z
(B) (y+z)/x
(C) (x+y)/z
(D) xy/z
(E) yz/x

Explanation:
Evidently, x and are divisible by z and everything that has z as its denominator is necessarily
an integer. Thus, the only answer choices possible are and E. z/x is NOT necessarily an integer
since from the stem it is only known that x/z is always an integer. Thus, is the answer.

7. (OG) If r and s are positive integers, is r/s an integer?
(1) Every factor of s is also a factor of r
(2) Every prime factor of s is also a prime factor of r

Explanation:
(1) is sufficient to answer the stem question since s has at least two factors - s itself and 1. Both s
and 1 are factors of r, according to the stem. Thus, r is divisible by s and r/s is an integer. Thus, the
answer must be A or D.
(2) alone is insufficient to answer the stem question since if r = 18 and s = 16 r/s is not an integer,
while if = 18 and s = 2 r/s is an integer. Thus, A is the answer.

8. (OG) If the integer n is greater than 1, is n equal to 2?
(1) n has exactly 2 positive factors
(2) The difference of any two positive factors of n is odd.

!

Your way to Top Business Schools _____________
63


Explanation:
(1) is insufficient to answer the stem question since it actually states that n is prime. So, n can
be 2, or 3, or 5, etc.
(2) Statement says that the difference of any two positive factors of n is odd. Thus, n does not
have odd factors except for 1. Therefore, no prime greater than 2 is a factor of n. Having this, we
can state that n = 2. Thus, is the answer.

9. (OG) If is an integer, is y
3
divisible by 9?
(1) is divisible by 4
(2) is divisible by 6

Explanation:
(1) is not sufficient.
(2) is sufficient, since is divisible by 6 and consequently, is divisible by 3. Thus, y
3
is divisible
by 27 and consequently, y
3
is divisible by 9. is the answer.

Property:
- for every two consecutive integers n and n + 1 product n (n + 1) is divisible by 2;
- for every three consecutive integers n, n + 1, n + 2 product n (n + l) (n + 2) is divisible by 3
and by 2;
- for every consecutive integers product n(n + l) . . . (n +k 1) is divisible by k!
- For consequent integers n, ..., n + - 1 their sum is always divisible by k.

10. (OG) If n is a positive integer, then n(n +1)(n + 2) is
(A) even only when n is even
(B) even only when n is odd
(C) odd whenever n is odd
(D) divisible by 3 only when n is odd
(E) divisible by 4 whenever n is even

Explanation:
n(n + l)(n + 2) is always divisible by 6, so A, B, C, and D are wrong. E is the answer. Indeed, n(n +
l)(n + 2) is divisible by 4 whenever n is even.

11. (OG) If a, b, and are consecutive positive integers and a < b < c, which of the following must be
true?
I.c - a = 2
II.abc is an even integer
III.(a + b + c)/3 is an integer
(A) I only
() II only
() I and II only
(D) II and III only
(E) I, II, and III

Explanation:
If a, b, c consecutive integers, a = a, b = a + 1, c = a + 2. Then c a = a +2 2 = 2. Also, abc
must be an even integer since there is at least one even integer is among a, b, and . Moreover, sum

!

Your way to Top Business Schools _____________
64

a + b + = a +a +1 +a +2 = Sa + S = S(a +1) is also divisible by 3. You can also check it
by picking the numbers. Thus, E is the answer.

!

Your way to Top Business Schools _____________
65

LCM and GCD
Consider a pair of integers: 48 and 180. They have several common divisors (i.e. numbers, which
are the factors of both, 48 and 180): 1, 2, 3, 4, etc. In general, every pair of integers has one or more
common divisors. The least of them is always 1 and the greatest one is called GCD (Greatest
Common Divisor). Lets find the GCD for numbers 48 and 180.
First, find the prime factorizations of the two numbers:
48 = 2 2 2 2 S,
18u = 2 2 S S S.
What they share in common is two "2"s and a "3":

Greatest common divisor = 2 2 3 = 12.

In general, to find the GCD of two numbers
1) Find prime factorizations of two numbers;
2) Find the overlap of these factorizations;
3) Multiply the primes from the overlap to get the GCD.
Numbers a and b are called mutually prime if and only if they do not have common factors greater
than 1. In other words a and b are called mutually prime if GCD(a,b)=1.

Examples: 3 and 16 are mutually prime, 16 and 22 are not mutually prime since they have common
factor other than 1 (it is 2).

Lets now find common multiples (i. e. a numbers, which are divisible by both the integers) of 48
and 180: they are 720, 1440, and so on. There is infinite number of common multiples for any given
pair of integers. The greatest of them cannot be found and the least of them is called LCM (Lowest
Common Multiple). To find it just multiply all of the prime numbers in the diagram above.
Least common multiple = 2 2 2 2 3 3 5 = 720.

In general, to find the LCM of two numbers
1) Find prime factorizations of two numbers;
2) Find the overlap of these factorizations;
3) Multiply the primes from the overlap by the non-repeating primes of each number.

!

Your way to Top Business Schools _____________
66

In other words, the LCM(a,b) will be the product of multiplying the highest power of each prime in
the factorizations of a and b together.
Example:
Find the value of LCM(8,9,21).
First, factor out each number and express it as a product of prime number powers.
8 = 2
3
S
0
7
0

9 = 2
0
S
2
7
0

21 = 2
0
S
1
7
1

Out of the 4 prime factor categories 2, 3, and 7, the highest powers from each are 2
3
, 3
2
, and 7
1
.
Thus,
LCN(8,9,21) = 2
3
S
2
7
1
= 8 9 7 = Su4

Properties:
1) LCM(a,b) and GCD(a,b) are defined for only positive integers a and b.
2) _
uCB(a, b) a
uCB(a, b) b

; _
LCN(a, b) a
LCN(a, b) b
.


3) uCB(a, b) LCN(a, b) = a b.
4) LCN(a, b) = uCB(a, b)if anu only if a = b.
5)
X is a multiple of a
X is a multiple of b
X is a multiple of LCN(a, b)

1. (OG) Which of the following CANNOT be the greatest common divisor of two positive integers x
and y?
(A) 1
(B) x
(C) y
(D) x-y
(E) x+y

Explanation:
Evidently, x + > min(x, ) > GCD(x, ). Thus, x + CANNOT be the greatest common divisor of
x and . is the answer.

2. (GC) Is a two-digit number PQ odd?
(1) lowest common multiple of P and Q is even
(2) P is odd

Explanation:
Statement (1) actually says that at least one digit (P or Q) is even. (1) itself is insufficient to answer
the stem question.
Statement (2) alone is insufficient since no information about Q is given. Thus, the answer must be
or E. Statements (1) and (2) together are sufficient since P is odd and Q must be even to satisfy
information from (1). Therefore, Q is even and PQ is even, and is the answer.

!

Your way to Top Business Schools _____________
67

3. (GC) What is the value of ab?
(1) a = 4m, b = 4n, and the greatest common divisor of m and n is 1
(2) The lowest common multiple of a and b is 24

Explanation:
Statement (1) actually says that the greatest common divisor of a and b is 4.
Statement (2) states that the lowest common multiple of a and b is 24. As we already know, LCM(a,
b)xGCD(a, b) = axb. Therefore, is the answer.

4. (OG) If a positive integer n is divisible by both 5 and 7, the n must also be divisible by which of
the following?
I. 12
II. 35
III. 70
(A) None
() I only
() II only
(D) I and II
(E) II and III

Explanation:
If n is divisible by both 5 and 7, then it must be divisible by LCM(5, 7) = 35. Thus, only II is
satisfied and the answer is C.

5. (OG) If n is a positive integer less than 200 and 14n/60 is an integer, then n has how many different
positive prime factors?
(A) Two
() Three
() Five
(D) Six
(E) Eight

Explanation:
If 14n/60 is an integer, then 7n/30 is also an integer. Thus, according to the second property, n is
divisible by 30. Therefore, n can be {30, 60, 90, 120, 150, 180}. All these numbers have exactly
three prime factors: 2, 3, and 5. is the answer.

6. (OG) Is integer n odd?
(1) n is divisible by 3
(2) n is divisible by 5

Explanation:
Neither (1) nor (2) is sufficient to answer the stem question (3 and 6 or 5 and 10 can serve as
examples). The answer must be or E.
(1) and (2) combined together are also insufficient. Indeed, from (1) and (2) combined we have than
n is divisible by 15. But n can be 15 or 30, so it is again unknown whether n is even or odd. E is the
answer.

7. (OG) If r and s are integers, is r + s divisible by 3?
(1) s is divisible by 3

!

Your way to Top Business Schools _____________
68

(2) r is divisible by 3

Explanation:
Neither (1) nor (2) alone is sufficient to answer the stem question.
(1) and (2) together are sufficient to answer the stem question since both r and s are divisible by 3,
and their sum is divisible by 3. is the answer.

8. (OG) If and n are integers, is n divisible by 7?
(1) n - 3 = 2k
(2) 2k - 4 is divisible by 7

Explanation:
(1) or (2) alone is insufficient to answer the stem question.
From (1) and (2) we have n = 2k + 3 and 2k - 4 = 7m. Therefore, = (7m + 4)/2, n = 2x(7m + 4)/2
+ 3 = 7m + 7. Evidently, n is divisible by 7. is the answer.

9. (OG) If r and s are integers, is r divisible by 7?
(1) The product rs is divisible by 7
(2) s is not divisible by 7

Explanation:
(1) or (2) alone is not sufficient.
Combined together, these two statements are sufficient to answer the stem question, since if rs is
divisible by 7 and s is not, then r must be divisible by 7. is the answer.

10. (OG) Is integer x divisible by 36?
(1) x is divisible by 12
(2) x is divisible by 9

Explanation:
Clearly, (1) or (2) alone is not sufficient.
Combined together (1) and (2) imply that x is divisible by LCM(9, 12)=36. is the answer.

!

Your way to Top Business Schools _____________
69

Division with Remainder
We have had some practice in figuring out whether one number (f) is a factor of another (n). To
answer such questions you must divide n by f. If the quotient is an integer then f is a factor of n
(and n is divisible by f). If the quotient is not an integer, then f is not a factor of n, and youll end up
with a remainder after dividing. For example, lets divide 145 by 6:

In other words, 145 contains 6 twenty four times and one more unit: 14S = 6 24 +1
Generally, the remainder is r when x is divided by y means x = y q +i, i < y the integer q is
called quotient, the integer r is called remainder, and the integer y is called the divisor.
One more way to express the same idea is x = i(mou y).
For instance, The remainder is 1 when 7 is divided by 3 means 7 = 3 2 + 1 or 7=1(mod 3).
Dividing both sides b of
x = y q +i by y gives the following alternative form
x
y
= q +
r
y
.
Notation
X = 4q +2
Is a general expression of the integer X that leaves remainder 2 when divided by 4. Giving q
different integer values: 0, 1, 2, one can find all such numbers: 2, 6, 10,
All in all, the latter expression means, that beginning with 2 every fourth number leaves remainder
2 when divided by 4. Thus, 2 is the least such number.
Note: r is the least number that leaves remainder r when divided by every integer greater, than r.
Consider the following problem:
The integer X leaves remainder 2 when divided by 4 and remainder 1 when divided by 3.

On the GMAT one of three types of tasks will follow:
a) Find the least possible X.
b) Find one general rule for X.
c) Find the number of such integers in a certain interval.

Explanation:
a) First of all, interpret the statement of the problem as follows:
X = 4a + 2 anu X = Sb + 1
Note: Be sure to use different variables for quotients in different rules.
Secondly, write out the equation
4a +2 = Sb + 1

and express b through a:
b =
4a +1
S
.
We choose to express b, because its coefficient is less then the one before a.

!

Your way to Top Business Schools _____________
70

Thirdly, try different whole values (0, 1, 2,) for a until b becomes the integer.
In our case:
for a=0, b=1/3 not integer,
for a=1, b=5/3 not integer,
for a=2, b=3 integer.

Thus, a = 2 anu b = S are the least possible values of a and b to integers simultaneously.
Finally, substitute a for 2 in the rule for X (or b for 3, which yields the same X):

X
mIn
= 4 a
mIn
+ 2 = 4 2 +2 = S S +1 = 1u

Ten is the least possible X, that leaves remainder 2 when divided by 4 and remainder 1 when
divided by 3. Thus,
X
mIn
= 1u.

b) In order to obtain the general rule for X use the following formula:

X = LCN(u
1
, u
2
) c + X
mIn


where u
1
, u
2
the divisors, X
mIn
- the least possible X.
In our example: LCN(S, 4) = 12 anu X
mIn
= 1u.
Hence,
X = 12c +1u.
c) Lets find out, how many integers with the required properties there are between 100 and 150.
1uu < 12c + 1u < 1Su,
9u < 12c < 14u,
7.S < c < 11. (6).
Since c is an integer, there are 4 possibilities: c= 8, c=9, c=10 or c= 11, thus the answer is 4.

Sample problems:
1. (OG) When 10 is divided by the positive integer n, the remainder is n - 4. Which of the following
could be the value of n?
(A) 3
(B) 4
(C) 7
(D) 8
(E) 12

Explanation:
Let us write the main equation that will be used in this section: 10 = nxq + n - 4. Since n is positive,
q 0. Expressing q from the equation, we get q = (14 - n)/n = 14/n - 1. Since q is non-negative
integer number, 14 must be divisible by n. Evidently, among numbers {3, 4, 7, 8, 12} only 7
satisfies this condition. Thus, C is the answer.

2. (OG) Which of the following CANNOT yield an integer when divided by 10?
(A) The sum of two odd integers
(B) An integer less than 10
(C) The product of two primes

!

Your way to Top Business Schools _____________
71

(D) The sum of three consecutive integers
(E) An odd integer


Explanation:
A is wrong since there are two odd integers (5 and 5, 3 and 7) that sum up to 10.
B is wrong since 0 is less than 10 but it yields an integer (0, itself) when divided by 10.
C does not satisfy the stem since the product of two primes, 2 and 5, is divisible by 10.
D is wrong since the sum of three consecutive integers (9, 10, and 11) is 30 which is a multiple of
10.
E is the answer, since odd number cannot yield an integer when divided by 10.

3. (OG) If x is an integer and y = 3x + 2, which of the following CANNOT be a divisor of y?
(A) 4
(B) 5
(C) 6
(D) 7
(E) 8

Explanation:
Since y = 3x + 2, y is not equally divisible by 3. Thus, y is not divisible by any multiple of 3,
including 6. A is wrong since y = 8 = 3x2 + 2 and y is divisible by 4. There are similar examples for
B, D, and E. C is the answer.

4. (OG) When N is divided by T, the quotient is S and the remainder is V. Which of the following
expressions is equal to N?
(A) SxT
(B) S+V
(C) SxT+V
(D) Tx(S+V)
(E) Tx(S-V)

Explanation:
Using empirical rule N = Txquotient + remainder = TxS + V, one can easily get the answer, C.

5. (GC) If 4 students were added to a dance class, would the teacher be able to divide her students
evenly into a dance team (or teams) of 8?
(1) If 12 students were added, the teacher could put everyone in teams of 8 without any leftovers
(2) The number of students in the class is currently not divisible by 8

Explanation:
Let S be the number of students in the class. Then (1) implies that S + 12 is divisible by 8, while (2)
simply states that S is not a multiple of 8. If S + 12 is divisible by 8, then S + 12 = 8xq, where q is
an integer. Then S + 4 = (8q - 12) + 4 = 8q - 8 = 8x(q - 1). Thus, S + 4 is divisible by 8 and groups
of 8 students could be formed without any leftovers. (1) is sufficient to answer the stem. (2) is not
sufficient since if S = 4 one group of eight dancers could be formed with new 4 students, while if S
= 5 this is impossible. A is the answer.

!

Your way to Top Business Schools _____________
72

6. (GC) What is N?
(1) N =7x+3
(2) N = 4y+2

Explanation:
First statement gives the set of numbers: 3, 10, 17, etc. The same does second statement. It means
the answer is C or E. Taken together statements give new rule N=28z+10 (you can find it using the
algorithm described above), which describes set of numbers: 10, 38, 66, etc. The answer is E.

7. (GC) An old woman goes to market and a horse steps on her basket and crashes the eggs. The rider
offers to pay for the damages and asks her how many eggs she had brought. She does not remember
the exact number, but when she had taken them out two at a time, there was one egg left. The same
happened when she picked them out three, four, five, and six at a time, but when she took them
seven at a time they came out even. What is the smallest number of eggs she could have had?
(A) 60
(B) 81
(C) 241
(D) 301
(E) 361

Explanation:
The easiest way to solve is to look at multiples of 7 among answers: only 301 is divisible by 7. The
answer is D.

8. (OG) What is the units digit of (13)
4
(17)
2
(29)
3
?
(A) 9
(B) 7
(C) 5
(D) 3
(E) 1

Explanation:
To find the units digit of product we need to know only last digits of factors:
(_3)
4
(_7)
2
(_9)
3
=(_1)(_9)(_9)=_1. The answer is E.

!

Your way to Top Business Schools _____________
73

Home assignment
I. Factorization.
1. How many prime factors does positive integer n have?
(1)
5
n
has only a prime factor.
(2) 3n
2
has two different prime factors.

2. If m = p
x
q
y
, where p and q are different positive prime numbers. Is m a square of integer?
(1) xy is an odd number
(2) x + y is n even number.

3. If 5x
2
has two different prime factors, at most how many different prime factors does x have?
(A) 1
(B) 2
(C) 3
(D) 4
(E) 5

4. If m and n are positive integer, and 1800m = n
3
, what is the least possible value of m?
(A) 2
(B) 3
(C) 15
(D) 30
(E) 45

5. If 5400mn = k
4
, where m, n, and k are positive integers, what is the least possible value of
m + n?
(A) 11
(B) 18
(C) 20
(D) 25
(E) 33

6. If n = 4p , where p is a prime number greater than 2, how many different positive even divisors does
n have, including n ?
(A) Two
(B) Three
(C) Four
(D) Six
(E) Eight

7. If (2
x
)(2
y
) = 8 and (9
x
)(3
y
) = 81, then (x, y) =
(A) (1,2)
(B) (2,1)
(C) (1,1)

!

Your way to Top Business Schools _____________
74

(D) (2,2)
(E) (1,3)

II. LCM, GCD
1. (GC) What is the greatest possible common divisor of two different positive integers which are
less than 144?
(A) 143
(B) 71
(C) 72
(D) 13
(E) 11

2. (GC) The GCD of two numbers is 20 and the LCM is 840. One of the numbers is 120. What is
the other number?
(A) 140
(B) 120
(C) 40
(D) 20
(E) 10

3. (GC) What is the greatest common divisor of a, b, and c?
(1) greatest common divisor of a and b is 3
(2) greatest common divisor of b and is 4

4. (GC) a and b are positive integers. What is the value of b?
(1) a=5
(2) The lowest common multiple of a and b is equal to the greatest common divisor of a and b

5. (GC) What is the greatest common divisor of a, b, and 3a+23b?
(1) a=4
(2) The greatest common divisor of a, b, and 23a + 3b is 4

6. (GC) What is the greatest common divisor of a + b and 4?
(1) GCD(a, 4)= 2

!

Your way to Top Business Schools _____________
75

(2) GCD(b,4) = 2

7. (OG) If x and are positive integers such that x = 8y + 12, what is the greatest common divisor of x
and y?
(1) x = 12u, where u is an integer
(2) = 12z, where z is an integer

8. What is the greatest common divisor of positive integers x and y?
(1) x and y share exactly one common factor
(2) x and y are both prime numbers

9. Comet A is seen near the Earth every 12 years while comet B every 20 years. If both comets
were observed in 1979, for how many years do we have to wait to see the two comets together
again? (Assume now is 2004)
(A) 20
(B) 16
(C) 25
(D) 35
(E) 32

10. Deborah and Mike visit the university library at regular intervals every 3 and 4 days
respectively. If both of them were in the library on Monday, what day of the week will it be when
they meet in the library again?
(A) Thursday
(B) Wednesday
(C) Saturday
(D) Friday
(E) Sunday

11. (OG) Is p
2
an odd integer?
(1) p is an odd integer
(2) p is an odd integer

12. Integer m has 4 different prime factors and n has 3 different prime factors. If m and n has the
greatest common factor of 15, how many different prime factors does mn have?
(A) 4
(B) 5
(C) 6
(D) 7
(E) 8

Division with remainder

1. (OG) If a is a positive integer, and if the units' digit of a
2
is 9, and the units' digit of (a + l)
2
is 4,
what is the units' digit of (a + 2)
2
?
(A) 1

!

Your way to Top Business Schools _____________
76

(B) 3
(C) 5
(D) 7
(E) 9

2. What is the sum of digits of number 10
28
28?
(A) 227
(B) 228
(C) 236
(D) 237
(E) 243

3. (GC) A certain number when successively divided by 8 and 11 leaves remainders of 3 and 7
respectively. What will be remainder when the number is divided by the product of 8 and 11?
(A) 3
(B) 7
(C) 51
(D) 10
(E) 0

4. (GC) What is the least positive number which leaves a remainder of 1 when divided by 3, 2 when
divided by 4 and 3 when divided by 5?
(A) 60
(B) 58
(C) 118
(D) 12
(E) 68

5. (GC) Find the least positive number, which leaves, if divided by 3, 5, and 12, a remainder of 2.
(A) 62
(B) 42
(C) 12
(D) 2
(E) 122

6. (GC) How many numbers less than 500 exist such that when any of these is divided by 7 the
remainder is 3 and when any of these is divided by 3 the remainder is 1?
(A) 21
(B) 22
(C) 23
(D) 24
(E) 25

7. When n is divided by 13, the remainder is 2 and the quotient is k. When n is divided by 17, the
remainder is 2. What is the remainder when k is divided by 17?
(A) 0
(B) 2
(C) 4
(D) 13

!

Your way to Top Business Schools _____________
77

(E) 15

8. What is the remainder when positive integer n is divided by 6?
(1) When n is divided by 4, the remainder is 3.
(2) When n is divided by 12, the remainder is 3.

9. When positive integer a is divided by 7, the remainder is 2; when positive integer b is divided by 7,
the remainder is 3; when positive integer is divided by 7, the remainder is 4, What is the
remainder when a + b + c is divided by 7?
(A) 0
(B) 1
(C) 2
(D) 3
(E) 4

10. Is integer x divisible by 3?
(1) When x is divided by 5, the remainder is 1.
(2) When x is divided by 15, the remainder is 1.

11. n is an integer between 80 and 200, inclusive. When n is divided by 8, the remainder is 4. What is
the value of n?
(1) n is divisible by 9.
(2) n is divisible by 12.

12. If m = 7x + 9, and n = 7y + 8, what is the remainder when mn is divided by 7?
(A) 1
(B) 2
(C) 3
(D) 4
(E) 5

13. Is x divisible by y?
(1) x is multiple of 6.
(2) is multiple of 3.

14. When positive integer n is divided by 7, the remainder is 2. What is the value of n?
(1) n is less than 20.
(2) When n is divided by 8, the remainder is 0.


!

Your way to Top Business Schools _____________
78

Test 3 Prime factorization, LCM, GCD, division with remainder

1. If n is an integer greater than 10 and less than 40, what is the value of n?
(1) When n is divided by 5, the remainder is 3.
(2) When n is divided by 3, the remainder is 2.

2. If m, n, and k are positive integer, is mnk = 4?
(1) mn = 4.
(2) nk = 4.

3. For positive integer x, y, and z, is y > z?
(1) When x is divided by 4, the remainder is y.
(2) When x is divided by 6, the remainder is z.

4. If the product of the integers from 1 to n is divisible by 490, what is the least possible value of n?
(A) 7
(B) 14
(C) 21
(D) 28
(E) 35

5. If n = 2
4
35, how many factors of n are greater than or equal to 8 and less than or equal to 30?

(A) 8
(B) 9
(C) 10
(D) 12
(E) 15

6. S is the sum of integers from 20 to 40, inclusive. How many different prime factors does S have?

(A) 1
(B) 2
(C) 3
(D) 4
(E) 5

7. When positive integer n is divided by 7, the remainder is 2. What is the value of n?
(1) n is less than 20.
(2) When n is divided by 8, the remainder is 0.

8. If n is an even integer and k is an odd integer, which of the following CANNOT be an integer?
(A)
(B)
k
n
1 + k
n

!

Your way to Top Business Schools _____________
79

(C)
(D)
(E)

9. What is the sum of the remainders when n positive integers were divided by 2?
(1) The sum of n numbers is an even number.
(2) Eight numbers are odd.

10. How many positive integers less than 30 have no common prime factor with 30?
(A) 5
(B) 6
(C) 7
(D) 8
(E) 9

11. In the number line, is point A and symmetry about the zero point?
(1) The distance from 0 to A is equal to the distance from 1 to B.
(2) The distance from 0 to A plus the distance from 1 to is less than 1.

12. The numbers in which of the following pairs do NOT have pair of distinct prime divisors in
common?
(A) 10 and 20
(B) 12 and 18
(C) 24 and 32
(D) 21 and 63
(E) 22 and 88

13. The average (arithmetic mean) of five consecutive integers is an odd number. Which of the
following must be true?
I. The largest of the integers is even.
II. The sum of the integers is odd.
III. The difference between the largest and smallest of the integers is an even number.
(A) I only
(B) II only
(C) III only
(D) I and II
(E) II and III

14. If n is a positive integer between 30 and 60, inclusive, what is the value of n?
(1) When n is divided by 4, the remainder is 1.
(2) When n is divided by 5, the remainder is 2.

15. For any positive integer n, <n> denotes the remainder when 5n
2
+ 1 is divided by 2. If m is a
k
n 1
n
k
1
2

+
k
n

!

Your way to Top Business Schools _____________
80

positive integer, what is the value of <m>?
(1) m is an even number.
(2) m is divisible by 5.

16. When positive integer n is divided by 7, the remainder is 1. If n is less than 50, what is the value of
n?
(1) When n is divided by 9, the remainder is 7.
(2) When n is divided by 4, the remainder is 3.


Answers:
1. E
2. E
3. E
4. B
5. A
6. D
7. C
8. D






























9. B
10. D
11. B
12. C
13. E
14. E
15. A
16. A

!

Your way to Top Business Schools _____________
81

Lesson 4 Algebra
Functions. Substitution. Symbolism.
An algebraic expression in one variable can be used to define a function of that variable. A function
is denoted by a letter such as f or g along with the variable in the expression:
f(x) =3x+5, g(z) =
z
2
-5z+1
7-z
, F(a) =a!*5a, etc.
The symbols f(x), g(z), F(a) do not represent products; each is merely the symbol for an expression,
and is read f of x or g of z or F of a.
If you need to find value of f at x = 1, it means that you can just substitute=1 in the first
expression. The result can be written f(1) =3*1+5 =8.

The set of all allowable inputs for a function is called the domain of the function.
For example, the domain of the function f(x) =3x+5 is the set of all real numbers and the domain of
the function g(a)=a +1is the set of all numbers not less than 1, as under-root expression must be
non-negative. The domain of any function can be arbitrarily specified, as in the function defined by
f(x) =4x-1 for 0<x<3. Without such a restriction, the domain is assumed to be all values of x that
result in a real number when substituted into the function. To find the domain you need to check
such items:
1) if there is a fraction a/b, its denominator must be b0.
2) if there is x
0
, x must be 0.
3) if there is x
2n
, x must be 0.
4) u
x
, x > u

For any algebraic equation you can calculate its value for given variables. This operation is called
substitution. For example, substituting x=1 and y=2 into x
2
+2xy Sy
3
+ 14, we get 1
2
+2 1
2 S 2
3
+14 = 1 + 4 24 +14 = S.
Symbolism is some unknown new operation: x#y =x+2y, xTy =x
2
-4xy+5, etc.
These problems are not so difficult, you have just substitute your variables in the formulae given in
the task.

1. (GC, M) If P# = P/(P - 1), what is the value of P##?
(A) P/(P - 1)
(B) 1/P
(C) P
(D) 2 - P
(E) P - 1

Explanation:
At first lets find P#.
P# = P/(P - 1), now, lets take this value and substitute it instead of P to find out P##.
Here our function P/(P - 1) becomes an argument.

!

Your way to Top Business Schools _____________
82

We will get
P
P-1
P
P-1
-1
=
P
P-1
P-P+1
P-1
=
P
P-1
1
P-1
= P.
The answer is (C).

2. (MG, M) The operation # is defined in the following way for any two numbers:
p # q = (p q)*(q p)
If p # q = 1, then which of the following are true?

I. p could equal 5 and q could equal 4
II. p could equal 4 and q could equal 5
III. p could equal 1 and q could equal 1
IV. p could equal 1 and q could equal 1

(A) I and II only
(B) I and III only
(C) II and IV only
(D) III and IV only
(E) I, II, III, IV

Explanation:
The best way to solve this is to plug the values into the equation:
(5 4) * (4 5) = 1
(4 5) * (5 4) = 1
(1 1) * (1 1) = 4
(1 1) * (1 1) = 4
Statements I and II give the stated value, 1. The correct answer is (A).

3. (MH, E) What is the value of 3x
2
-1.8x +0.3 for x =0.6?
(A) -0.3
(B) 0
(C) 0.3
(D)1.08
(E)2.46

Explanation:
Just substitute x=0.6 in the equation: 3(0.6*0.6) -1.8*0.6 + 0.3 = 1.08 - 1.08 + 0.3 = 0.3. We get
(C).

4. (OG, E) The symbol @ represents one of the following operations: addition, subtraction,
multiplication, or division. What is the value of 15@14?
(1) 0@1 =1
(2) 1@0 =1

!

Your way to Top Business Schools _____________
83

Explanation:
Using statement 1 we can see that the only operation giving 1 as result is addition. So, @=+ and
15+14 can be found. Sufficient.
In statement 2, both 1+0=1 and 1 0=1, so, @ can be both division and addition. Insufficient.
The answer is A.

!

Your way to Top Business Schools _____________
84

Simplifying algebraic expressions
Algebra is based on the operations of arithmetic and deals with variables (or unknown quantities).
Expression containing arithmetic operations, variables and constants is called algebraic
expression:
x
5
+3x -4,
x-4
3x+19
, 6x +S, etc.

Expressions like x
3
-4x +15 are called polynomials. The degree of polynomial can be found as the
highest power of variable. For example, x
2
+ Sx 1u is called a second degree (or quadratic)
polynomial in x since the highest power of x is 2. The expression y-4 is a first degree (or linear)
polynomial in y since the highest power of y is 1.
When working with algebraic expressions, it is often necessary to factor them or combine like
terms. For example, the expression 7a + 5a is equivalent to (7 + 5)a, or 12a. In the expression 6xy -
3y, 3y is a factor common to both terms: 6xy - 3y = 3y(2x - 1).

Here are some useful rules for simplifying algebraic expressions:
) )( (
) )( (
3 3 ) (
3 3 ) (
2 ) (
2 ) (
) ( ) (
0 ,
) ( ) (
2 2 3 3
2 2 3 3
3 2 2 3 3
3 2 2 3 3
2 2 2
2 2 2
2 2
b ab a b a b a
b ab a b a b a
b ab b a a b a
b ab b a a b a
b ab a b a
b ab a b a
b a b a b a
x if
z
y
z x
y x
w y z y w x z x w z y x
+ + =
+ + = +
+ =
+ + + = +
+ =
+ + = +
+ =
= =

+ + + = + +


Sample problems:
5. (MH, M) If (a b)
2
= 64 and ab = 3, find a
2
+ b
2
.
(A) 61
(B) 67
(C) 70
(D) 58
(E) 69

Explanation:
(a b)
2
= a
2
2ab + b
2
which is equal to 64.
a
2
2ab + b
2
= 64
a
2
+ b
2
= 64 + 2ab
Since ab = 3, 2ab = 6, and therefore a
2
+b
2
= 64 + 6, or 70. The correct answer is (C).

!

Your way to Top Business Schools _____________
85


6. (PR, E) What is the value of
m-n
m
2
-n
2
?
(1) m
2
n
2
= 40
(2) m + n = 10

Explanation:
Here, remembering that the difference of two squares can always be rewritten as the product of the
sum and the difference of the two variables is key:
m
2
n
2
= (m-n)*(m+n).
Then it is easy to see that the piece of the puzzle you are looking for is the value of m+n.
Lets look at the statement 1 first. We need to find the value of m+n, but it does not give it to us.
Its insufficient. So, the answer will be B, C or E.
Statement 2 gives us the needed value, its sufficient, so the correct answer here is (B).

7. (OG, M) What is the value of a
4
-b
4
?
(1) a
2
-b
2
=16
(2) a+b =8

Explanation:
a
4
-b
4
= (a
2
-b
2
)( a
2
+b
2
).
Clearly, (1) or (2) alone is not sufficient to answer the stem question, because they dont give us the
opportunity to find the value of a or b themselves. Therefore, the answer must be (C) or (E). Given
(1) and (2) combined, we have a-b =(a
2
-b
2
)(a + b) = 2. Having this, we can make a system of
two linear equations and find both a and b.
Thus, the right answer is (C).



!

Your way to Top Business Schools _____________
86

Linear equations. Equations with the absolute value. Systems of linear equations.

The main task of algebra is to solve equations involving algebraic expressions. Some examples of
such equations are:

3x+5 = 4-2x - a linear equation with one unknown

2x y =3x+4y-1 - a linear equation with two unknowns

|x-1| =5 equation with the absolute value

_
x +2y = 1
2x y = S

- system of linear equations



The solutions of an equation with one or more unknowns are those values that make the equation
true, or satisfy the equation, when they are substituted for the unknowns of the equation. An
equation may have no solution or one or more solutions. If two or more equations are to be solved
together, the solutions must satisfy all of the equations simultaneously.

To solve a linear equation with one unknown (that is, to find the value of the unknown that satisfies
the equation), the unknown should be isolated on one side of the equation. This can be done by
performing the same mathematical operations on both sides of the equation.
Remember that if the same number is added to or subtracted from both sides of the equation, this
does not change the equality; likewise, multiplying or dividing both sides by the same nonzero
number does not change the equality.

For example, lets solve such equation:

3x+1
2
= 5
3x+1 = 10 - multiply each side by 2
3x = 9 subtract 1 from each side
x=3 - divide each side by 3.
x = 3 is the solution.

As for solving one linear equation with two unknowns, in general it has infinite number of
solutions.
Equation x+y=2, for example, can be performed as x=2-y. As y can be any number, we have infinite
number of pairs (x,y) that suit this equation.

But sometimes if you have additional information about variables (they are integers, digits,
primes, etc.) even one equation with two unknowns can have one solution. For example, if A and B
are positive integers and A(B-1)=1 we can find both A and B. If the product of two positive integers
equals 1 then both these digits equal 1 => A=1 and B-1=1, so, B =2. The answer is A=1, B=2.

!

Your way to Top Business Schools _____________
87

To solve an equation with the absolute value the main property of the absolute value is to be
known:
|x| =x if x0 and |x| = -x if x<0. Therefore, |x|0 for any x.
An equation with the absolute value may have no solution, one or more solutions.

For example, look at such equation:
|x-4| =5
x- 4 = 5 or x- 4 = -5
x =9 or x = -1
So, this equation has two solutions: -1 and 9.

To solve a system of linear equations of two unknowns you have to remember some rules:
Here is a system of two linear equations with two unknowns:
_
ax + by =
dx +ey = I


x, y variables, a,b,c,d,e,f- constants.

1) If a/d = b/e =c/f, system has infinite number of solutions.
For example, system:
_
x +y = 1
2x +2y = 2


has infinite number of solutions, because if we multiple first equation by 2, well get two equivalent
equations, or simply x+y =1 is all information we have about x and y.

2) If a/d = b/e c/f, system has no solution.
For example, system:
_
x +y = 1
2x +2y = 17


has no solution, because if we multiple first equation by 2, well get that 2x+2y =2 and 2x+2y =17,
or 2=17. But its not truth. We can say that the first equation contradicts the second equation. Thus,
no values of x and y can simultaneously satisfy both equations.
With any method, if a contradiction is reached, then the equations have no solution; if a trivial
equation such as 0 = 0 is reached, then the equations are equivalent and have infinitely many
solutions. Otherwise, a unique solution can be found.

3) If neither 1, nor 2 happens, system has the only solution. There are several methods of solving
two linear equations with two variables. Consider the following example:
_
2x + y = 2
x y = 4


The first method is to express one of the unknowns in terms of the other using one of the
equations, and then substitute the expression into the remaining equation to obtain an equation with
one unknown. Lets express y from the first equation:
y =2- 2x
x- (2-2x) =4 - substitute in the second equation

!

Your way to Top Business Schools _____________
88

x- 2+ 2x =4
3x = 6,
x=2.
Now we can find y:
y =2 -2x =2- 2*2 =2-4=-2.
So, the pair (2, -2) is the solution of this system.

The second method is to add first and second equations:
(2x+x) +(y-y) =(2+4)
3x=6
x=2
Now, to find y, we can substitute x in the first equation:
2*2 +y =2
y =2-4 =-2.
Weve got the same solution (2, -2).

The third method can be used if we want to find y at first. To do it, we have to multiple the second
equation by 2 and then to subtract it from the first equation:
_
2x +y = 2
2x 2y = 8

- multiple by 2 the second equation


(2x-2x) +(y-(-2y)) =(2-8) subtract second equation from the first
3y =-6
y= -2
If we want to find x, we can substitute this value of y in the first or second equation.

To solve a system of equations with more than two unknowns that contains equal number of
variables and linear independent equations we can use all rules, as for the system with two
unknowns. If the system contains more variables than equations, in general it cannot be solved. But
if there is additional information about variables (for example, if they are integers or digits),
sometimes such system can be solved.

8. (OG, M) x =0.rstu, where r,s,t,u are nonzero digits. What is the value of x?
(1) r =2s =3t =6u
(2) rs =tu

Explanation:
To find x we need to find all the digits r, s, t, u.
From the first statement we get a system with three equations and four unknowns:
_
i = 2s
i = St
i = 6u


We have to remember that r, s, t, u are nonzero digits integers from 1 till 9. Look at the third
equation: r =6u. If u =1, r=6. If u =2, r=12, but it cannot be so, because r is the digit! Clearly, if

!

Your way to Top Business Schools _____________
89

u=3,4,5, etc., r wont be a digit as well. So, the only possible value of u is 1. And we have such
solution: r=6, s=3, t=2, u=1. So, x=0.6321. Statement 1 is sufficient.
Statement 2 alone is not sufficient. So, the answer is (A).

9. ( MH, E) If x+y =36, what is the value of xy?
(1) y-x=14
(2) y =2x+3

Explanation:
(1) alone is sufficient: add the two equations together and you get 2y = 50, so y = 25 and x = 11.
(2) alone is sufficient, because if you plug the equation into the question, you get x + 2x + 3 = 36,
or 3x = 33, so x = 11 and
y = 25. The answer is D.

10. (GC, M) |2 x| +|x S| = 7. Find x.
(A) {0,1,7}
(B) {0,7}
(C) {1,7}
(D) {0,2,4}
(E) {7}

Explanation:
Each expression with the absolute value has a point in which x changes its sign. For |2-x| it is 2-x=0,
x=2, and for |x-5| it is x-5=0, x=5. So, the number line will be divided into three intervals:
1) x<2, then 2-x>0, x-5<0;
2) 2x<5, then 2-x 0, x-5<0;
3) x5, then 2-x <0, x-50
Lets look at each of them.
1) x<2: 2-x>0, x-5<0:
2-x +(-x+5) =7
2-x-x+5 =7
-2x =0, x=0 the first part of the answer.
2) 2x<5: 2-x 0, x-5<0.
-(2-x) (x-5) =7
-2 +x x+5 =7
3=7 contradiction, no solution.
3) x5: 2-x <0, x-50.
-(2-x) +x-5 =7
-2 +x+x-5 =7
2x =14
x=7 -the second part of the answer.
Answer (B).

!

Your way to Top Business Schools _____________
90

Quadratic equations

The standard form for a quadratic equation is ax
2
+ bx + = , where a, b, and c are real
numbers and a0; for example:
x
2
+6x + S = u
2x
2
7x + 19 = u
x
2
1ux = u
Quadratic equation may have one, two or no solutions. Number of solutions is essential to solve
data sufficiency problems.

Here you can see the geometric interpretation:
y=ax
2
+ bx +c - its parabola.

We can find if its branches are going up or down:
if a>0, branches are going up: if a<0, branches are going down:



The number of points of intersection with x-axis equals the number of roots.





!

Your way to Top Business Schools _____________
91

Two roots: One root: No roots:


Sometimes it is useful to find the vertex of the parabola (Xo,Yo).
If y=ax
2
+bx + c, then Xo =-b/2a, Yo =aXu
2
+bXu +.

For example, y = x
2
4x S.
Xo=
-(-4)
21
=2,
Yo =2
2
4 2 S = 4 - 8 - 5 = -9,
So, the vertex is (2, -9):



One of the most useful rules for solving quadratic equations is:
a
D b
x
a
D b
x
ac b D
c bx ax
2 2
4
0
1 1
2
2

=
+
=
=
= + +

Number Discriminant

D can tell you, how many roots an equation has:
If D<0, there is no solution.

!

Your way to Top Business Schools _____________
92

If D =0, there is

only one root.
If D>0, there are

two roots.

Most quadratic equations invite you to use Viet method, which is applicable to equation with a=1:

p x x
q x x
q px x
= +
=
= + +
2 1
2 1
2
0


Remember, that roots of quadratic equation help to simplify the expression:


11. (OG, E) x > 2 onJ x
2
4x +S = u. Find x.
(A) 3
(B) 4
(C) 1
(D) 2
(E) 5

Explanation:
Lets solve this equation with the Discriminant.
D= 44 -431=16-12=4>0, 2 roots.

1
=
4-4
21
=
2
2
=1, or

2
=
4+4
21
=
6
2
=3.
X has to be greater then 2, so, x=3. The answer is A.

Alternative solution: Viet theorem.

1
+
2
=4

2
=3.
The roots are 1 and 3, because their sum = 4 and their product =3. The answer is A.

12. (GC, M) What is X?
(1) X
2
+ 24 = 8 (X + 1)
(2) X
2
-6X +5 =0

Explanation:
Statement (1) simplifies to X
2
-8X +16 =0 or (X - 4)
2
= 0 from where X = 4. Sufficient.
Statement (2) gives us X
2
-6X +5 =0 or X =5, X=1. We cannot find the only value of X.
Insufficient.
Answer: (A).

13. (OG, M) If t-8 is a factor of t
2
-kt -48, then k =
0 ) )( (
2 1
2
= = + + x x x x a c bx ax

!

Your way to Top Business Schools _____________
93

(A) -6
(B) -2
(C) 2
(D) 6
(E) 14

Explanation:
If t-8 is a factor of t
2
-kt -48, then this polynomial can be expressed as t
2
-kt -48 = (t-8)(t-a),
where a is some number. According to Viet theorem, 8 and a will be roots of the equation
t
2
-kt -48=0
Thus, substituting t=8 in t
2
-kt -48, well get 0:
64 -8k-48=0
8k =16, k=2.
Answer (C).


!

Your way to Top Business Schools _____________
94

Exponential equations

To solve an exponential equation with the same bases of power:
) ( ) ( x g x f
a a =

just equalize the powers of left and right sides:
) ( ) (
) ( ) (
x g x f a a
x g x f
= =
.
If you have different bases, both powers are to be =0.
For example, equation
(S)
2m-1
= 217
15
gives us 2m-1=15x=0, or m=1/2 and x=0.


14. (OG, M) If (2)
2m
= 2
9-m
, m integer, then m=
(A)1
(B)2
(C)3
(D)4
(E)6

Explanation:
At first look at (2)
2m
. 2m is an even integer, so, (2)
2m
=2
2m
:
2
2m
=2
9-m
, equalizing the powers of left and right sides we get 2m =9-m, 3m =9, m=3. The answer
is (C).

15. (OG, M) If (7
3
4
)
n
=7, then n=
(A)1/3
(B)2/3
(C)4/3
(D)5/3
(E)6/3

Explanation:
(7
3
4
)
n
= 7
3n
4
, we get:
7
3n
4
= 7
1
, so, 3n/4 =1, 3n =4, n=4/3. Answer (C).


!

Your way to Top Business Schools _____________
95

Inequalities. Inequalities with the absolute value. Exponential inequalities.
An inequality is a statement that uses one of the following symbols:
not equal to;
> greater than;
greater than or equal to;
< less than;
less than or equal to.

Some examples of inequalities are: 3x - 4y + 5 < 1 - 7x, y 3, 4a b 0,etc.

Very often in the problem you will need to find not the solution of the inequality but the interval
that contains this solution. For example, for x>3 the answer on the question What must be true for
all values of x? could be x 1.

Solving a linear inequality with one unknown is similar to solving an equation; the unknown is
isolated on one side of the inequality. As in solving an equation, the same number can be added to
or subtracted from both sides of the inequality, or both sides of an inequality can be multiplied or
divided by a positive number without changing the truth of the inequality. However, multiplying or
dividing an inequality by a negative number reverses the order of the inequality.

For example, 6 > 2, but (1)*6 <(1)*2.

Such rules are useful:
1) a>b, for any c a+c>b+c
2) if a>b and c>0 then ac>bc
3) if a>b and c<0 then ac<bc
4) if a>b and b>c then a>c
5) if a>b and c>d then a+c>b+d

To solve an inequality with the absolute value you need to remember two simple rules:
1) |x|<a -a<x<a

2) |x|>a x>a or x<-a.


Solving an exponential inequality is very alike with solving an exponential equation:
to solve an exponential inequality with the same bases of power:
a
I(x)
< a
g(x)

!

Your way to Top Business Schools _____________
96

Look at the base. If it is in the interval (0 ,1), write down the same inequality with the powers of left
and right sides, but change the sign: f(x)>g(x).
If it is in the interval [1, +), just write down the same inequality with the powers of left and right
sides: f(x)<g(x).

16. (MG, E) If 4 x > 5, then
(A) x > 1
(B) x > 1
(C) x < 1
(D) x < 1
(E) x = 1

Explanation:
4 x > 5
x > 1
Divide by 1 and change the inequality sign:
x < 1
The correct answer is (D).

17. (GC, M) If x
2
- 100 < 300, how many integers x satisfy this condition?
(A) 42
(B) 39
(C) 38
(D) 37
(E) 19

Explanation:
First of all the equation we have to solve is x
2
<400 or |x| < 400. Then -400 < x < 400, or - 20 <
x < 20. This gives us 19 2 + 1 = 39 integers (we have to add 1 for zero). Answer (B).

18. (PR, M) Is mn <m/n?
(1) mn is a positive integer.
(2) n is a negative number.

Explanation:
Statement 1 is not sufficient, because we can take m=-5, n=-6 and we will get 30> 5/6, answer no.
Or we can take m=-5, n=-0.1 and we will get 0.5<50, answer yes.
Statement 2 does even less, since we do not know something about m. Together, the statements still
do not tell us what pair to take: m=-5, n=-6 or m=-5, n=-0.1, so the correct answer is (E).

19. (OG, M) What is the smallest integer n for which 2S
n
> S
12
?
(A)6
(B)7

!

Your way to Top Business Schools _____________
97

(C)8
(D)9
(E)10

Explanation:
2S
n
= (S
2
)
n
= S
2n
. We get S
2n
> S
12
, so, we can write down the same inequality with the powers
of left and right sides: 2n>12, n>6. The smallest integer n that suits this inequality is n=7. Correct
answer is (B).

!

Your way to Top Business Schools _____________
98

Home assignment.
1. (GC H) If x is a number such that -2 x 2, which of the following has the greatest possible
absolute value?
(A) 3x - 1
(B) x
2
+ 1
(C) 3 - x
(D) x - 3
(E) x
2
- x

2. (OG E) If x = -1, then -(x
4
+ x
3
+ x
2
+ x) =
(A) -10
(B) -4
(C) 0
(D) 4
(E) 10

3. (OG E) If x*y = xy- 2(x + y) for all integers x and y, then 2*(-3) =
(A) -16
(B) -11
(C) -4
(D) 4
(E) 16

4. (OG M) If represents one of the operations +, -, and x, is k(n + m) = (kn) + (km) for all
numbers k, n,and m?
(1) kn is not equal to nk for some n numbers k
(2) represents subtraction

5. (OG M) If the operation is defined for all a and b by the equation a b = a
2
b/3, then
2 (3 -1) =
(A) 4
(B) 2
(C) -4/3
(D) -2
(E) -4

6. (OG E) If x and y are different integers and x
2
= xy, which of the following must be true?
I. x=0
II. y=0
III. x=-y

(A) I only
(B) II only

!

Your way to Top Business Schools _____________
99

(C) III only
(D) I and III only
(E) I, II, and III

7. (OG H) Is (x/m)(m
2
+ n
2
+ k
2
) = xm + yn + zk?
(1) z/k=x/m
(2) x/m = y/n

8. (OG E) Which of the following equations is NOT equivalent to 25x
2
= y
2
- 4?






9.

9. (OG E) If a, b, and c are nonzero numbers and a + b = c, which of the following is equal to 1?







10. (OG M) If x 0, what is the value of (x
p
/x
q
)
4
?
(1) p=q
(2) x=3

11. (OG M) If m/n = 5/3, what is the value of m + n?
(1) m > 0
(2) 2m + n = 26

12. (OG E) If b < 2, and 2x - 3b = 0, which of the following must be true?

(A) x >-3
(B) x > 2
(C) x =3
(D) x<3
(E) x >3

13. (OG E) If 4x + 3y = -2 and 3x + 6 = 0, what is the value of y?
(A) 25x
2
+ 4= y
2

(B) 75x
2
=3y
2
-12
(C) 25x
2
=(y-2)(y+2)
(D) 5x=y-2
(E) x
2
=(y
2
-4)/25

(A) (a-b)/c


(B) (a-c)/b
(C) (b-c)/a
(D) (b-a)/c
(E) (c-b)/a
(A) -10/3


!

Your way to Top Business Schools _____________
100







14. (GC H) Which of the following describes value of x
2
- x when x is between 0 and 1?
(A) 1 > x
2
- x > 0
(B) 1 > x
2
- x -1/4
(C) 0 > x
2
- x -1/4
(D) 1 > x
2
- x > -1
(E) 0 > x
2
- x -1

15. (GC M) Is ((x - 5)
2
) = 5 - x?
(1) -x|x| > 0
(2) 5 - x > 0

16. (OG H)If x increases from 165 to 166, which of the following must increase?
I. 2x- 5
II. 1 - 1/x
III. 1/(x
2
- x)
(A) I only
(B) III only
(C) I and II
(D) I and III
(E) II and III

17. (GC H) A perfect number is defined as one for which the sum of all the unique factors less the
number itself equals to the number. For instance, 6 is a perfect number, because the factors of 6
(apart from 6 itself) are 1, 2 and 3, and 1 + 2 + 3 = 6. Which of the following is a perfect number?

(A) 12
(B) 28
(C) 15
(D) 13
(E) 67

18. (GC M) $Y$ = (Y
Y
)/(Y Y 2). What is the last digit of $($4$)$?

(A) 8
(B) 6
(C) 4
(D) 2
(B) -2
(C) -2/3
(D) 2/3
(E) 2

!

Your way to Top Business Schools _____________
101

(E) 1

19. (GC M) Is integer R positive?
(1) R
3
= R
(2) |R| = R

20. (MG M) The operation # is defined in the following way for any two numbers:
p # q = (p q) times (q p)
If p # q = 1, then which of the following is true?

I. p could equal 5 and q could equal 4
II. p could equal 4 and q could equal 5
III. p could equal 1 and q could equal 1
IV. p could equal 1 and q could equal 1

(A) I and II only
(B) I and III only
(C) II and IV only
(D) III and IV only
(E) I, II, III, IV

21. (OG M) Is x = 0?
(1) x y = x
(2) x + y = y

22. (GC M) If x is a positive integer, is x > 2.5x - 5?
(1) x < 2
(2) x is a prime number

23. (GC H) If t/u = x/y and t/y = u/x and t, u, x, y non - zero integers, which of the following is true?
(A) t/u = 1
(B) y/x = - 1
(C) t = u
(D) t = +/- u
(E) None of the above

24. (MG M) The square root of 800 is between which of
the following integers?

(A) 21 and 25
(B) 24 and 28
(C) 27 and 31
(D) 30 and 34

!

Your way to Top Business Schools _____________
102

(E) 33 and 37

25. (MG M) The trinomial x
2
+ x 20 is exactly divisible by

(A) x 5
(B) x + 4
(C) x 10
(D) x 4
(E) x 2

26. (MH E) If 5a = 9b = 15c, what is the value of a + b + c?
(1) 3c a = 5c 3b
(2) 6cb = 10a

27. (GC H) How many roots does this equation have?
A x + A = B
(1) A does not equal 0
(2) B does not equal 0

28. (GC E) What is X?
(1) X
2
- 1 = X + 1
(2) X + 3 is a prime number

29. (GC M) Is 4t
3
- 2t
2
- 8t + 16 divisible by t
2
?
(1) t > 1
(2) t is an even prime number

30. (GC E) If

S + x 1 = 4, what is the value of x?


(A) 101
(B) 122
(C) 157
(D) 170
(E) 197

31. (MH M) What is the value of the greater of two numbers if one of the numbers is three times the
smaller number?
(1) One of the numbers is 12.
(2) The sum of the two numbers is 16.

32. (GC M) If a, b, and c are positive and a
2
+ c
2
= 202, what is the value of b - a - c?
(1) b
2
+ c
2
= 225
(2) a
2
+ b
2
= 265

!

Your way to Top Business Schools _____________
103

33. (MH H) Of the following, which is most nearly equal to but not greater than 1S?
(A) 3.7
(B) 3.8
(C) 3.9
(D) 4.0
(E) 4.1

34. (MG H) Every letter in the alphabet has a number value that is equal to its place in the alphabet;
thus, the letter A has a value of 1 and C a value of 3. The number value of a word is obtained by
adding up the value of the letters in the word and then multiplying that sum by the length of the
word. The word DFGH would have a number value of

(A) 22
(B) 44
(C) 66
(D) 100
(E) 108

35. (MG M) A perfect number is one which is equal to the sum of all its positive factors that are less
than the number itself. Which of the following is a perfect number?

(A) 1
(B) 4
(C) 6
(D) 8
(E) 10

36. (GC M) For integers x and y, x = y/6 + 1. Is xy even?
(1) (y + 1)/5 = x
(2) y/x is not odd

37. (GC M) What is the value of the following expression: 6x
2
+ 9y
2
?
(1) x = 2
(2) 6y
2
+ 4x
2
= 22

38. (GC H) What is (x
2
- 6 x + 9) + (2 - x) + (x - 3) if each term in this expression is well defined?

(A) (2 - x)
(B) 2 x - 6 + (2 - x)
(C) (2 - x) + (x - 3)
(D) 2 x - 6 + (x - 2)
(E) x + (x - 2)

!

Your way to Top Business Schools _____________
104

39. (GC M) What is 987 987?

(A) 974,169
(B) 974,219
(C) 974,549
(D) 975,019
(E) 975,369

40. (GC M) If x
2
- 100 < 300, how many integers x satisfy this condition?

(A) 42
(B) 39
(C) 38
(D) 37
(E) 19

41. (GC M) Is (X
3
) (Y
2
) (Z
2
) > 0?
(1) X Y > 0
(2) X Z > 0

42. (GC H) X, Y, and Z are positive integers. Is (X-Y) (Y-Z) (X-Z) > 0?
(1) X
2
+ Y Z = X Y + X Z
(2) X Y - Y
2
= X Z - Y Z

43. (MG M) If x < y, 2x = A, and 2y = B, then

(A) A = B
(B) A < B
(C) A > B
(D) A < x
(E) B < y

44. (MG M) If x and y are positive integers such that (x + y) < 10, then which of the following must be
true?

(A) x < 8
(B) x > 3
(C) x > y
(D) x + y = 5
(E) x y 7

45. (MG E) If point X is on line segment AB, all of the following may be true EXCEPT

!

Your way to Top Business Schools _____________
105

(A) AX = XB
(B) AX > XB
(C) AX < XB
(D) AB > XB
(E) AX + XB < AB

46. (MG M) If x*y =(4x 3y)(x+y) then 5*6=

(A) 1
(B) 11
(C) 15
(D) 22
(E) 34

47. (PR M) Is mn < m/n?
(1) mn is a positive number.
(2) n is a negative number.

48. (PR M) If 2 < a < 11 and 3 < b < 12, then which of the following is NOT true?

(A) 1 < a + b < 23
(B) -14 < a b < 8
(C) -7 < b a < 14
(D) 1 < b + a < 23
(E) -24 < ab < 132

49. (MH M) Is y between 1 and 2, exclusive?
(1) y
2
is less than y
(2) y
2
is between 1 and 2, exclusive


!

Your way to Top Business Schools _____________
106

Test 4 algebra.
1. If x < y, which of the following must be true?
I. |x| < |y|.
II. x y < 0.
III. x
2
y
2
< 0.
(A) I only
(B) II only
(C) III only
(D) I and II only
(E) II and III only

2. If x and are integers, what is the value of (x y)
4
?
(1) The product of x and is 7.
(2) The sum of x and is 8.

3. If x
2
+ 12x + 20 < 0, how many integer values can x be?
(A) 6
(B) 7
(C) 8
(D) 9
(E) 10

4. If x
2
y
2
< 0, which of the following must be true?
I. x + y < 0
II. x y < 0
III. x
2
y
3
< 0
(A) None
(B) I only
(C) II only
(D) III only
(E) I and III only

5. If is a positive integer and x is a negative integer, is y
x
< l?
(1) is greater than 1.
(2) x > 5.


6. If m, n, , and q are positive integers, and m

= n
q
, what is value of pq?
(1) m
2
= n.
(2) p
p
= 4.

7. If x and are positive, is greater than 1?
y
x

!

Your way to Top Business Schools _____________
107

(1) xy > 1.
(2) x > 0.

8. Is x = y?
(1) x
2
+ y
2
= 2xy.
(2) .

9. If the product of x and is 15, what is the sum of x and y?
(1) xy
2
= 75.
(2) x y = 8.

10. How many positive integers, from 2 to 100, inclusive, are not divisible by odd integers greater than
1?
(A) 5
(B) 6
(C) 8
(D) 10
(E) 50

11. On four successive days, a farmer picks exactly twice as many apples each day as on the previous
day. If in the course of the four days he picks a total of 12,000 apples, how many apples does he
pick on the second of the four days?
(A) 800
(B) 1,000
(C) 1,600
(D) 2,000
(E) 6,000

12. If for any none-zero integer N, then, N* (N l)*=
(A)
(B)
(C)
(D)
(E)

13. If equation
2
+ b + = 0 have two distinct roots, which of the following must be true?

I. b > 0
II. ac > 0
1 4
2 2
=
+ y x
N
N

=
1
1
1 *
) 2 )( 1 (
1
N N
2
1
N
) 2 )( 1 (
1
N N
) 1 (
1
N N
1
1
N

!

Your way to Top Business Schools _____________
108

III. ac < 0
(A) None
(B) I only
(C) II only
(D) III only
(E) I and II only

14. The function f is defined for any integer n that f(n) is the product of all positive even numbers less
than or equal to n, for example, f(10) = 246810. So, what is the greatest prime factor of
f(24)?
(A) 3
(B) 5
(C) 7
(D) 11
(E) 13

15. If 0 < x < l, which of the following is the greatest?
(A) x
1/2
(B) x + 1
(C) 2x
(D) x
2
(E) x
2
+ 1

16. Which of the following CANNOT be the sum of two prime numbers?
(A) 19
(B) 45
(C) 68
(D) 79
(E) 88

17. Is x divisible by y?
(1) x is multiple of 6.
(2) is multiple of 3.

18. If x is not equal to 1, is ?
(1) x > 0.
(2) x < 2.

19. If n is an integer, is 30 the factor of n?
(1) 45 is a factor of .
(2) 30 is a factor of n
2
.

x
x
x
>
1
2
2
n

!

Your way to Top Business Schools _____________
109

20. If x is a positive integer, what is the value of x?
(1) x
3
= 4x.
(2) x
2
4x = 4.

21. Is k
m + n
> 0?
(1) k < 0.
(2) k
m n
< 0.

22. If xy
2
> 0, which of the following must be true?
I. x > 0
II. y > 0
III. yx
2
> 0
(A) I only
(B) II only
(C) III only
(D) I and II only
(E) I and III only

23. If x and are none negative integer, what is the value of xy?
(1) 7
x
= 13
y
.
(2) 3
x
= 9
y
.


Answers:
1. B
2. A
3. B
4. A
5. A
6. C
7. B
8. D
9. A
10. B
11. C








12. A
13. A
14. D
15. B
16. D
17. E
18. E
19. D
20. D
21. B
22. A
23. A

!

Your way to Top Business Schools _____________
110

Lesson 5 Geometry

Lines and angles



In geometry, the word line refers to a straight line that extends without end in both directions.
The line above can be referred to as line PQ or line l. The part of the line from P to Q is called a
line segment. P and Q are the endpoints of the segment. The notation PQ is used to denote both
the segment and the length of the segment. The intention of the notation can be determined from
the context.

Intersecting lines and angles









If two lines intersect, the opposite angles PRQ and TRS are called vertical angles and have the
same measure. Angles x and y can be called neighboring angles.

Perpendicular and parallel lines

An angle that has a measure of 90 is a right angle.
If two lines intersect at right angles, the lines are perpendicular.




If two lines that are in the same plane do not intersect, the two lines are parallel. In the figure,
lines l
1
and l
2
are parallel. All four angles x are equal. So are four angles y.

1. (OG E) In the figure below, the value of y is

!

Your way to Top Business Schools _____________
111


(A) 6
(B) 12
(C) 24
(D) 36
(E) 42

Explanation:
Clearly, 2x and 3x are neighboring angles. Thus, 2x + 3x = 180 and 5x = 180, x = 36. 2x and y +
30 are vertical angles, thus, 2x = y + 30 and y = 2x 30 = 42. E is the correct answer.


!

Your way to Top Business Schools _____________
112

Triangles
Triangle - a closed figure consisting of three line segments linked end-to-end. A 3-sided polygon.

The sum of interior angles is 180.

The length of any side of a triangle must be larger than the positive difference of the other two
sides, but smaller than the sum of the other two sides:

a +b > c > |a-b| ,
b +c > a > |b-c|,
a +c > b > |a-c|.


Line segment CH on the first picture denotes the height (or altitude) of the triangle. Its
perpendicular to AB. Line segment CM is the median. It means that point M divides AB on two
equal parts: AM =MB. Line segment CL denotes the bisector, which means that angles ACL and
LCB are equal.
You can draw three heights in the triangle perpendicular to each side, - three medians, and three
bisectors.

To calculate the area of a triangle use the formula

S =
altttudebase
2
or

S = p(p a)(p b)(p c),
where p =
a+b+c
2
is the semi perimeter of the triangle.

!

Your way to Top Business Schools _____________
113

For a given area equilateral triangle has the smallest perimeter.
For a given perimeter equilateral triangle has the largest area.

Midsegment of a Triangle -a line segment joining the midpoints of two sides of a triangle.

The midsegment is always parallel to the third side of the triangle.
The midsegment is always half the length of the third side.

2. (OG E) In AABC below, what is x in terms of z?


(A) z+73
(B) z-73
(C) 70-z
(D) z-70
(E) 73-z

Explanation:
Since the sum of the interior angles is 180, x + (z + 37) + 70 = 180, x + z = 180 70 37 = 73.
Therefore, x = 73 - z. E is the answer.
Circumscribed circle


!

Your way to Top Business Schools _____________
114

Here, the circle is circumscribed around the triangle ABC. To find the center of this circle you have to
draw perpendicular bisectors to each side of the triangle. For example, for the side AB you will
find the midpoint M (AM=MB) and draw the line that passes through this point perpendicular to
AB. The point of intersection of such lines will be the center of our circle.

Inscribed circle

Here, the circle is inscribed in the triangle ABC. To find the center of this circle you have to draw
bisectors for each angle of the triangle. For example, for the angle ABC you will draw the line BO
that divides this angle on two equal angles (ABO=OBC). The point of intersection of such lines
will be the center of our circle.

Right triangle where one of its interior angles is a right angle (90 degrees).

Right triangle with a given hypotenuse has the largest area when it's an isosceles triangle.
Area can be calculated as follows S =
IcgIcg
2
or S =
hcIghhypotcnusc
2
.

A right triangle inscribed in a circle must have its hypotenuse as the diameter of the circle.

Pythagorean Theorem:
2
= a
2
+ b
2
, where c is the hypotenuse, a and b are legs.

!

Your way to Top Business Schools _____________
115

Any triangle whose sides are in the ratio 3:4:5 is a right triangle. Such triangles that have their
sides in the ratio of whole numbers are called Pythagorean Triples.

A right triangle where the angles are 30, 60, and 90:

This is one of the triangles one should recognize on sight. A fact to memory is: the sides are
always in the ratio 1: S: 2.

A right triangle where the angles are 45, 45, and 90:

This triangle is isosceles and right, so JK=JL and LK=JL2. The sides of such triangle are always
in the ratio 1: 1: 2.

3. (OG M)


In the figure above, segments RS and TU represent two positions of the same ladder leaning
against the side SV of a wall. The length of TV is how much greater than the length of RV?
(1) The length of TU is 10 meters
(2) The length of RV is 5 meters

!

Your way to Top Business Schools _____________
116

Explanation:
Since we know all angles of TUV: 45, 45, 90 and SRV: 30, 60, 90 and also that SR=TV (as the
same ladder), we need to know the length of any side of these two triangles. If we have such
information we can determine the lengths of the other sides, because for these two triangles we
have ratios of the sides: 1: 1: 2 and 1: S: 2 respectively.
So, having TU we can find TV. And as SR=TU we can find RV. Similarly, having the length of
RV, we can determine all lengths in this triangle. And SR=TU will give us information about TV.
Thus, each statement is sufficient to answer the stem question. Therefore, D is the best answer.

Isosceles triangle- two sides are equal in length.

An isosceles triangle also has two angles of the same measure.
Altitude traced to the base bisects the angle and the base.

4. (GC M)

In the triangle ABC above, what is the length of side BC?
(1) Line segment AD has length 6
(2) x = 36

Explanation:
Triangle BCD is isosceles (since 2 of its angles are equal). Thus, BC = BD. Angle ADB is equal to
180 2x since it is neighboring to the angle BDC. The sum of all angles of triangle ABD is equal
to 180: 180 = ABD + 180 2x + x, so, angle ABD is equal to x. This implies that triangle ABD is
isosceles with angle BAD equal to angle ABD. Having this, we can find that AD = BD = BC.
Statement (1) is sufficient to answer the stem question since BC = AD = 6. Thus, the answer must
be A or D. Statement (2) is insufficient to determine any length since only angles are given. Thus,
A is the answer.

Equilateral triangle - all sides have the same length, all angles equal 60 degrees.

!

Your way to Top Business Schools _____________
117

Area S =
a
2
3
4
.
Radius of inscribed circle is r =
a
23
and circumscribed circle is R =
a
3
.

5. (GC H)

If two triangles shown in the figure above are equilateral and the radius of the outer circle equals
to 3, what is the radius of the smallest circle?
(A) \3
(B) 3/2
(C) 3/4
(D) 1/\3
(E) 1/2\3

Explanation:
For an equilateral triangle radius of inscribed circle is i =
a
23
and of circumscribed circle is
R =
a
3
, so, r =R/2.
Let R be the radius of the outer circle, r the radius of the middle circle and r* - the radius of the
smallest circle. We have r =R/2, r*=r/2=R/4 =3/4.
Thus, the answer is C.

Congruent and similar triangles.

Triangles can be called congruent (equal) if all their respective sides are equal and all their
respective angles are equal.

We say that two triangles and are similar if either of the following conditions
holds:
1. Corresponding sides have lengths in the same ratio:
.

!

Your way to Top Business Schools _____________
118

2. = , and = . This also implies that = .

When two triangles and are similar, we write .

6. (GC M) Is area of triangle ABC greater than area of triangle DEF?
(1) Area of ABC < Perimeter of DEF
(2) Angles of ABC = Angles of DEF

Explanation:
Statement 1 is NOT sufficient. Lets look at the right triangles and pick numbers: if area of ABC
is 1 the sides are 1 and 2. The hypotenuse is S, Perimeter is 3+S. Perimeter is much greater
than the area of a triangle with these values. However if sides of ABC are 10, 10, and 102; then
perimeter is 20+102 and area is 50. Perimeter is much smaller than the area. Insufficient.
Statement 2 is insufficient as well. All it tells us is that both triangles are similar or proportionate
to each other, but nothing about their size. Combining the two statements, we still cannot
determine whether the triangles have small values of their sides that yield greater perimeters or
large values that yield greater area measurements. Not sufficient. Therefore E is the answer.


!

Your way to Top Business Schools _____________
119

Quadrilaterals
Quadrilaterals - a closed figure consisting of four line segments linked end-to-end, a 4-sided
polygon.
Sum of interior angles of any convex quadrilateral is S6u degrees.

Parallelogram is a quadrilateral with opposite sides parallel and congruent.


Opposite sides of a parallelogram are equal in length.
Opposite angles of a parallelogram are equal in measure.

The diagonals of a parallelogram bisect each other.

The area of a parallelogram is S = h b, where b is the base of the parallelogram and h is its
height.

7. (GC M) The area of a parallelogram is 100. What is the perimeter of the parallelogram?
(1) The base of the parallelogram is 10
(2) One of the angles of the parallelogram is 45 degrees

Explanation:
From Statement (1) we know the base and the height of the parallelogram. This height can equal
the side of the parallelogram (in case of a rectangle) but it can also exceed it. Thus, Statement (1)
is not sufficient.
From Statement (2) nothing can be established about the length of the base of the parallelogram.
In fact, the base can be arbitrarily long.
Statement (1) and Statement (2) completely define the parallelogram:


DC=AB=BD=10, the length of the sides BC and AD can be found by Pythagorean theorem.

!

Your way to Top Business Schools _____________
120

So, the right answer is C.

Rhombus is actually just a special type of parallelogram.








The diagonals of a rhombus always bisect each other at 90.

There are two ways to find the area of a rhombus.
S = altituue base or S =
dIagonaI
1
dIagonaI
2
2
.

8. (GC M) Is quadrilateral ABCD a rhombus?
(1) AC is perpendicular to BD
(2) AB + CD = BC + AD

Explanation:
ABCD can be a rhombus but it can also be a kite-shaped figure. The diagonals of this figure form
a cross and Statement (2) holds because of the symmetry (AB = BC and CD = AD). The answer
is E.

Rectangle

Opposite sides are parallel and congruent.
The diagonals bisect each other.
The diagonals are congruent.

The area of a rectangle is given by the formula S = length hight.

9. (MH, M) The diagonal of the rectangle is 13. What is the area of the rectangle?
(1) The length of the rectangle is 12
(2) The width of the rectangle is 5

!

Your way to Top Business Schools _____________
121

Explanation:
Statement (1): knowing the diagonal and the length we can find the width. So, Statement (1) is
sufficient.
Statement (2): knowing the diagonal and the width we can find the length. So, Statement (2) is
sufficient.
The answer is D.

Square

A square has a larger area than any other quadrilateral with the same perimeter.
Each diagonal of a square is the perpendicular bisector of the other.
Radius of inscribed circle is i =
a
2
and radius of circumscribed circles is R =
a
2
, where a is side of
square.
The area is S = a
2
or S =
d
2
2
where a is side and d is diagonal of a square.

10. (GC, M) If the area of square A is three times the area of square B, what is the ratio of the
diagonal of square B to that of square A?
(A) 1/3
1/2

(B) 1/3
1/3

(C) 3
1/3

(D) 3
1/2

(E) 3

Explanation:
The ratio of the diagonals is the same as the ratio of the sides. If the side of B is b, then the side of
A must be 3 b. The required ratio is b/(3 b) = 1/3 = 1/3
1/2
. We answer A.

Trapezoid
Trapezoid is quadrilateral which has one pair of parallel sides.

!

Your way to Top Business Schools _____________
122



The usual way to calculate the area is the average base length times altitude S = h
a+b
2
.

11. (MR, M)

What is the area of the trapezoid pictured above?
(1)
(2) The perimeter of trapezoid ABCD = 36.

Explanation:
At first lets draw a picture:

To calculate the area of this trapezoid we need CD, AB and AH. AB=6 by definition, so, the
question is to find CD and AH.
Her e ABMH is a rectangle, so HM=AB=6.
From statement 1 we can get that angle C equals 60 degrees. Then we can find CH and AH from
the triangle ACH (as its angles are 30, 60, and 90 its sides are always in the ratio 1: S: 2).
CH=MD, so, well find CD as CH+HM+MD = 2CH+6. We can answer the question. Statement 1
is sufficient.

!

Your way to Top Business Schools _____________
123

From statement 2 we can get CD=36-8-8-6=14. HM=AB=6, so, CH+MD=14-6=8. These line
segments are equal: CH=MD=4. Using Pythagorean theorem for the triangle ACH well find AH.
Statement 2 also is sufficient.
The answer is D.

Polygons
Sum of interior angles of any convex polygon is S = 18u (n 2), where n is the number of
sides.
Number of diagonals of any convex polygon is
n(n-3)
2
, where n is the number of sides.


!

Your way to Top Business Schools _____________
124

Circle
Circle is a simple shape of Euclidean geometry consisting of those points in a plane which are
equidistant from a given point called the center.

A circle is the shape with the largest area for a given length of perimeter.
To form a unique circle, it needs to have 3 points which are not on the same line.

Area can be calculated using the formula S = R
2
, R is a radius of a circle.
Circumference can be calculated using the formula l = 2R, R is a radius of a circle.

Area of a sector is S = R
2
u
360
, where is the central angle.
Arc Length is l = 2R
u
360
where is the central angle of the arc.

Inscribed angle is an angle ABC formed by points A, B, and C on the circle's circumference.
A central angle is an angle AOC with endpoints A and C located on a circle's circumference and
vertex O located at the circle's center. A central angle in a circle determines an arc AC.

The Central Angle Theorem states that the measure of inscribed angle is always half the measure

!

Your way to Top Business Schools _____________
125

of the central angle.

12. (OG M)

The figure above represents a circular flower bed, with its center O, surrounded by a circular path
that is 3 feet wide. What is the area of the path, in square feet?
(A) 25
(B) 38
(C) 55
(D) 57
(E) 64

Explanation:
Clearly, total area of the shaded region is the difference between areas of two circles. The area of
the outer is 11
2
and the area of the inner is 8
2
. So, the area of the path is (11
2
- 8
2
) = 57.
Thus, D is the answer.

13. (OG E)

If O is the center of the circle above, what fraction of the circular region is shaded?
(A) 1/12

!

Your way to Top Business Schools _____________
126

(B) 1/9
(C) 1/6
(D) 1/4
(E) 1/3

Explanation:
Each of the vertical angles is equal to 30 degrees. Thus, two angles have a share of 60/360 = 1/6 of
the entire circle. The right answer is C.


!

Your way to Top Business Schools _____________
127

Volume geometry

Rectangular solid


Rectangular solid is a three-dimensional solid object bounded by six square faces, facets or sides,
with three meeting at each vertex.

Volume v = abc

Surface area S = 2(ab + bc + ac)
Volume diagonal Max=a
2
+b
2
+ c
2


Cube is the rectangular solid with equal edges.

Volume v = a
3

Surface area S = 6a
2

Volume diagonal Max =aS
Radius of circumscribed sphere
3
2
a.
Radius of inscribed sphere
a
2
.

14. (GC M) What is the maximum straight line distance between any two points of the box 10x8x6?

(A) 10
(B) 241
(C) 2S4
(D) 8
(E) 102

Explanation:
The maximum possible straight line distance between any two points of the rectangular solid is its
volume diagonal. So, Max=a
2
+ b
2
+c
2
= 1u
2
+8
2
+6
2
= 2uu = 1u2. So, the answer is
E.


Sphere consisting of those points in a 3-dimensional space which are equidistant from a given
point called the center.

!

Your way to Top Business Schools _____________
128


Volume v =
4
3
R
3

Surface area S = 4R
2


15. (PR M) Find the volume of the sphere inscribed in the cube with edge 6.

(A) 12
(B) 16
(C) 24
(D) S6
(E) 42

Explanation:
For the cube with the side a radius of inscribed sphere equals
a
2
. So, for our cube radius of
inscribed sphere is 3. The volume of the sphere can be found as v =
4
3
R
3
=
4
3
S
3
= S6. The
answer is D.

Cylinder is geometric shape, the surface formed by the points at a fixed distance from a
given straight line, the axis of the cylinder. The solid enclosed by this surface and by two planes
perpendicular to the axis is also called a cylinder.

Volume v = i
2
h
Area of surface S = 2ih + 2i
2


16. (PR M) Cylindrical tank with height 4 and radius of the base 3 is half full of water. Find the
volume of this water.

!

Your way to Top Business Schools _____________
129


(A) 6
(B) 12
(C) 18
(D) 24
(E) 36

Explanation:
As the tank is half full of water, we need to find the volume of the cylinder with height 2 and
radius of the base 3. v = i
2
h = S
2
2 = 18. The answer is C.

!

Your way to Top Business Schools _____________
130

Coordinate geometry

Quadrants
When the origin is in the center of the plane, they divide it into four areas called quadrants.


Given coordinates of two points, distance D between two points is given by:
u = (x
1
x
2
)
2
+ (y
1
y
2
)
2
.

17. (MH E) Bob lives in the town A that has point coordinates (1,5). His friend Paul lives in the city
B that has point coordinates (-5,-3). Find the straight line distance between two friends.

(A) 6
(B) 8
(C) 10
(D) 12
(E) 14

Explanation:
We have to find the distance between two points A and B. Using formulae:
AB=
_
(1 (S))
2
+(S (S))
2
= (1 + S)
2
+(S +S)
2
= S6 +64 = 1uu = 1u.
The answer is C.

Slope
Slope is defined as the ratio of the "rise" divided by the "run" between two points on a line, or in
other words, the ratio of the altitude change to the horizontal distance between any two points on
the line.
Given two points (x
1
, y
1
) and (x
2
, y
2
) on a line, the slope k of the line is:
slope =
y
1
y
2
x
1
x
2

!

Your way to Top Business Schools _____________
131

If the equation of the line is given in the Point-intercept form: y = kx +b, then k is the slope.

If two lines are parallel then their slopes are equal: k
1
= k
2

If two lines are coincident they are not parallel
If two lines are perpendicular then product of their slopes is -1: k
1
k
2
= 1




18. (GC H) In the Cartesian coordinate system, Point A has coordinates (6, - 7) and Point B has
coordinates (4, 5). If a line is drawn to connect Point A and Point B, does point C lie on line AB?
(1) Coordinates of Point C are ( 5, - 1)
(2) Point C equidistant from Point A and Point B.

Explanation:
The fastest way is to see that we have all the info we need to set up the equation of the line and we
can easily check whether C lies on it because Statement 1 gives us that information. We dont
need to spend time on calculations. A is thus sufficient.
Here is the detailed Explanation:
Slope =
y
1
-y
2
x
1-x
2
= (5 + 7)/(4 - 6 ) = 12/- 2 = - 6
Equation of line:
y - y
1
= m ( x - x
1
)
y - 5 = - 6 ( x - 4)
y = - 6x + 29
1) now ,we know that coordinates of C are ( 5, - 1). for C to lie on AB,(5, - 1) should be one of the
solutions for the equation. lets plug in x = 5 , we get , y = - 6 (5) + 29 = - 1 So, A is sufficient.
2) From Statement 2, we cant say whether the point C is on Line A. There are many possibilities.
So, B is insufficient.


!

Your way to Top Business Schools _____________
132

Circle on a plane
In an x-y Cartesian coordinate system, the circle with center (a, b) and radius r is the set of all
points (x, y) such that: (x a)
2
+ (y b)
2
= i
2
.



19. (GC M) On the coordinate graph a circle is centered at the point (3, 3). If the radius of the circle is
6, and there is a square ABCD inscribed into the circle cutting it into 5 regions, what is the area
of one of the smallest?
(A)
(B) 6 - 12
(C) 6 - 2 6
(D) 3 ( /2 - 1)
(E) 3 /4 - 3/2

Explanation:
To find the answer, we need to know the areas of the square and the circle. We will get 6 as the
area of the circle and 12 as the area of the square (diagonal is 26 and by squaring it and dividing
by 2, we will get values of the sides: 12.
However, CD is one of the four outside segments, so the answer will be (6 - 12)/4 or 3/2 3 or
3( /2 1). The answer is D.

!

Your way to Top Business Schools _____________
133

Home assignment

1. (GC E) In the figure below, if isosceles right triangle PQR has an area of 4 and P is the centre of
the circle, what is the area of the shaded portion of the figure?

(A)
(B) 2
(C) 2
(D) 4
(E) 8

2. (GC E) Line y = k x + b intersects X-axis at (2, 0) and Y-axis at (0, 5). What is k/b?

(A) -5/2
(B) -5/4
(C) -1/2
(D) 2/5
(E) 4/5

3. (OG E)

In the triangle above, does a
2
+ b
2
= c
2
?
(1) x + y = 90
(2) x = y

4. (GC M) Circle A, centre X. XB is the radius. There is a chord AC which intersects XB. D is the
point of intersection between XB and AC. BD = 2, AC = 12, XDA = 90 degrees. What is the
circle's area?

(A) 10
(B) 20

!

Your way to Top Business Schools _____________
134

(C) 50
(D) 80
(E) 100

5. (OG M) What is the number of 360-degree rotations that a bicycle wheel made while rolling 100
meters in a straight line without slipping?
(1) The diameter of the bicycle wheel, including the tire, was 0.5 meter
(2) The wheel made twenty 360-degree rotations per minute

6. (OG M)

What is the radius of the circle above with center O?
(1) The ratio of OP to PQ is 1 to 2
(2) P is the midpoint of chord AB

7. (GC M) The vertices of a triangle have coordinates (x, 1), (5, 1), and (5, y) where x < 5 and y > 1.
What is the area of the triangle?
(1) x = y
(2) Angle at the vertex (x, 1) = angle at the vertex (5, y)

8. (OG H)


The shaded portion of the rectangular lot shown above represents a flower bed. If the area of
the bed is 24 square yards and x = y + 2, then z equals
(A) 13
(B) 213
(C) 6
(D) 8
(E) 10

!

Your way to Top Business Schools _____________
135


9. (OG M) Starting from Town S, Fred rode his bicycle 8 miles due east, 3 miles due south, 2 miles
due west, and 11 miles due north, finally, stopping at Town T. If the entire region is flat, what is
the straight-line distance between Towns S and T?
(A) 10
(B) 82
(C) 157
(D) 14
(E) 24

10. (OG M)


In the figure above, V represents an observation point at one end of a pool. From V, an object
that is actually located on the bottom of the pool at point R appears to be at point S. If VR =
10 feet, what is the distance RS, in feet, between the actual position and the perceived
position of the object?
(A) 10 53
(B) 10 - 52
(C) 2
(D) 5/2
(E) 4

11. (GC H) What is the area of a triangle with the following vertices L(1, 3), M(5, 1), and N(3, 5)?

(A) 3
(B) 4
(C) 5
(D) 6
(E) 7

12. (OG M)

!

Your way to Top Business Schools _____________
136


What is the circumference of the circle above with center O?
(1) The perimeter of OXZ is 20 + 102
(2) The length of arc XYZ is 5

13. (OG M) If the circle below has center O and circumference 18, then the perimeter of sector
RSTO is

(A) 3 + 9
(B) 3 + 18
(C) 6 + 9
(D) 6 + 18
(E) 6 + 24



14. (OG E) The annual budget of a certain college is to be shown on a circle graph. If the size of each
sector of the graph is to be proportional to the amount of the budget it represents, how many
degrees of the circle should be used to represent an item that is 15 percent of the budget?
(A) 15
(B) 36
(C) 54
(D) 90
(E) 150

15. (GC M) If equation |x| + |y| = 5 encloses a certain region on the graph, what is the area of that
region?

(A) 5
(B) 10

!

Your way to Top Business Schools _____________
137

(C) 25
(D) 50
(E) 100

16. (MG M) A spotlight on the ceiling is 5 feet from one wall of a room and 10 feet from the wall at
right angles to it. How many feet is it from the intersection of the two walls?

(A) 15
(B) 5 2
(C) 5 S
(D) 10 2
(E) 10 S

17. (MG M) Peter lives 12 miles west of school and Bill lives north of the school. Peter finds that the
direct distance from his house to Bills is 6 miles shorter than the distance by way of school. How
many miles north of the school does Bill live?

(A) 6
(B) 9
(C) 10
(D) 12
(E) None of these

18. (GC M) At what angle do the lines y = K x + L and x = y + K L intersect?
(1) K = 2
(2) K = L

19. (MG M) If a triangle of base 6 has the same area as a circle of radius 6, what is the altitude of the
triangle?

(A) 6
(B) 8
(C) 10
(D) 12
(E) 14

20. (PR M) If rectangle A has width w and length n , and w < n , what is the value of w?
(1) The area of rectangle A = 24
(2) The perimeter of rectangle A = 20

21. (GC E) Which of the following points is not on the line y = 5 x + 3?

(A) (1/2, 11/2)

!

Your way to Top Business Schools _____________
138

(B) (1/3, 14/3)
(C) (8, 3 + 10 2)
(D) (4, 13)
(E) (2, 31/3)

22. (MG E) The vertex angle of an isosceles triangle is p degrees. How many degrees are there in one
of the base angles?

(A) 180 p
(B) 90 p
(C) 180 2p
(D) 180 p/2
(E) 90 p/2

23. (MH M) A rectangle is equal in area to a square with sides of length 12. Is the diagonal of the
rectangle greater in length than 20?
(1)The rectangle has a length of 16.
(2)The rectangle has a width of 9.

24. (GC M) Into a square with side K is inscribed a circle with radius r. If the ratio of area of square
to the area of circle is P and the ratio of perimeter of the square to that of the circle is Q. Which of
the following must be true?

(A) P/Q > 1
(B) P/Q = 1
(C) 1 > P/Q >
(D) P/Q =
(E) P/Q <

25. (MH M) If a cube has a total surface area of 96, what is its volume?

(A) 16
(B) 36
(C) 64
(D) 81
(E) 96

26. (GC M) Assuming the Earth's orbit around the Sun is a circle, by how much will the length of the
Earth's orbit increase if the radius of this orbit grows by /2 meters?

(A) 1 meter
(B) 2 meters
(C) meters
(D) 2 meters

!

Your way to Top Business Schools _____________
139

(E)
2
meters

27. (GC M) Which of the following lines is parallel to line x = 2.66 - 2y?

(A) 2x + 25
(B) 2.66x 2
(C) x + 3y
(D) - 1/2x 32
(E) 1/2x + 3

28. (MH M) A hat company ships its hats, individually wrapped, in 8-inch by 10-inch by 12-inch
boxes. Each hat is valued at $7.50. If the companys latest order required a truck with at least
288,000 cubic inches of storage space in which to ship the hats in their boxes, what was the
minimum value of the order?

(A) $960
(B) $1,350
(C) $1,725
(D) $2,050
(E) $2,250

29. (MH E) What is the area of square floor X?
(1) The perimeter of the floor is a whole-number multiple of 10.
(2) The diagonal of the floor measures 52 .

30. (GC M) A circle is inscribed in a square. If a diagonal running through the center of the circle is 4
cm long, what is the area of the square that is not occupied by the circle?

(A) 1.7
(B) 2.7
(C) 12
(D) 24
(E) 25

31. (GC M) Line L passes through points (a/2, b) and (a, -2 b). Which of the following represents
the slope of line L?

(A) -a/(6 b)
(B) -b/a
(C) -6 (b/a)
(D) 3 a/(2 b)
(E) a/b

!

Your way to Top Business Schools _____________
140

32. (GC E) The radius of the front wheels of the cart is half that of the rear wheels. If the
circumference of the front wheels is 1 meter and the cart went 1 kilometer, how many revolutions
did the rear wheels make?

(A) 250/
(B) 500/
(C) 250
(D) 500
(E) 750

33. (MH M) A truck driver wants to load as many identical cylindrical canisters of olive oil as can fit
into the 3-meter x 4-meter x 9-meter storage space of his truck. What is the maximum number of
canisters can he load into the truck?
(1) Each canister has a volume of 62, 500 cubic centimeters.
(2) The height of each canister is four times the radius.

34. (MG E) In isosceles triangle ABC, what is the value of C?
(1) The measure of B is 47 degrees
(2) The measure of A is 96 degrees

35. (MG E) If the radius of a circle is decreased by 10%, by what percent is its area decreased?

(A) 10
(B) 19
(C) 21
(D) 79
(E) 81

36. (MH M) When Greenville State University decided to move its fine arts collection to a new
library, it had to package the collection in 20-inch by 20-inch by 15-inch boxes. If the university
pays $0.50 for every box, and if the university needs 3.06 million cubic inches to package the
collection, what is the minimum amount the university must spend on boxes?

(A) $255
(B) $275
(C) $510
(D) $1,250
(E) $2,550

37. (MG E) A square is inscribed in a circle of area 18n. Find a side of the square.

(A) 3
(B) 6

!

Your way to Top Business Schools _____________
141

(C) 3 2
(D) 6 2
(E) It cannot be determined from the information given.

38. (MG E) If point X is on line segment AB, all of the following may be true EXCEPT

(A) AX = XB
(B) AX > XB
(C) AX < XB
(D) AB > XB
(E) AX + XB < AB

39. (PR M) In the rectangle coordinate system, triangle ABC has a vertex at point (0, 56). If point B is
at the origin, then how many points on line AC have integer values for both their x and y values?
(1) The third vertex of triangle ABC lies on the x-axis, and the triangle has an area of 196
(2) Point A has a positive x coordinate and a y coordinate of zero


40. (MG M) If a triangle of base 6 has the same area as a circle of radius 6, what is the altitude of the
triangle?

(A) 6
(B) 8
(C) 10
(D) 12
(E) 14

41. (PR H) A sphere with a radius of 5 is hollowed out at the center. The part removed from the sphere
has the same center, and a radius of 3. What fractional part of the original sphere remained? (The
formula for the volume of a sphere is
4
3
R
3
)

(A) 2/5
(B) 16/25
(C) 27/125
(D) 98/125
(E) 3/5

42. (MG M) In a circle with center O, the measure of arc RS = 132 degrees. How many degrees are
there in angle RSO?

(A) 66
(B) 20
(C) 22

!

Your way to Top Business Schools _____________
142

(D) 24
(E) 48

43. (MH E) If 4 and 11 are the lengths of two sides of a triangular region, which of the following can
be the length of the third side?
I. 5
II. 13
III. 15

(A) I only
(B) II only
(C) I and II only
(D) II and III only
(E) I, II, and III

44. (PR M) A certain cube floating in a bucket of water has between 80 and 85 percent of its volume
below the surface of the water. If between 12 and 16 cubic centimeters of the cubes volume is
above the surface of the water, then the length of a side of the cube is approximately

(A) 4
(B) 5
(C) 7
(D) 8
(E) 9

45. (MH E) Is the triangle with angles a, b, c isosceles?

(1)180 = (a +c) +60
(2) a <2b

46. (MG M) A circle graph shows how the MegaTech corporation allocates its Research and
Development budget: 14% microphotonics; 24% home electronics; 15% food additives; 29%
genetically modified microorganisms; 8% industrial lubricants; and the remainder for basic
astrophysics. If the arc of each sector of the graph is proportional to the percentage of the budget it
represents, how many degrees of the circle are used to represent basic astrophysics research?
(A) 8
(B) 10
(C) 18
(D) 36
(E) 52

47. (MH E) A line segment is drawn from the point (3, 5) to the point (9, 13). What are the coordinates
of the midpoint of this line segment?
(A) (3, 4)

!

Your way to Top Business Schools _____________
143

(B) (12, 18)
(C) (6, 8)
(D) (9, 6)
(E) (6, 9)

48. (GC M )To build a rectangular chicken pen, Mike has 40 meters of net. If Mike wants to maximize
the area of the pan, what will be the most favorable dimensions?

(A) 12 8
(B) 15 8
(C) 10 10
(D) 15 15
(E) 15 5

49. (GC M) Right triangle ABC has a height BD. What is the value of AB times BC?

(1) AB is equal to 6
(2) The product of the non - hypotenuse sides is equal to 24

50. (PR E) If a cube has a volume of 125, what is the surface area of one side?

(A) 5
(B) 25
(C) 50
(D) 150
(E) 625

!

Your way to Top Business Schools _____________
144

Test 5 Geometry
1. In the figure shown above, a pentagon is composed with three right triangles. What is the
perimeter of the pentagon?
(A)
(B)
(C)
(D) 12
(E) 15

2. If a triangle has sides a, b, and c, the area of the triangle can yield with the formula:
, where s is half of the perimeter. If the sides of a triangle are 4, 5, and 7, what
is the area of the triangle?
(A)
(B)
(C)
(D)
(E)

3. From left to right, points , , , D and E lie on the number line. Is the distance between A and
less than the distance between and D?
(1) The distance between A and is less than the distance between and D.
(2) The distance between A and D is less than the distance between and E.

4. If the curve represented by y = x
2
5x + t intersects with the x-axis at two points and one of the
points is (1, 0), what is the other point?
(A) (1, 0)
(B) (2, 0)
(C) (5, 0)
(D) (6, 0)
(E) (3, 0)


5. The figure shows the position on a radar screen of a sweeping beam that is rotating at a constant
rate in a clockwise direction. In which of the four quadrants will the beam lie 30 seconds from
now?
(1) In each 30-second period, the beam sweeps through 270.
3 2 8 +
6 8 +
5 8+
) )( )( ( c s b s a s s
6 2
3 6
6 4
2 6
3 8
y
x
r
2
2
2
2

!

Your way to Top Business Schools _____________
145

(2) r = 30.

6. Is quadrilateral ABCD a square?
(1) AB = = CD = DA.
(2) Two diagonals have the same length.

7. If x and are integer, is x - y even?
(1) x = 3y.
(2) x 3y is even.

8. p and q are integers. If p is divisible by 10
q
and cannot be divisible by 10
q + 1
, what is the value of
q?
(1) p is divisible by 2
5
, but is not divisible by 2
6
.
(2) p is divisible by 5
6
, but is not divisible by 5
7
.

9. Is mx < m + x?
(1) 0 < x < 1.
(2) m is a positive integer.

10. A new released book includes hardback and paperback. If the sales price for the hardback is $20,
and the sales price for the paperback is $15, what is the average price for all of the books sold
during a certain day?
(1) The ratio of the number of the hardback sold to the number of paperback sold is 5 to 3.
(2) A total of 160 books were sold during the day.

11. Line m and n are perpendicular to each other. If m intersects with y-axis at point (0, 2), at which
point does line n intersect with x-axis?
(1) Line m intersects with x-axis at (2, 0).
(2) Line n passes through point (0, 10).

12. What is the area of a rectangle?
(1) The perimeter of the rectangle is 24.
(2) The square of the length of the diagonal is 100.


13. As the figure shown above, the taller tree is 30 feet high and has a 40 feet shadow. What is the
height of the shorter tree?
(1) The shorter tree has a 22 feet shadow.
Tree 1
Tree 2
30
40
22
x
Sun

!

Your way to Top Business Schools _____________
146

(2) The distance between two trees is 18 feet.


14. As the figure above shows, two identical circles are inscribed in a square. If the radius of the
circle is r, what is the length of the diagonal of the square?
(A)
(B)
(C)
(D)
(E) 5r


15. In the figure above, if DE is parallel to , what is the sum of the degree of angle ADE and angle
ACB?
(1) AB = BC.
(2) Angle A is 70 degree.

16. Do integers p and t have the same number of different prime factors?
(1) .
(2) pt = 35.

17. If n is an integer, what is the value of n?
(1) n is positive.
(2) n
2
= 2
n
.

18. About 300 employees in Company X usually use a laptop. What percent of the male employees
usually use a laptop?
(1) 200 female employees usually use a laptop.
(2) 60% of the female employees usually use a laptop.

19. Is quadrilateral ABCD a parallelogram?
(1) Two of the sides have length of 7.
(2) Two of the opposite sides have length of 9.
) 2 4 ( + r
) 2 2 ( + r
) 2 1 ( 2 + r
2 4r
t p
7
5
=
A
B C
D E

!

Your way to Top Business Schools _____________
147


20. A square and an equilateral triangle have the same perimeter. What is the ratio of the area of the
triangle to the area of the square?
(A)
(B)
(C)
(D)
(E)


21. As the figure shown above, four identical circles are tangent to each other and the 4 centers of the
circles are the vertex of a square. If the diameter of the circles is 2, what is the area of the shaded
region?
(A) 16 42
(B) 16 22
(C) 8 22
(D) 4 2
(E) 22 4


22. Square R is inscribed in circle C and C is inscribed in square T. Is the circumference of
greater than 10?
(1) The side length of R is greater than 2.
(2) The side length of T is greater than 4.

23. If one of the sides of a right triangle has length of 10, which of the following could be the
length of other two sides of the triangle?
I. 6 and 8
II. and
III. and
(A) I only
(B) II only
4
3 9
3
3 2
9
3 4
9
3 2
2
3
2
15
2
25
3
40
3
50

!

Your way to Top Business Schools _____________
148

(C) III only
(D) I and III only
(E) I, II and III

24. If 3x + 5y < 15 and 5x + 3y > 15, which of the following mast be true?
I. x > y
II. x < y
III. x > 3
(A) I only
(B) II only
(C) III only
(D) I and II only
(E) I and III only

25. Of the students in a certain class, 40 learn Spanish, 30 learn French, and 20 learn Italian. If each of
her students learn at least one of the languages, how many students learn exactly two of the
languages?
(1) There are 60 students in the class.
(2) 10 of the students learn all of the three languages.

26. The side surface of a cylinder is rolled with a rectangular plate. If the height of a cylinder tank is
12 feet and the perimeter of the circular base Is 9 feet, what is the diagonal of the rectangular
plate?
(A) 13
(B) 15
(C) 18
(D) 20
(E) 21

27. What is the volume of a rectangular solid?
(1) Length of one of sides of the solid is 8.
(2) One of the faces has area of 30 and perimeter of 22.

28. If the hypotenuse of the right triangle is 10, what is the perimeter of the triangle?
(1) The area of the triangle is 25.
(2) The triangle is an isosceles triangle.

!

Your way to Top Business Schools _____________
149


29. In the figure, ADE is an isosceles right triangle above on the square ABCD. If the side length of
the square is 10, what is the area of the whole region?
(A) 110
(B) 115
(C) 120
(D) 125
(E) 133


30. In the figure above, ABCD is a rectangle and DA and CB are radii of the circles shown. If AB = 4,
what is the perimeter of the shaded region?
(A) 2 + 4
(B) 4 + 4
(C) 4 + 8
(D) 8 + 8
(E) 8 + 16

31. Triangle As base is 10% greater than the base of triangle B, and As height is 10% less than the
height of triangle B. The area of triangle A is what percent less or more than the area of triangle
B?
(A) 9 % less
(B) 1 % less
(C) Equal to each other
(D) 1 % more
(E) 9 % more

32. Farmer Brown drives his tractor 40 kilometers. If he travels at r kilometers per hour for one-third
of the distance and 2r kilometers per hour for the remainder, how many hours does the entire trip
take, in terms of r?
(A)
r
24

A B
C D
A
B C
D
E

!

Your way to Top Business Schools _____________
150

(B)
r
36

(C)
r 3
80

(D) 20r
(E)
36
r

33. A sphere is inscribed in a cube with an edge of 10. What is the shortest possible distance from one
of the vertices of the cube to the surface of the sphere?
(A) ( ) 1 3 10
(B) 5
(C) ( ) 1 2 10
(D) ( ) 1 3 5
(E) ( ) 1 2 5

34. How many integers are between 100 and 150, inclusive, cannot be evenly divided by neither 3 nor
5?
(A) 35
(B) 27
(C) 25
(D) 26
(E) 28




!

Your way to Top Business Schools _____________
151

Answers:
1. D
2. C
3. A
4. D
5. A
6. C
7. D
8. C
9. C
10. A
11. C
12. C
13. D
14. C
15. B
16. C
17. E
18. E
19. C
20. C
21. D
22. B
23. E
24. A
25. C
26. B
27. C
28. D
29. D
30. A
31. B
32. C
33. D
34. B

!

Your way to Top Business Schools _____________
152

Lesson 6 Word Problems
Many tasks on GMAT contain the large part of text. Such tasks are named Word Problems. There
are several types of them: Rate Problems, Mixture Problems, Work Problems, Percentage
Problems, Problems with sets. etc.
Lets look at each of these types.

Systems of Linear Equations
At first we will talk about Word Problems that dont use special formulae. These problems can
take a big part of your test time, but their solution is not so hard. You just have to write down and
to solve the system of linear equations or one linear equation.

1. (GC M) Aunt Marge is giving candy to each of her nephews and nieces. She has 20 candy pieces
and she gives candy to children according to her wish. Thus, Robert gets 2 more candy pieces than
Kate. Bill gets 6 less than Mary. Mary, in turn, gets 2 more candy than Robert. Kate gets 2 more
candy than Bill. How many candy pieces does Kate get?
(A) 2
(B) 4
(C) 6
(D) 8
(E) 10

Explanation:
Number of candy given to Robert = r. Similarly pick initial letter of each name as name of
variable (convenience saves time), so other variables are b, m, and k.
Read the problem line by line and write the equations:
r = k + 2
b = m - 6
m = r + 2
k = 2 + b
Let us find the values of b ,m, and k in terms of r:
from equation 1: k = r 2;
from equation 4: b = k - 2 = (r - 2) - 2 = r - 4 ;
from equation 3: m = r + 2.
Since the sum of all these r + b + m + k = 20, substituting each value for expression with r, we
obtain r + r - 4 + r + 2 + r - 2 = 20, so, r = 6. Then
k = r - 2 = 6 - 2 = 4
So the answer is B.

2. (OG M) Bill can buy 3 pairs of jeans and 2 shirts for a price of $69 or he can buy 2 pairs of jeans
and 3 shirts for the price of $66. How much does one shirt cost?
(A) $10
(B) $12
(C) $13.20

!

Your way to Top Business Schools _____________
153

(D) $15
(E) $16.80

Explanation:
Let the price of shirt be s and of jeans j, then we can construct system of equations:
3j + 2s = 69;
2j + 3s = 66.
It is wise to subtract the second from the first: j - s = 3 or j = s + 3. Then plug j in either of the
equations and solve for s. s = $12, so the answer is B.

3. (OG M) Jan lives x floors above the ground floor of a high-rise building. It takes her 30 seconds
per floor to walk down and 2 seconds per floor to ride the elevator. If it takes Jan the same amount
of time to walk down the steps to the ground floor as to wait for the elevator for 7 minutes and
ride down, then x equals
(A) 10
(B) 12
(C) 13
(D) 14
(E) 15

Explanation:
If it takes Jan 30 seconds per floor to walk down and she lives x floors above the ground floor then
total time needed to walk down the steps to the ground floor is 30x seconds. As for elevator, it will
take 2x seconds to ride down to the ground floor. If it takes Jan the same amount of time to walk
down the steps to the ground floor as to wait for the elevator for 7 minutes (=420 seconds) and
ride down, we can write down such an equation:
30x =2x +420;
28x = 420;
x= 15. The answer is E.

4. (OG E) Salesperson A's compensation for any week is $360 plus 6 percent of the portion of A's
total sales above $1000 for that week. Salesperson B's compensation for any week is 8 percent of
B's total sales for that week. For what amount of total weekly sales would both salespeople earn
the same compensation?
(A) $21,000
(B) $18,000
(C) $15,000
(D) $4,500
(E) $4,000

Explanation:
As compensation for any week can be written as 360 + 0.06(t-1000), where t is amount of total
sales. Bs compensation is 0.08t. To find for what amount of total weekly sales would both
salespeople earn the same compensation we have to solve the equation:

!

Your way to Top Business Schools _____________
154

360 + 0.06(t-1000) =0.08t;
360 +0.06t -60 -0.08t =0;
0.02t =300;
t=15 000. The answer is C.

Venn diagrams are sometimes used in such problems. They will help you to write down the
linear equation and to find the solution.

On this picture you can see two sets S and T. The grey portion denotes the common part for these
two sets. Lets have a look, how this can help you to solve the problem.

5. In a school, 10 students are taking math class and 6 are taking literature class. 4 students are taking
both classes. How many students are taking either math or literature class?
(A) 8
(B) 10
(C) 12
(D) 14
(E) 16
Lets draw the Venn diagram.

We are looking for the sum of the number of only Math students, only literature students, and
students of both subjects.
Only Math can be counted as 10 (students taking math class) 4 (students taking both classes) = 6.
Only literature can be counted as 6(students taking literature class) 4 (students taking both
classes) = 2. So, our diagram now looks like

!

Your way to Top Business Schools _____________
155


So, we can find the answer as 6+4+2 = 12, which is answer C.
6. (OG PS) Of the 65 cars on a car lot, 45 have air-conditioning, 30 have power windows, and 12
have both air conditioning and power windows. How many of the cars on the lot have neither air-
conditioning nor power windows?
(A) 2
(B) 8
(C) 10
(D) 15
(E) 18
Explanation
Lets draw the Venn diagram for this problem.

The outer circle denotes all cars that are on the lot; a number of cars that have only air
conditioning; p number of cars that have only power windows; b - have both air conditioning
and power windows; n - have neither air-conditioning nor power windows.
Now we can substitute numbers: total number of cars is 65; a = 45 12=33, b= 30-12 =18.

!

Your way to Top Business Schools _____________
156


We can write down the equation 65 = 18+12+33+n, or simply n=2. That is the answer.



!

Your way to Top Business Schools _____________
157

Rate problems
For successful solving rate problems you need to remember two rules:
1)The distance that an object travels is equal to the product of the average speed at which it travels
and
the amount of time it takes to travel that distance that is

RateTime=Distance.

2) To determine the average rate at which an object travels, divide the total distance traveled by
the total amount of traveling time.
Average speed =
tutal dtstane
tutal ttme
.

But these formulae allow several extensions.
For example, when 2 objects are moving in the same direction, their relative speed is the
difference between speed of each of them.
Relative speed (overtaking)= speed (faster object ) - speed (slower object)

Similarly, relative speed of oncoming object is sum of both speeds:
Relative speed (oncoming)= speed (object 1) + speed (object 2)

It is usually the case, when something is swimming in the river speed of the stream should be
taken into account.
Now lets apply this methodology to some rate problems.

7. (OG M) A hiker walked for 2 days. On the second day the hiker walked 2 hours longer and at an
average speed 1 mile per hour faster than he walked on the first day. If during the two days he
walked a total of 64 miles and spent a total of 18 hours walking, what was his average speed on
the first day?








Explanation:
If the time spent walking on the first day is t
1
and the time spent walking on the second day is t
2

then t
1
+t
2
= 18.
We know that on the second day the hiker walked 2 hours longer, so, t
2
= t
1
+1. Solving the
system of these two equations we get that walker spent 8 hours walking on the first day and 10
hours on the second day.
(A) 2
() 3
() 4
(D) 5
(E) 6

!

Your way to Top Business Schools _____________
158

If his speed on the first day was v
1
and the speed on the second day was v
2
, the distance he walked
a total is d = t
1
v
1
+ t
2
v
2
or 64 = 8v
1
+ 10v
2
and, as on second day his average speed was 1 mile
per hour greater, v
1
= v
2
1.
So, we get 64=8(v
2
1) +10 v
2
. Thus, v
1
= 3.

8. (GC M) A passenger sitting near the window in a train moving at 40 km/h noticed that it took 3
seconds for the oncoming train to pass by. What was the speed of the oncoming train if the length
of the oncoming train was 75 meters?
(A) 50 km/h
(B) 52 km/h
(C) 56 km/h
(D) 60 km/h
(E) 70 km/h

Explanation:
Here we have 2 objects train 1 and train 2. Speed of train 1 is 40 km/h. Relative speed of two
trains is equal to 75m/3s=25 m/s. It can be expressed as 90 km/h, because 1 m/s = 3.6 km/h. Since
train 2 is oncoming, its speed is the difference between relative speed and the speed of the train 1:
Train 2 speed= 90 40 = 50 km/h.
Here we used formula:
Relative speed (oncoming)= speed (object 1) + speed (object 2)
The right answer is A.

9. (OG M) One hour after Yolanda started walking from X to Y, a distance of 45 miles, Bob started
walking along the same road from Y to X. If Yolanda's walking rate was 3 miles per hour and
Bob's was 4 miles per hour, how many miles had Bob walked when they met?








Explanation:
If t denotes time that Bob spent walking, then (t+1) is the time that Yolanda spent walking.
Distance which Yolanda covered is 3(t + 1), and distance which Bob covered is 4t. The total
distance then is:
4t + 3(t + 1) = 45.
So, t = 6 and Bob walked 6x4 = 24 miles. Therefore, A is the answer.

(A) 24
(B) 23
() 22
(D) 21
(E) 19.5

!

Your way to Top Business Schools _____________
159

Work problems
In a work problem, the rates at which certain persons or machines work alone are usually given,
and it is necessary to compute the rate at which they work together (or vice versa).
The work speed can be counted in such a way:
Speed =
tutal wurh dune
tutal ttme
.
If two persons are working at a certain job together their work speed can be expressed as the sum
of their individual speeds:

tutal wurh dune
tutal ttme
=
tutal wurh dune
ttme uI the Itrst
+
tutal wurh dune
ttme uI the seund
.

If we divide both parts of this equation by total work done, well get the basic formula for
solving work problems:
1/t
1
+1/t
2
= 1/T,

where t
1
is the number of hours it takes first person to complete a job when working alone, t
2
is
the number of hours it takes second person to complete a job when working alone, and T is the
number of hours it takes them to do the job when working together.

10. (OG, E) It takes one tube 6 hours to fill the dish with water. If it takes 4 hours for another pipe to
do the same job, how much time it would take for both pipes to fill the same dish?









Explanation:
Using our formula, we get
1/T = 1/4 + 1/6 = (3+2)/12 =5/12,
Therefore, T = 12/5 = 2.4 hours, thus, E is the answer.

11. (GC, M) If Samson is filling a bathtub with COLD water, it will take him 6 minutes and 40
seconds, and if he fills it with HOT water, it will take him 8 minutes. If draining the tub takes 13
minutes 20 seconds, how many minutes will it take to fill up the bath tub with both HOT and
COLD water running while the plug is out, so the water is constantly draining?
(A) 4.75
(B) 5
(C) 8.6
(D) 12
(E) 16
(A) 1
(B) 4.3
(C) 5/12
(D) 2
(E) 2.4

!

Your way to Top Business Schools _____________
160

Explanation:
Lets count the rate for each variant:
COLD: 400 seconds to fill the whole tub or 1/400 tub a second;
HOT: 480 seconds to fill the whole tub or 1/480 tub a second;
Drain: 800 seconds to drain the whole tub or 1/800 tub a second.
We get water inflow/outflow per second:
1/400 + 1/480 - 1/800 = 12/4800 + 10/4800 - 6/4800 = 16/4800 = 1/300 = 1/T
T =300.
So, it takes 300 seconds, or 5 minutes. The answer is (A).


!

Your way to Top Business Schools _____________
161

Mixture Problems
In mixture problems, substances with different characteristics are combined, and it is necessary to
determine the characteristics of the resulting mixture.
Concentration of the substance in some mixture can be determined as:

Concentration of the substance =
vulume uI the substane
tutal vulume uI the mtxture
.

The main property for solving mixture problems is:

x
1

1+
x
2

2
x
1
+x
2
= C
Where:
x
1
determines the volume of the first substance,
x
2
determines the volume of the second substance,
c
1
concentration of the first substance,
c
2
concentration of the second substance,
C concentration of the mixture.

Remember that mixture formula can be used not only for chemical substances. See the following
examples.
12. (OG M) For an agricultural experiment, 300 seeds were planted in one plot and 200 were planted
in a second plot. If exactly 25 percent of the seeds in the first plot germinated and exactly 35
percent of the seeds in the second plot germinated, what percent of the total number of seeds
germinated?
(A) 12%
(B) 26%
(C) 29%
(D) 30%
(E) 60%

Explanation:
If the numbei of seeus in the fiist plot is the volume of the fiist substance: x
1
= Suu,
the numbei of seeus in the fiist plot is the volume of the seconu substance: x
2
= 2uu,
Percent of germinated seeds is the concentration: c
1
= 2S, c
2
= SS,
And the consentiation of the mixture is the percent of the total number of seeds germinated, we
can write down such an equation:
30025+20035
300+200
= C;
C =
100 (325+235)
500
=
75+70
5
=
145
5
= 29 %.
So, the answer is C.

13. (GC M) If 12 ounces of strong solution of vinegar is mixed with 50 ounces of water to form three
percent vinegar, what was the original strength of the vinegar solution?
(A) 19.3

!

Your way to Top Business Schools _____________
162

(B) 17
(C) 16.67
(D) 15.5
(E) 12.5

Explanation:
50 = x
1
ounces of water that have 0% = c
1
of vinegar and 12 = x
2
ounces of strong solution have
c
2
% of vinegar. The resulting mixture has 3% = C of vinegar. Using our formula we get:
Su u +12 c
2
Su + 12
= S
Solving this equation we will get c
2
= 15.5%
The answer is D.

14. (OG M) A club sold an average of 92 raffle tickets per member. Among the female members, the
average number sold was 84, and among the male members, the average number sold was 96.
What was the ratio of the number of male members to the number of female members in the club?
(A) 1:1
(B) 1:2
(C) 1:3
(D) 2:1
(E) 3:1

Explanation:
Let the number of female members be f and the number of male members be m. Then we can
write down such equation:
84I+96m
I+m
= 92. If we multiple both parts by (f+m), we get:
84f +96m =92 (f+m);
84f +96m = 92f +92m;
4m =8f;
m =2f.
We need to find the the ratio of the number of male members to the number of female members in
the club, or simply m/f. From the last equation we can find m/f =2/1. The answer is D.
Percentage & profit problems
In the problems with interest, you have to remember two basic moments: if the price increased by
10%, you can get the new price multiplying the basic by 1.1. If the price decreased by 10%, you
can get the new price multiplying the basic by 0.9.
Interest can be computed in two ways. With simple annual interest, amount we get equals:
S*=S(1+in). If interest is compounded, then amount we get equals: S*=S(1 +i)
n
. Here S is
amount we put, i interest rate, n- number of time intervals.
As for profit problems, they can be solved using one simple rule: Gross profit is equal to revenues
minus expenses, or selling price minus cost.

!

Your way to Top Business Schools _____________
163

15. (OG E) The price of a certain television set is discounted by 10 percent, and the reduced price is
then discounted by 10 percent. This series of successive discounts is equivalent to a single
discount of
(A) 20%
(B) 19%
(C) 18%
(D) 11%
(E) 10%

Explanation:
If the price of the television is p, after first reducing it becomes p-0.1p =0.9p and after second
reducing: 0.9p u.9 p=0.81p. So, such series of successive discounts is equivalent to a single
discount of p-0.81p =0.19p or simply 19%. The answer is B.

16. (GC M) There are three kinds of business A, B and C in a company. 25 percent of the total
revenue is from business A; t percent of the total revenue is from B, the others are from C. If B is
$150,000 and C is the difference of total revenue and 225,000, what is t?
(A)50
(B)70
(C)80
(D)90
(E)100

Explanation:
If the total revenue is x, we can write such a system of equations:
A =0.25 x
B =
t
100
x
A +B +C =x
B =150,000
C =x -225,000
We need to find t, the other variables are not interesting for us.
B =150,000=
t
100
x, so, t =15,000,000/x.
A+B+C=x can be re-written as
0.25x + 150 000 + x 225 000 =x;
0.25x = 75 000
x = 300,000.
So, t =15,000,000/x =15,000,000/300,000 =50.
We can answer A.

!

Your way to Top Business Schools _____________
164

Home assignment.
Rate problems

1. (OG E) How many miles long is the route from Houghton to Callahan?
(1) It will take 1 hour less time to travel the entire route at an average rate of 55 miles per hour than at
an average rate of 50 miles per hour
(2) It will take 11 hours to travel the first half of the route at an average rate of 25 miles per hour

2. (MH H) The time it took car P to travel 600 miles was 2 hours less than the time it took car R
to travel the same distance. If car Ps average speed was 10 miles per hour greater than that of
car R, what was car Rs average speed, in miles per hour?
(A) 30
(B) 40
(C) 50
(D) 60
(E) 70

3. (MH M) A hiker walked for 3 days. She walked 18 miles on the first day, walking 3 miles per
hour. On the second day she walked for one less hour but she walked one mile per hour, faster
than on the first day. On the third day she walked the same number of hours as on the first day, but
at the same speed as on the second day. How many miles in total did she walk?
(A) 60
(B) 58
(C) 62
(D) 24
(E)44

4. (OG M) If it took Mike 3 hours to ride 162 kilometres, what was his average speed, meters per
second?
(A) 1.5
(B) 5
(C) 5.4
(D) 15
(E) 54

5. (GC E) Maureen runs a 26-mile marathon in 5.4 hours. If she runs the first half of the race in 2.8
hours, what is her average rate, in miles per hour, for the second half of the race?
(A) 10
(B) 9
2
7

(C) 7
(D) 6
1
2

(E) 5

!

Your way to Top Business Schools _____________
165


6. (GC H) A cyclist rides his bicycle over a route which is 1/3 uphill, 1/3 level, and 1/3 downhill. If he
covers the uphill part of the route at the rate of 16 miles per hour and the level part at the rate of
24 miles per hour, what rate in miles per hour would he have to travel the downhill part of the
route in order to average 24 miles per hour for the entire route?
(A) 32
(B) 36
(C) 40
(D) 44
(E) 48

7. (GC H) The time it took car A to travel 400 miles was 2 hours less than the time it took car to
travel the same distance. If car A's average speed was 10 miles per hour greater than that of car B,
what was car B's average speed in miles per hour?
(A) 20
(B) 30
(C) 40
(D) 50
(E) 80

8. (GP M) A bus trip of 450 miles would have taken 1 hour less if the average speed S for the trip
had been greater by 5 miles per hour. What was the average speed S, in miles per hour, for the trip?
(A) 10
(B) 40
(C) 45
(D) 50
(E) 55
9. (GP E) In traveling from a dormitory to a certain city, a student went
5
1
of the way by foot,
3
2
of
the way by bus, and the remaining 8 kilometers by car. What is the distance, in kilometers, from
the dormitory to the city?
(A) 30
(B) 45
(C) 60
(D) 90
(E) 120

10. (GP) If a motorist had driven 1 hour longer on a certain day and at an average rate of 5 miles per
hour faster, he would have covered 70 more miles than he actually did. How many more miles
would he have covered than he actually did if he had driven 2 hours longer and at an average rate
of 10 miles per hour faster on that day?
(A) 100
(B) 120
(C) 140

!

Your way to Top Business Schools _____________
166

(D) 150
(E) 160

Work Problems

1. (GC) Bill takes 20 days to empty a 50 - liter barrel of beer and, together with Henry, he can empty
it in 14 days, how many days will it take Henry to finish a barrel of beer by himself?
(A) 17
(B) 33
(C) 46
(D) 47
(E) 58

2. (GC) In a TV factory 9 persons can assemble 10 tv sets in 20 days of 7 1/2 working hours. How
long will it take for 12 persons to assemble 20 tv sets working 6h per day, it being given that 2
persons in the latter case do work as much as 3 men in the former? 25
(A) 10
(B) 12.5
(C) 20
(D) 25
(E) 50

3. (GC) Water flows into an empty tank of 54 liters via 12 small pipes. The rate of each pipe is 1 liter
per hour. However, water flows out of the tank via several big pipes at the rate of 1.5 liter per
hour. If after 12 hours, the tank is completely full, how many big pipes are there?
(A) 2.5
(B) 3
(C) 5
(D) 6
(E) 7

4. (MG) Mr. Bridges can wash his car in 15 minutes, while his son Dave takes twice as long to do
the same job. If they work together, how many minutes will the job take them?
(A) 5
(B) 7.5
(C) 8
(D) 10
(E) 22.5

5. (GC) It takes 6 technicians 10 hours to build and program a new server from Direct Computer,
with each working at the same rate. If six technicians start to build the server at 11:00 AM, and
one technician per hour is added beginning at 5:00 PM, at what time will the server be complete?
(A) 6:40 PM

!

Your way to Top Business Schools _____________
167

(B) 7:00 PM
(C) 7:20 PM
(D) 7:40 PM
(E) 8:00 PM

6. (MH) If a copier makes 3 copies every 4 seconds, then continues at this rate, how many minutes
will it take to make 9,000 copies?
(A) 3,000
(B) 200
(C) 120
(D) 100
(E) 60

7. (MH) An employee at a company was given the task of making a large number of copies. He
spent the first 45 minutes making copies at a constant rate on copier A, but copier A broke down
before the task was completed. He then spent the next 30 minutes finishing the task on copier B,
which also produced copies at a constant rate. How many total minutes would the task have taken
had copier A not broken down?
(1) Copier B produces 10 copies per minute.
(2) Copier A produced twice as many copies in its first 5 minutes of operation as copier B
produced in its first 15 minutes.

8. (OG) Tree machines, individually, can do a certain job in 4, 5, and 6 hours, respectively. What is
the greatest part of the job that can be done in one hour by two of the machines working together
at their respective rates?
(A) 11/30
(B) 9/20
(C) 3/5
(D) 11/15
(E) 5/6

9. (GC) Machine A can produce 50 components a day while machine B only 40. The monthly
maintenance cost for machine A is $1500 while that for machine B is $550. If each component
generates an income of $10, what is the least number of days per month that the plant has to work
to justify the usage of machine A instead of machine B?

(A) 11
(B) 10
(C) 9
(D) 7
(E) 6

10. (OG) Hoses X and Y simultaneously fill an empty swimming pool that has a capacity of 50,000
liters. If the flow in these hoses is independent, how many hours will it take to fill the pool?

!

Your way to Top Business Schools _____________
168

(1) Hose X alone would take 28 hours to fill the pool.
(2) Hose Y alone would take 36 hours to fill the pool.

Systems of linear equations

1. (OG) Marion rented a car for $18.00 plus $0.10 per mile driven. Craig rented a car for $25.00 plus
$0.05 per mile driven. If each drove d miles and each was charged the same amount for the rental,
then d equals
(A) 100
(B) 120
(C) 135
(D) 140
(E) 150

2. (PR) A tourist has travelers checks in $20 and $100 denominations. How many $20 checks are
there?
(1) If half of the $20 checks are spent, the remaining amount is $520.
(2) The total value of the checks is $740

3. (OG) In a weight-lifting competition, the total weight of Joe's two lifts was 750 pounds. If twice
the weight of his first lift was 300 pounds more than the weight of his second lift, what was the
weight, in pounds, of his first lift?
(A) 225
(B) 275
(C) 325
(D) 350
(E) 400

4. (OG) Jack is now 14 years older than Bill. If in 10 years Jack will be twice as old as Bill, how old
will Jack be in 5 years?
(A) 9
(B) 19
(C) 21
(D) 23
(E) 33

5. (OG) A sum of $200,000 from a certain estate was divided among a spouse and three children.
How much of the estate did the youngest child receive?
(1) The spouse received 1/2 of the sum from the estate, and the oldest child received 1/4 of the
remainder
(2) Each of the two younger children received $12,500 more than the oldest child and $62,500 less
than the spouse

!

Your way to Top Business Schools _____________
169


6. (OG) If the Lincoln Library's total expenditure for books, periodicals, and newspapers last year
was $35,000, how much of the expenditure was for books?
(1) The expenditure for newspaper was 40 percent greater than the expenditure for periodicals
(2) The total of the expenditure for periodicals and newspapers was 25 percent less than the
expenditure for books

7. (OG) A certain bakery sells rye bread in 16-ounce loaves and 24-ounce loaves, and all loaves of
the same size sell for the same price per loaf regardless of the number of loaves purchased. What
is the price of a 24-ounce loaf of rye bread in this bakery?
(1) The total price of a 16-ounce loaf and a 24-ounce loaf of this bread is $2.40
(2) The total price of two 16-ounce loaves and one 24-ounce loaf of this bread is $3.40

8. (OG) In a certain company the ratio of the number of managers to the number of production-line
workers is 5 to 72. If 8 additional production-line workers were to be hired, the ratio of the
number of managers to the number of production-line workers would be 5 to 74, How many
managers does the company have?
(A) 5
(B) 10
(C) 15
(D) 20
(E) 25

9. (OG) The sum of the ages of Doris and Fred is y years. If Doris is 12 years older than Fred, how
many years will Fred be y years from now, in terms of y?
(A) y-6
(B) 2y-6
(C) y/2-6
(D) 3y/2-6
(E) 5y/2-6

10. (OG) In a certain furniture store, each week Nancy earns a salary of $240 plus a 5 percent of the
amount of her total sales that exceeds $800 for the week. If Nancy earned a total $450 one week,
what were her total sales that week?
(A) $2200
(B) $3450
(C) $4200
(D) $4250
(E) $5000

11. (OG) What is the number of female employees in Company X?
(1) If Company X were to hire 14 more workers and all of these workers were females, the ratio of
the number of male employees to the number of female employees would then be 16 to 9
(2) Company X has 105 more male employees than female employees

!

Your way to Top Business Schools _____________
170


12. (OG) A swim club that sold only individual and family memberships charged $300 for an
individual membership. If the club's total revenue from memberships was $480,000, what was the
charge for a family membership?
(1) The revenue from individual memberships was 1/4 of the total revenue from memberships
(2) The club sold 1.5 times as many family memberships as individual memberships

13. (OG) Currently there are 50 picture books on each shelf in the children's section of a library. If
these books were to be placed on smaller shelves with 30 picture books on each shelf, how many
of the smaller shelves would be needed to hold all these books?
(1)The number of smaller sieves needed is 6 more than the current number of shelves
(2)Currently there are 9 shelves in the children's section

14. (OG) If Sara's age is exactly twice Bill's age, what is Sara's age?
(1) Four years ago, Sara's age was exactly 3 times Bill's age
(2) Eight years from now, Sara's age will be exactly 1.5 times Bill's age

15. (OG) If Aaron, Lee, and Tony have a total of $36, how much money does Tony have?
(1) Tony has twice as much money as Lee and 1/3 as much as Aaron
(2) The sum of the amounts of money that Tony and Lee have is half the amount that Aaron
has

16. (OG) A bookstore that sells used books sells each of its paperback books for a certain price and
each of its hardcover books for a certain price. If Joe, Marina, and Paul bought books in this store,
how much did Maria pay for 1 paperback book and 1 hardcover book?
(1) Joe bought 2 paperback books and 3 hardcover books for $12.50
(2) Paul bought 4 paperback books and 6 hardcover books for $25.00

17. (GC) Aunt Marge is giving candy to each of her nephews and nieces. She has 20 candy pieces and
she gives candy to children according to her wish. Thus, Robert gets 2 more candy pieces than
Kate. Bill gets 6 less than Mary. Mary, in turn, gets 2 more candy than Robert. Kate gets 2 more
candy than Bill. How many candy pieces does Kate get?
(A) 2
(B) 4
(C) 6
(D) 8
(E) 10

18. (GC) Mary bought some kiwis, bananas, and lemons at the grocery store in proportion of 1:4:7
accordingly. How many lemons did Mary buy?
(1) The total number of fruits Mary bought is 24.
(2) Mary bought 8 bananas.

!

Your way to Top Business Schools _____________
171

19. (GC) One fisherman was telling his friends that he caught a fish that had a 60 foot long head. It
also had a tail that was the size of the fish's head and a half of its body, and the body was half the
size of the whole fish. What is the length of this fish?
(A) 120
(B) 200
(C)240
(D)400
(E) 480

20. (GC) If the farmer sells 75 of his chickens, his stock of feed will last for 20 more days than
planned, but if he buys 100 more chickens, he will run out of feed 15 days earlier than planned. If
no chickens are sold or bought, the farmer will be exactly on schedule. How many chickens does
the farmer have?
(A) 60
(B) 120
(C) 240
(D) 275
(E) 300

Mixture Problems

1. (OG) If 3 pounds of dried apricots that cost x dollars per pound are mixed with 2 pounds of
prunes that cost y dollars per pound, what is the cost, in dollars, per pound of the mixture?
(A) (3x+2y)/5
(B) (3x+2y)/(x+y)
(C) (3x+2y)/(xy)
(D) 5(3x+2y)
(E) 3x+2y

2. (OG) A rabbit on a controlled diet is fed daily 300 grams of a mixture of two foods, food X and
food Y. Food X contains 10 percent protein and food Y contains 15 percent protein. If the rabbit's
diet provides exactly 38 grams of protein daily, how many grams of food X are in the mixture?
(A) 100
(B) 140
(C) 150
(D) 160
(E) 200

3. (OG) A $500 investment and a $1,500 investment have a combined yearly return of 8.5
percent of the total of the two investments. If the $500 investment has a yearly return of 7 percent,
what percent yearly return does the $1,500 investment have?
(A) 9%
(B) 10%

!

Your way to Top Business Schools _____________
172

(C) 10.625%
(D) 11%
(E) 12%

4. (OG) On Jane's credit card account, the average daily balance for a 30-day billing cycle is the
average of the daily balances at the end of each of the 30 days. At the beginning of a certain 30-
day billing cycle, Jane's credit card account had a balance of $600. Jane made a payment of $300
on the account during the billing cycle. If no other amounts were added to or subtracted from the
account during the billing cycle, what was the average daily balance on Jane's account for the
billing cycle?
(1) Jane's payment was credited on the 21
st
day of the billing cycle
(2) The average daily balance through the 25
th
day of the billing cycle was $450

5. (GC) If 12 ounces of strong solution of vinegar is mixed with 50 ounces of water to form three
percent vinegar, what was the original strength of the vinegar solution?
(A) 16.67
(B) 12.5
(C) 15.5
(D) 19.3
(E) 17

6. (MG) How many ounces of pure acid must be added to 20 ounces of a solution that is 5% acid
to strengthen it to a solution that is 24% acid?
(A) 5
(B) 5.5
(C) 6
(D) 7.5
(E) 10

7. (GC) How must a grocer mix 4 types of peanuts worth 54 c, 72 c, $1.2 and $1.44 per pound so
as to obtain a mixture at 96 cents per pound?
(A) 8:4:4:7
(B) 24:12:12:50
(C) 4:8:7:4
(D) 16:42:28:10
(E) Can not be uniquely determined

8. (GC) In what proportion must sugar at $3.1 per pound be mixed with sugar at $3.6 per pound,
so that the mixture be worth $3.25 per pound?
(A) 1:2
(B) 3:2
(C) 7:3
(D) 3:7
(E) 2:3

!

Your way to Top Business Schools _____________
173


9. (GC) Committee X and Committee Y, which have no common members, will combine to
form Committee Z. Does Committee X have more members than Committee Y?
(1) The average (arithmetic mean) age of the members of Committee X is 25.7 years and the
average age of the members of Committee Y is 29.3 years
(2) The average (arithmetic mean) age of the members of Committee Z will be 26.6 years

10. (OG) During a certain season, a team won 80 percent of its first 100 games and 50 percent
of its remaining games. If the team won 70 percent of its games for the entire season, what was the
total number of games the team played?
(A) 180
(B) 170
(C) 156
(D) 150
(E) 105
Percentage & profit problems

1. (GC) If today the price of an item is $3,600, what was the price of the item exactly 2 years ago?
(1) The price of the item increased by 10 percent per year during this 2-year period
(2) Today the price of the item is 1.21 times its price exactly 2 years ago

2. (OG) Tickets for all but 100 seats in a 10,000-seat stadium were sold. Of the tickets sold, 20
percent were sold at half of the price and the remaining tickets were sold at the full price of $2.
What was the total revenue from ticket sales?
(A) $15,840
(B) $17,820
(C) $18,000
(D) $19,800
(E) $21,780

3. (OG) Increasing the original price of an article by 15 percent and then increasing the new price by
15 percent is equivalent to increasing the original price by
(A) 32.25%
(B) 31.00%
(C) 30.25%
(D) 30.00%
(E) 22.50%

4. (GC) $20,000 was deposited into bank, with interest compound quarterly. What is compound
interest?
(1) The interest in 2nd quarter is 1.1 times more than the first quarter
(2) The interest in 2nd quarter is 2 dollars more than the first quarter

!

Your way to Top Business Schools _____________
174


5. (OG) A certain manufacturer produces items which the production costs consist of annual fixed
costs totalling $130,000 and variable costs averaging $8 per item. If the manufacturer's selling
price per item is $15, how many items must the manufacturer produce and sell to earn an annual
profit of $150,000?
(A) 2,858
(B) 18,667
(C) 21,429
(D) 35,000
(E) 40,000

6. (OG) An author received $0.80 in royalties for each of the first 100,000 copies of her book sold,
and $0.60 in royalties for each additional copy sold. If she received a total of $260,000 in
royalties, how many copies of her book were sold?
(A) 130,000
(B) 300,000
(C) 380,000
(D) 400,000
(E) 420,000

7. (OG) Henry purchased 3 items during a sale. He received a 20 percent discount off the regular
price of the most expensive item and 10 percent discount off the regular price of each of the other
2 items. Was the total amount of the 3 discounts greater than 15 percent of the sum of the regular
prices of the 3 items?
(1) The regular price of the most expensive item was $50, and the regular price of the next most
expensive item was $20
(2) The regular price of the least expensive item was $15

8. (GC) The price of a certain commodity increased at a rate of X% per year between 2000 and 2004.
If the price was A dollars in 2001 and B dollars in 2003, what was the price in 2002 in terms of A
and B?
(A) (A B)
(B) B (B/A)
(C) B A
(D)B A/B
(E) B A
3/2

9. (GC) If the circus were to sell all of its 220 tickets for this month's performance at their usual
price, the revenue from sales would be 10% greater than that collected last month. However, the
circus raised the ticket price by 5% and sold only 200 tickets as a result. What percent less was
last month's revenue than that of this month?
(A) 2
(B) 4
(C) 110/20

!

Your way to Top Business Schools _____________
175

(D) 100/21
(E) 8/4

10. (GC) A filter decreases concentration of harmful substances in the water by 50%. How many
filters are required to clean 200 liters of water containing 0.5 liters of harmful substances if the
water that contains less than 0.1 percent of harmful substances is considered clean?
(A) 8
(B) 6
(C) 5
(D) 4
(E) 2

!

Your way to Top Business Schools _____________
176

Test 6 Word problems
1. Of the 200 students in a college, 125 learned Spanish and 81 learned French. How many students
learn neither Spanish nor French?
(1) 20 percent of the students who learned Spanish also learned French.
(2) 50 out of the students who learn French did learn Spanish.

2. Train A leaves the station traveling at 30 miles per hour. Two hours later train leaves the same
station traveling in the same direction at 40 miles per hour. How far from the station was train A
overtaken by train B?
(A) 180
(B) 240
(C) 270
(D) 300
(E) 320

3. Each of six students, four girls and two boys, spent an average of $60 on books. If the girls spent a
total amount of $300, how much did boy X and Y spends on books?
(1) X spent twice as much as Y spent on books.
(2) X spent $20 more than Y on books.

4. Of the students in a certain class, 55% of the female and 35% of the male passed an exam. Did
more than half of the students in the class pass the exam?
(1) More than half of the students in the class are female.
(2) The number of the female students is 20 more than the number of the male students.

5. Jim traveled from X to Y, and arrived at Z. If the average speed for the whole trip was 60 miles
per hour, what is the average speed for the trip from Y to Z?
(1) The average speed for the trip from X to Y is 55 miles per hour.
(2) It took Jim 0.5 hour from Y to Z.

6. A riverboat leaves Mildura and travels upstream to Renmark at an average speed of 6 miles per
hour. It returns by the same route at an average speed of 9 miles per hour. What is its average
speed for the round trip, in miles per hour?
(A) 7.0
(B) 7.2
(C) 7.5
(D) 7.8
(E) 8.2

7. There are only three kinds of books on a bookshelf, science book, novel book, and workbooks, in
the ratio of 2 to 5 to 7. What is the total number of the books on the shelf?
(1) There are less than 5 science books on the shelf.
(2) The total number of novel book and workbook is 24.

!

Your way to Top Business Schools _____________
177


8. If 0 < n < m < 1, m
2
n
2
must be less than which of the following expressions?
(A) m n
(B)
(C) mn
(D) (m + n)
2
(E) (m n)
2
9. Steve took a math test that consists of basic questions and advanced questions. A correct answer to
the basic question will gain 1 point and a correct answer to advanced question will gain 2 points.
How many questions does the test consist of?
(1) Steve answered 80% of the basic question and 30% of the advanced questions correctly, and
gained 88 points.
(2) There are twice as many advanced questions as the basic question.

10. A set consists of 12 numbers, all are even or multiple of 5. If 5 numbers are even and 8 numbers
are multiple of 5, how many numbers is multiple of 10?
(A) 0
(B) 1
(C) 2
(D) 3
(E) 5

11. Seven students each bought some books on a book fair. Is the total number of books they bought
greater than 27?
(1) No student bought more than 7 books.
(2) No two students bought same number of books.

12. Of the people stand in a line waiting for service, 20% have waited for more than 5 minutes and 8%
have waited for more than 8 minutes. How many people have waited for more than 5 minutes and
no more than 8 minutes?
(1) 20 people waited for no more than 5 minutes.
(2) 2 people have waited for more than 8 minutes.

13. On a certain date, Jack invested $13,000 at x percent simple annual interest and a different amount
at percent simple annual interest. If he earned same amount of interest from two investments,
what amount did Jack invest at percent simple annual interest?
(1) The amount of interest earned by Jack from the investment that pay x% interest was $390.
(2) The ratio of x to is 3 to 2.

14. Maura drives to work in 40 minutes. She takes the same route to return home. If her average speed
on the trip home is half as fast as her average speed on the trip to work, how much time does she
spend driving on the round trip?
(A) 1 hour
2
n m +

!

Your way to Top Business Schools _____________
178

(B) 1 hour, 15 minutes
(C) 1 hour, 20 minutes
(D) 1 hour, 40 minutes
(E) 2 hours

15. If m, n, and m
n
are nonzero integers, which of the following must be true?
I. m
n
is positive
II. n
m
is an integer
III. n
m
is an integer
(A) None
(B) I only
(C) II only
(D) III only
(E) I and II only

16.What is the remainder when positive integer x is divided by 6?
(1) 15x is divisible by 2.
(2) 15x is divisible by 3.

17. Which of the following must be greater than x?
I. 2x
II. x
2
III. x
2
+ 1
(A) I only
(B) II only
(C) III only
(D) I and II only
(E) II and III only

18. Working together at their respective rates, machine , , and can finish a certain work in
hours. How many hours will it take A to finish the work independently?
(1) Working together, A and can finish the work in 4 hours.
(2) Working together, and can finish the work in hours.

19. of the employees at a global company are female. If all the employees are from American,
Canadian, or Mexican, what percent of the male employees are Canadian?
(1) of the female are American or Mexican.
(2) of all the employees are Canadian.

20. One inlet pipe fills an empty tank in 5 hours. A second inlet pipe fills the same tank in 3 hours. If
3
8
7
48
4
3
5
4
3
1

!

Your way to Top Business Schools _____________
179

both pipes are used together, how long will it take to fill of the tank?
(A)
(B)
(C)
(D)
(E)

21. In a survey of 200 college graduates, 30 percent said they had received student loans during their
college careers, and 40 percent said they had received scholarships. What percent of those
surveyed said that they had received neither student loans nor scholarships during their college
careers?
(1) 25 percent of those surveyed said that they had received scholarships but no loans.
(2) 50 percent of those surveyed who said that they had received loans also said that they had received
scholarships.

22. Machine A can finish of a work in 4 hours, while can finish of the work in 2 hours, and
can finish of the work in 8 hours. How many hours will it take for A, B, and together to
finish the work?
(A)
(B)
(C)
(D)
(E)

23. John always jogs to school at a speed of 6 kilometers per hour, and walks home along the same
route at a speed of 3 kilometers per hour. If he spends exactly one hour total traveling both ways,
how many kilometers is his school from his home?

(A) 2
(B) 3
(C) 4
(D) 6
(E) 9

3
2
15
8
8
15
4
3
4
5
3
8
4
1
6
1
3
1
3
8
3
14
3
16
8
7
3
7

!

Your way to Top Business Schools _____________
180

24. What is the remainder when x is divided by 12?
(1) When x is divided by 4, the remainder is 3.
(2) When x is divided by 36, the remainder is 27.

25. Of the residents in Stamford, 75% have a laptop, 40% have both laptop and desktop, and 5% have
none. What percent of the residents have only desktop?
(A) 10 %
(B) 15 %
(C) 20 %
(D) 25 %
(E) 30 %

26. The students in a certain class each joined a club: basketball club, soccer club, or swimming club.
What fraction of the students joined swimming club?
(1) The number of the students who joined the basket club is three times the number of the students
who joined the soccer club.
(2) The number of the students who joined the swimming club is twice the number of the students
who joined soccer club or basketball club.

27. During a certain day, a bookshop sold 100 used books and 50 new books. Is the revenue on new
books sold greater than the revenue on used books sold?
(1) The price for any new book is more than twice the price for the most expensive used book.
(2) The average price for all of the new books is more than twice the average price for all of the used
books.

28. On January 1, 2003, Jack invested $5,000 at r percent interest rate, compounded annually. What is
the value of r?
(1) The investment plus the total interest at the end of 2004 was $550 more than the amount at the end
of 2003.
(2) The investment plus the total interest at the end of 2004 increased by 21% from the original
investment.

29. A 50-liter solution of alcohol and water is 5 percent alcohol. If
2
1
1
liters of alcohol and
2
1
8

liters of water are added to this solution, what percent of the solution produced is alcohol?
(A)
2
1
5
%
(B) 6 %
(C)
3
1
6
%
(D)
3
2
6
%
(E) 7 %

!

Your way to Top Business Schools _____________
181


30. Working at their respective constant rates, printing machine X, Y, and Z can finish a certain work
in 9, 12, and 18 hours. If three machines work together to finish the work, what fraction of the
work will be finished by the machine Z?
(A)
(B)
(C)
(D)
(E)

31. Solution X is 25 percent mercury and solution Y is 10 percent mercury. If 12 ounces of solution X
are added to 13 ounces of solution Y, approximately what percent of the resulting solution will be
mercury?
(A) 12 %
(B) 17 %
(C) 21 %
(D) 23 %
(E) 37 %

32. What is the sum of 5 evenly spaced integers?
(1) The middle integer is zero
(2) Exactly 2 of the integers are negative

Answers
1. D
2. B
3. D
4. E
5. E
6. B
7. B
8. D
9. C
10. B
11. B
12. D
13. B
14. E
15. A
16. E
17. C
18. B
19. C
20. D
21. D
22. C
23. A
24. B
25. C
26. B
27. D
28. D
29. D
30. D
31. B
32. A

9
4
3
1
4
1
9
2
9
1

!

Your way to Top Business Schools _____________
182

Lesson 7
Combinatorics

Factorial.
The factorial symbol ! is used when you need to multiply an integer by all the positive integers
that are less than it.
For example:
S! = S 4 S 2 1
n! = n (n 1) (n 2) 2 1
Note:
u! = 1! = 1. u!
and 1! are the only odd factorials because all other factorials have 2 as a factor.
1. What is value of
8!
5!
?
(A) 336
(B) 40
(C) 120
(D) 360
(E) 1680

Explanation: To calculate
8!
5!
we wont compute 8! and then divide it by 5! That would take too
long. If we write out the problem, we get
87654321
54321
=
8765!
5!
. And 5! Cancels, leaving
us with 8 7 6 = SS6. The answer is (A).

Arrangements of n different objects.
The number of arrangements of n different objects in a raw is a typical problem that can be solve
this way:

1. How many objects we can put at 1-st place? n.
2. How many objects we can put at 2-nd place? n - 1. We can't put the object that already placed
at 1st place.
.....
n. How many objects we can put at n-th place? 1. Only one object remains.
Therefore, the total number of arrangements of n different objects in a raw is
N = n (n 1) (n 2) 2 1 = n!

2. How many ways can you arrange the letters in the word
MICRO?
(A) 5
(B) 1
(C) 120
(D) 20
(E) 60

Explanation: The basic idea is we have 5 objects, and 5
possible positions they can occupy.
We multiply all numbers S 4 S 2 1 = 12u. The
answer is (C).
W
e

h
a
v
e

5

l
e
t
t
e
r
s

t
o

p
i
c
k

f
r
o
m

i
n
i
t
i
a
l
l
y











W
e

v
e

u
s
e
d


u
p

o
n
e

l
e
t
t
e
r
,

s
o

w
e

h
a
v
e

4

t
o

p
i
c
k

f
r
o
m

3

l
e
t
t
e
r
s

r
e
m
a
i
n
i
n
g

2

l
e
t
t
e
r
s

r
e
m
a
i
n
i
n
g

1

l
e
t
t
e
r

r
e
m
a
i
n
i
n
g
.

5 4 3 2 1

!

Your way to Top Business Schools _____________
183

Permutation.
A permutation is an ordered collection of k objects taken from a set of n distinct objects. The
number of ways how we can choose and arrange k objects out of n distinct objects is denoted as:
P
n
h
.
Knowing how to find the number of arrangements of n distinct objects we can easily find
formula for permutations:

1. The total number of arrangements of n distinct objects is n!
2. Now we have to exclude all arrangements of remaining (n-k) objects ((n-k)!) as the order of
remained (n-k) objects doesn't matter.
P
n
h
=
n!
(n h)!


Remember, n is the total number of items, k is the number you want to order and n is always
more or equal to k.

Colors, names, numbers, letters, positions are the keys that indicate permutation. When
item has some identification, like color or letter, an order of it item is important this is the
difference permutation from combination.

3. There are five girls, Amy, Barbara, Candice, Diane, and Elain, playing in the park. If three of
these girls will be chosen to be a princess, queen and sorceress, how many different groups of
girls can be chosen to play?
(A) 120
(B) 90
(C) 60
(D) 40
(E) 10

Explanation: This question is permutation since there are roles of girls their identifications,
that means that order is important. There are three girls being chosen to play in the game, so the
layout begins with three slots for the girls. .
Then we must label the slots that we have laid out. In this case each slot will contain a girl.

For the princess we have 5 options. No girls have been chosen yet and all are still available.

For the queen we now have 4 options. One girl has already been chosen and only four are still
available.


For the sorceress we now have 3 options. Two girls have been chosen and that leaves only three
girls available.

Once all of the slots have been filled with values, the question is considered to be set up.
We now multiply across to find the answer.

!

Your way to Top Business Schools _____________
184

S 4 S = 6u.
Also we could find the answer just using the formula for permutation:
P
5
3
=
5!
(5-3)!
=
5!
2!
=
2!345
2!
= 6u
The answer is (C).

4. How many four-digit numbers can you form using ten numbers (0, 1, 2, 3, 4, 5, 6, 7, 8, 9) if the
numbers can be used only once?
(A) 5040
(B) 3556
(C) 4536
(D) 10000
(E) 24

Explanation: It seems to be easy, we take 10x9x8x7 since we have 4 positions, and get 5040, but
there is a trick to this problem because 5040 will include numbers that start with a 0, and in
reality we don't have those. So, we need to subtract the number of the fake 4-digit numbers. Use
a Permutation formula: P(9,3) = 9x8x7 = 504. 5040-504 = 4536.

Or we can count the number of numbers as we used to do with slots: ___ ___ ___ ___ .
For the thousands digit we can use all digits except for zero: _9 ___ ___ ___.
For the hundreds digit we can use all remaining 9 digits: one digit has already been chosen and
only nine are still available 9 _9_ ___ ___.
For the tens digit there are 8 options. Two digits have been chosen and that leaves only eight
digits available 9 _9_ _8_ ___.
For the units digit there are 7 options _9 _9_ _8_ _7_.
To find the answer we will just multiply 9 9 8 7 = 4SS6. The answer is (C).

Combination.
In the previous examples, when using the permutations, the order of items to be arranged
mattered. If all you want to do is select items, and dont care what order theyre in, you can use
combinations.
A combination is an unordered collection of k objects taken from a set of n distinct objects. The
number of ways how we can choose k objects out of n distinct objects is denoted as: C
n
k


C
n
h
=
n!
h! (n h)!


Pay your attention that we always choose k objects out of n, and n is greater or equal to k.

5. From a group of 8 secretaries, select 3 persons for promotion. How many distinct selections are
there?
(A) 24
(B) 56
(C) 336
(D) 8
(E) 3

Explanat ion: This quest ion is combinat ion since order does not important.
C
8
3
=
8!
3!(8-3)!
=
8!
3!5!
=
5!678
3!5!
=
678
123
= S6

!

Your way to Top Business Schools _____________
185

The answer is (B).

6. A pizza can have 3 toppings out of a possible 7 toppings. How many different pizzas can be
made?
(A) 35
(B) 21
(C) 3
(D) 12
(E) 7

Explanation: There are 7 toppings in total, and by selecting 3, we will make different types of
pizza. This question is a combination since having a different order of toppings will not make a
different pizza.
C
7
3
=
7!
3!(7-3)!
=
7!
3!4!
=
4!567
3!4!
=
567
123
= SS
The answer is (A).

At least / at most
These questions will require you to add all the possible cases together but in these problems it
will take a lot of time. Know and use whenever you see at least or at most the shortcuts:

at leastat must = tutal ases unwanted ases

7. A committee of 3 people is to be formed from a group of 5 men and 5 women. How many
possible committees can be formed if at least one woman is in the committee?
(A) 120
(B) 110
(C) 100
(D) 90
(E) 80

Explanation: If at least one woman is on the committee, that means we can have a committee
with 3 women, 2 women, or 1 woman. And we can find the combinations for each case
separately, then add them all together. But it too long. We can use shortcut instead.
The total possible cases would be the number of committees of any 3 people out of 10: C
10
3
.
The unwanted cases would be the number of committees of 3 men without women: C
5
3
.
C
10
3
C
5
3
=
1u!
S! 7!

S!
2! S!
= 12u 1u = 11u
The second way is easier because it need two calculations instead of five in the first way.
The answer is (B).

8. A three-person committee must be chosen from a group of 7 professors and 10 graduate students.
If at least one of the people on the committee must be a professor, how many different groups of
people could be chosen for the committee?
(A) 70
(B) 560
(C) 630
(D) 1,260
(E) 1,980

!

Your way to Top Business Schools _____________
186

Explanation: In the task we see at least and it is the key to solution: to find a total without a
limit and then subtract the number of situations when there are no professors on the team.
We get C
17
3
=
17!
3!14!
=
151617
122
= 68u total possible committees.
Now, the number of teams with students only is, using the same formula
C
10
3
=
1u!
S! 7!
=
8 9 1u
1 2 2
= 12u
Now, 680-120=560 the number of groups with could be chosen to the committee,
where at least one professor. The answer is (B).

Circular arrangements
Let's say we have 6 distinct objects, how many relatively different arrangements do we have if
those objects should be placed in a circle.



The difference between placement in a row and that in a circle is following: if we shift all object
by one position, we will get different arrangement in a row but the same relative arrangement in
a circle. It means number arrangement in a row n times the number the same relative
arrangement in a circle. So, for the number of circular arrangements of n objects we have:

R =
n!
n
= (n 1)!

Or we can say that it does not matter where the first item is placed, since only the order around
that item is important. And total possible arrangements of remaining n-1 items are (n-1)!
Generally, n different items in a circle can be arranged in (n-1)! ways.

9. Seven family members are seated around their circular dinner table. If only the only
arrangements that are considered distinct are those where family members are seated in different
locations relative to each other, then how many distinct arrangements around the table are
possible?
(A) 7
(B) 42
(C) 5040
(D) 524
(E) 720

Explanation . If all the orderings mattered, the answer would be 7 6 S 4 S 2 1 =
Su4u. But they dont all matter, so clearly the answer is not E. Which ones dont matter? Well,
imagine you have seven people seated around a table, and they all get up and move one seat to
the right. Their relative positions havent changed, so according to the rules of the question, this
shouldnt really count as a new ordering. How many times can we shift people around the table

!

Your way to Top Business Schools _____________
187

before they arrive back at their original seats? We can shift them six times, resulting in seven
identical arrangements, counting the initial arrangement. So you have to divide by 7, it will give
you 720. The answer is (E)

Combinations from multiple selection pools.
When presented with multiple groups of items from which you are required to make a selection,
you will multiply the separate cases together.
10. A committee of 3 boys and 5 girls is to be formed from a group of 5 boys and 7 girls. How many
committees are possible?
(A) 420
(B) 210
(C) 105
(D) 35
(E) 15

Explanation: Out of 5 boys, we must choose 3: C
5
3
= 1u. Out of 7 girls, we must choose 5:
C
7
5
= 21. All together: C
5
3
C
7
5
=10 21 = 21u. The answer is (B).

If question does not ask us how many groups of people that we can form or how many subsets
we can create and the number and type of objects are both fixed and the only concern we have is
how they can be rearranged then we can multiply separate combinations together, as we did in
the previous sample, but there is special formula

(tutal number uI ub|ets)!
(number tn subsets 1)! (number tn subset 2)! (number tn subset 3)!


Whenever you have to rearrange number of different subsets, each of which consist of identical
elements, use this formula.

11. There are three men and four women lining up against a wall from left to right. If the only
concern is gender of each individual, how many different ways can we arrange the people
against the wall by gender?
(A) 8,640
(B) 2,160
(C) 720
(D) 84
(E) 35

Explanation: There are two subsets of men and women. We see difference in gender. But in each
subsets there are uniform people (we dont know their names or positions). And we have just to
rearrange people, so, we can use the formula.
The question tells us that we have three men and four women. So the total number of people we
are rearranging is seven. Substituting into the formula we get:
7!
3!4!
=
4!567
1234!
= SS. The
answer is (E).

Always together.
Frequently, certain item must always be kept together. To do these questions, you must treat the
joined items as they were only one object.

!

Your way to Top Business Schools _____________
188

12. In how many ways can you sit 8 people on a bench if 3 of them must sit together?
(A) 720
(B) 2,160
(C) 2,400
(D) 4,320
(E) 40,320

Explanation: Treat the three that sit together as one person for the time being. Now, you have
only 6 people (5 and the three that act as one) on 6 places: 6! = 72u. Now, you have to
remember that the three that sit together can also change places among themselves: S! = 6. So,
the total number of possibilities is 6! S! = 4S2u. The answer is (D).

13. The choir consists of 5 boys and 6 girls. In how many ways can the singers be arranged in a row,
so that all the boys are together? (Assume the only concern is gender).
(A) 120
(B) 30
(C) 24
(D) 11
(E) 7

Explanation: The five boys as one person. Now there are only 7 people and 6 of them are
uniform. To seat the boy (five boys act as one) on seven pleases there are C
7
1
=
7!
1!6!
= 7 options.
And then 6 remaining girls seat on remaining 6 pleases the only way. The total number of ways
is 7 1 = 7. The answer is (E).

Permutation with repetition.
Sometimes we are interested in arrangements allowing the use of items more than once. Usually
it means that objects are independent. Then to calculate number of arrangements we use formula
n
k
, where k is the number of objects and n is the number of options for each object.

14. There are three lamps in the hall. Each lamp can be switched on and off independently. In how
many ways can the hall be illuminated? (The hall is illuminated when at least one of the lamps in
on)
(A) 9
(B) 8
(C) 7
(D) 6
(E) 5

Explanation: Each switch has two possible positions, on and off. Placing a 2 in each of the 3
positions, we have 2
3
. But among that 8 cases there is one, when all lamps are switched off, and
we must subtract this case because we are interested when the hall is illuminated: 2
3
1 = 8
1 = 7. The answer is (C).

15. 7 different objects must be divided among 3 people. In how many ways can this be done if one
or two of them can get no objects?
(A) 15,1200
(B) 2,187
(C) 3,003

!

Your way to Top Business Schools _____________
189

(D) 792
(E) 344

Explanation: Each of 7 objects can go to 3 different people and by the rule of multiplying we get
33...3 = 3
7
ways. The answer is (B).

Assurance of occurrence.
There are some problems which ask you to find minimum number of attempts to ensure some
outcome. There are no formulas, just count the worst scenario.
16. Of the science books in a certain supply room, 50 are on botany, 65 are on zoology, 90 are on
physics. 50 are on geology, and 110 are on chemistry. If science books are removed randomly
from the supply room, how many must be removed to ensure that 80 of the books removed are
on the same science?
(A) 81
(B) 159
(C) 166
(D) 285
(E) 324

Explanation: Such problems invite us to provide a foolproof solution that would work in 100%
of the cases. Thus, this means we will need to find a solution for the worst case. So after we will
have removed all of the botany and geology books as well as 65 on zoology, 50 on botany, and
50 on geology, but we still don't have 80 of the same kind. So, after another 79 on chemistry and
79 on physics still not enough. Now we will have a total of 50+65+50+79+79=323 books
removed. Now, however, we need to remove only one book because we will know that we have
only two kinds of books left (either chemistry or physics) and any of them will give us a set of
80. Of course in reality it would not be that bad, but we have to take the worst situation to be
sure.


!

Your way to Top Business Schools _____________
190

Probability
An event is defined as any outcome that can occur.

There are four Basic Types of Events
Mutually Exclusive Events: These are events that cannot occur at the same time.
Complementary Events: These are events that have two possible outcomes. The probability of
event A plus the probability of A' equals one. P(A) + P(A') = 1.
Independent Events: These are two or more events for which the outcome of one does not
affect the other. They are events that are not dependent on what occurred previously.
Conditional Events: These are events that are dependent on what occurred previously.

Probability is defined as the chance that an event will happen. The definition of probability is

prubabtltty =
number uI Iavurable events
number uI tutal events


The favorable events are the events of interest. They are the events that the question is
addressing. The total events are all possible events that can occur relevant to the question asked.

Example. Coin



There are two equally possible outcomes when we toss a coin: a head (H) or tail (T). Therefore,
the probability of getting head is 50% or
1
2
and the probability of getting tail is 50% or
1
2

All possibilities: {H,T}

Example. Dice



There are 6 equally possible outcomes when we roll a die. The probability of getting any number
out of 1-6 is
1
6
. All possibilities: {1,2,3,4,5,6}

!

Your way to Top Business Schools _____________
191

Example. Marbles, Balls, Cards...



Let's assume we have a jar with 10 green and 90 white marbles. If we randomly choose a marble,
what is the probability of getting a green marble?
The number of all marbles: N = 10 + 90 =100
The number of green marbles: n = 10
Probability of getting a green marble: p =
n
N
=
10
100
=
1
10
.

1.There is a group of six people; among them are Mr X and Mr Y. Three persons are chosen at
random without repetitions. Find the probability of choosing Mr X.
(A)
1
2

(B)
1
6

(C)
1
3

(D)
5
6

(E)
Explanation: If 3 persons are chosen, then Mr X has 3 chances to be chosen among them. So,
there are three favourable events. And there are 6 total events, because there are six people, and
Mr has three chances to be chosen and three chances not to be chosen. Therefore the probability
of choosing Mr X is
numbcr oI IavorabIc cvcnts
numbcr oI totaI cvcnts
=
3
6
=
1
2
. The answer is (A).

Multiplication Rule
Two events are independent if occurrence of one event does not influence occurrence of other
events.
If two (or more) independent events are occurring, and you know the probability of each, the
probability of BOTH (or ALL) of them occurring together (event A and event B and event C etc)
is a multiplication of their probabilities.

Piobability of A anu B = p(A anu B) = p(A) p(B)
Piobability of A anu B anu C . . . anu Z = p(A anu B anu C . . . anu Z) = p(A) p(B) p(C) . . .

The multiplication theorem is used to answer the following questions:
- What is the probability of two or more events occurring either simultaneously or in succession?
- For two events A and B: What is the probability of event A and event B occurring?

The word "and" is the key word that indicates multiplication of the individual
probabilities.
For mutually exclusive events: Probability (A and B) = P(A) and P(B) = 0

!

Your way to Top Business Schools _____________
192

For independent events: Probability (A and B) = P(A) and P(B) = P(A) X P(B)

2.If there is a 20% chance of rain, what is the probability that it will rain on the first day but not on
the second?
(A) 0.2
(B) 0.8
(C) 0.16
(D) 0.04
(E) 0.4

Explanation: The probability of rain is 0.2; therefore probability of sunshine is q = 1 - 0.2 = 0.8.
This yields that the probability of rain on the first day and sunshine on the second day is:
P = 0.2 * 0.8 = 0.16. Best answer is (C).

3.There are two sets of integers: {1,3,6,7,8} and {3,5,2}. If Robert chooses randomly one integer
from the first set and one integer from the second set, what is the probability of getting two odd
integers?
(A)
2
5

(B)
3
5

(C)
2
3

(D)
5
8

(E)
1
5


Explanation: There is a total of 5 integers in the first set and 3 of them are odd: {1, 3, 7}.
Therefore, the probability of getting odd integer out of first set is
3
5
. There are 3 integers in the
second set and 2 of them are odd: {3, 5}. Therefore, the probability of getting an odd integer out
of second set is
2
3
. Finally, the probability of getting two odd integers is: P =
3
5

2
3
=
2
5
. The
answer is (A).

The conditional probability of an event B, in relation to event A, is the probability that
event B will occur given the knowledge that an event A has already occurred.

4.There is a group of six people; among them are Mr X and Mr Y. Three persons are chosen at
random without repetitions. Find the probability of choosing both Mr X and Mr Y.
(A)
2
6

(B)
2
3

(C)
2
5

(D)
3
5

(E)
1
5


Explanation:Choosing Mr. X and Mr. Y is conditional events. Probability of choosing Mr. X is
: there are 3 successful outcomes for Mr. X among 6 possible.

!

Your way to Top Business Schools _____________
193

And now there are 5 people and only 2 vacancies in the group, because Mr. X is already in the
group. It means that for Mr. Y there are only 2 successful outcomes out of 5 possibilities.
Probability of choosing Mr. Y is 2/5.
And when we see and we multiply probabilities of each event: probability of choosing both Mr
X and Mr Y= P(X anu Y) = P(X) P(Y) =
1
2

2
5
=
1
5
. The answer is (E).

5.Given that there are 5 married couples. If we select only 3 people out of the 10, what is the
probability that none of them are married to each other?
(A)
7
9

(B)
3
10

(C)
3
5

(D)
2
3

(E)
1
3


Explanation:1-st person:
10
10
= 1 - we choose any person out of 10.
2-nd person:
8
9
- we choose any person out of 8=10-2(one couple from previous choice)
3-rd person:
6
8
- we choose any person out of 6=10-4(two couples from previous choices).
p = 1
8
9

6
8
=
2
3
. The answer is (D).

Addition rule
Two events are mutually exclusive if they cannot occur at the same time. For n mutually
exclusive events the probability is the sum of all probabilities of events, the probability of
EITHER of them occurring (event A or event B or event C etc) is a sum of their probabilities.

Piobability of A oi B = p(A oi B) = p(A) + p(B)
Piobability of A oi B oi C . . . oi Z = p(A oi B oi C . . . oi Z) = p(A) + p(B) + . . . + p(Z)

Incompatible, mutually exclusive events, means that they can't happen together, i.e.
p(A anu B) = u.

The addition theorem is used to answer the following questions:
- What is the probability of one event or another event or both events occurring?
- What is the probability of event A or event B occurring?

The word "or" indicates addition of the individual probabilities. The answers to the above
questions are different depending on whether the events are mutually exclusive or independent.

6.If Jessica rolls a die, what is the probability of getting at least a "3"?
(A)
2
3

(B)
1
2

(C)
1
6

(D)
1
3

(E)
5
6

!

Your way to Top Business Schools _____________
194


Explanation: There are 4 outcomes that satisfy our condition (at least 3): {3, 4, 5, 6}. The
probability of each outcome is 1/6. The probability of getting at least a "3" is:
P =
1
6
+
1
6
+
1
6
+
1
6
=
2
3
. The is answer is (A).

7.What is the probability that a fair die rolled once will land on either 4 or 5?
(A)
1
6

(B)
2
6

(C)
3
6

(D)
4
6

(E)
5
6


Explanation: This is an or problem, because it is asking for the probability that the die will
land on either 4 or 5. The probability that the die will land on 4 is 1/6. The probability that the
die land on 5 is 1/6. Therefore, the probability that the die will land on either 4 or 5 is
1
6
+
1
6
=
1
3
.
The answer is (B).

In case of two compatible events, the OR tool looks a bit more complicated:

p(A oi B) = p(A) + p(B) p(A anu B)

If we know that A and B are independent, we can apply AND rule to rewrite:

Piobability (A oi B) = P(A) oi P(B) = P(A) + P(B) P(A ) P( B)

8.A new Machine is bought by a factory to produce 2 parts, A and B. A and B when assembled
produce C. During the testing stages of the machine, 1 out of 10 in A is defective and 2 out of 10
in B are defective. What is the probability of C being defective?
(A) 0.02
(B) 0.2
(C) 0.28
(D) 0.32
(E) 0.72

Explanation: P (A is defective) = 1/10, P(B is defective) = 2/10 = 1/5. C is defective if A is
defective or if B is defective or if both of them are defective. P(C is defective) = P (A or B is
defective) =P(A) + P(B) P(A B).
P(A) + P(B) P(A) P(B) = u.1 + u.2 u.u2 = u.28.
Therefore, the best answer is (C).

Probability that a specific event will not occur.
The maximum probability for any event is 1. A probability of 1 means that the event is certain to
occur.
If you know that the probability of an event (or one of the outcomes) is p, the probability of this
event NOT happening (or the probability of it NOT having this given outcome) is (1 p).

p(nut A) + p(A) = 1

!

Your way to Top Business Schools _____________
195


When you see the words "at least" in a probability question, chances are it is easier to find the
probability of the opposite happening and then subtract from 1. The word "at least" indicates
problems to solve which one use the formula

P(A) = 1 P(not A).

9.What is the probability that, on three rolls of a single fair die, at least one of the rolls will be a
six?
(A)
125
216

(B)
91
216

(C)
191
216

(D)
1
6

(E)
5
6

(F)
1
2


Explanation: We could list all the possible outcomes of three rolls of a die (1-1-1, 1-1-2, 1-1-3,
etc.) and then determine how many of them have at least one six, but this would be very time-
consuming. Instead, it is easier to think of this problem in reverse before solving:
What is the probability that not one of the rolls will yield a 6? On each roll, there is a 5/6
probability that the die will not yield a 6.
Thus, the probability that on all 3 rolls the die will not yield a 6 is
5
6

5
6

5
6
=
125
216
. Now we
return to answer the question by subtracting our result from 1. The probability that at least one of
the rolls will be a 6 includes every outcome, except when three consecutive non-sixes are rolled.
1
125
216
=
91
216
is the probability that at least one six will be rolled. The answer is (B).

10. There are 3 black balls and 7 white balls in a box. If two balls are chosen randomly, what
is the probability that at least one will be black?
(A)1/15
(B) 7/15
(C) 8/15
(D) 7/30
(E)23/30

Explanation: We see the words "at least" in a probability question, so it is easier to find the
probability of the opposite happening and then subtract from 1. The opposite of at least one ball
being black is both balls being white. The probability of drawing two white balls is 71u
69 = 71S. Don't forget to subtract that from 1: 1
7
15
=
8
15
. So the correct answer is (C).

Probability involving combinations.
There are a few typical ways that you can use for solving probability questions. Let's consider
examples, how it is possible to apply different approaches:

11. There are 8 employees including Bob and Rachel. If 2 employees are to be randomly
chosen to form a committee, what is the probability that the committee includes both Bob and

!

Your way to Top Business Schools _____________
196

Rachel?
(A)
1
14

(B)
1
28

(C)
2
8

(D)
1
56

(E)
1
64


Explanation:
1. Combinatorial approach: The total number of possible committees is N = C
2
8
= 28. The number
of possible committee that includes both Bob and Rachel is n = 1.
P =
n
N
=
1
C
2
8
=
1
28


2. Probability approach: The probability of choosing Bob or Rachel as a first person in committee
is 2/8. The probability of choosing Rachel or Bob as a second person when first person is already
chosen is 1/7. The probability that the committee includes both Bob and Rachel is.
P =
2
8

1
7
=
2
S6
=
1
28


12. The probability that a visitor at the mall buys a pack of candy is 30%. If three visitors
come to the mall today, what is the probability that exactly two will buy a pack of candy?
(A) 0.343
(B) 0.027
(C) 0.063
(D) 0.189
(E) 0.147

Explanation: Without the combination you find the probability of a situation when the first
customer doesn't buy, the second and the third do: P = u.S u.S u.7. Let's denote it BNN.
But there are also two other combinations that match: NBN and NNB.
P(BNN oi NBN oi NNB ) = u.S u.S u.7 S = u.189. The answer Is (D).





!

Your way to Top Business Schools _____________
197

Home assignment.
Combinatorics
1. There are 5 Rock songs, 6 Pop songs and 3 jazz. How many different albums can be formed
using the above repertoire if the albums should contain at least 1 Rock song and 1 Pop song?
(A) 15624
(B) 16384
() 6144
(D) 384
(E) 240

2. How many even numbers of 3 digits can be formed with the digits 0, 1, 2, 3, 4, 5, and 6?
(A) 168
(B) 162
(C) 120
(D) 105
(E) 100

3. 7 different objects must be divided among 3 people. In how many ways can this be done if
one or two of them can get no objects?
(A) 15120
(B) 2187
(C) 3003
(D) 792
(E) 344

4. There are 5 married couples and a group of three is to be formed out of them; how many
arrangements are there if a husband and wife may not be in the same group?
(A) 680
(B) 480
(C) 80
(D) 70
(E) 60

5. What is the total number of ways, in which 3 red and 3 blue beads can be put in a circular
bracelet?

!

Your way to Top Business Schools _____________
198

(A) 4
(B) 5
(C) 6
(D) 120
(E) 60

6. There are 4 contestants in the competition for 6 different prizes. A contestant can win only
one prize. How many different outcomes are possible at the competition for the four
contestants?
(A) 360
(B) 720
(C) 4
(D) 6
(E) 1

7. How many numbers between 0 and 1670 have a prime tens digit and a prime units digit?
(A) 268
(B) 272
(C) 202
(D) 112
(E) 262

8. How many different words with up to 4 letters can be made with the letters A and B?
(A) 2
(B) 4
(C) 16
(D) 30
(E) 32

9. How many different signals can be made by hosting up to 6 different coloured flags one
above the other, when any number of them can be hoisted at once?
(A) 6!
(B) 6!+6!/5!+6!/4!+6!/3!+6!/2!+6!
(C) 6+5+4+3+2+1

!

Your way to Top Business Schools _____________
199

(D) C (6, 6)
(E) P (6, 6)

10. Alan has a flock of sheep from which he will choose 4 to take with him to the livestock show
in Houston. If Alan has 15 distinct possible groups of sheep he could take to the show then
which of the following is the number of sheep in his flock?
(A) 6
(B) 7
(C) 15
(D) 30
(E) 5

11. How many 4 digit numbers begin with a digit that is prime and end with a digit that is prime?
(A) 1600
(B) 800
(C) 80
(D) 1440
(E) 16

12. John has 12 clients and he wants to use color coding to identify each client. If either a single
color or a pair of two different colors can represent a client code, what is the minimum number
of colors needed for the coding? (the order of a pair of colors does not matter)
(A) 24
(B) 12
(C) 7
(D) 5
(E) 4

13. 4 women and 6 men work in the accounting department. In how many ways can a committee
of 3 be formed if it has to include at least one woman?
(A) 36
(B) 60
(C) 72
(D) 100
(E) 110

14. There are 3 yellow chips and 2 green chips. When arranged in a row, they form a certain
color pattern, for example RBRRB. How many color patterns are possible?
(A) 10
(B) 12
(C) 24
(D) 60
(E) 100

15. There are three lamps in the hall. Each lamp can be switched on and off independently. In
how many ways can the hall be illuminated? (The hall is illuminated when at least one of the
lamps in on)
(A) 9
(B) 8

!

Your way to Top Business Schools _____________
200

(C) 7
(D) 6
(E) 5

16. (SC) A 6-people committee in a film festival will be selected from 8 directory and 6 film
critics. If the committee consists of 4 directors and 2 critics, how many such committees are
possible?
(A) 2,100
(B) 1,400
(C) 1,050
(D) 840
(E) 700

17. (SC) In how many arrangements can a photographer seat 3 girls and 3 boys in a row of 6
seats if the boys are to have the first, third, and fifth seats?
(A) 6
(B) 9
(C) 12
(D) 36
(E) 720

18. Of the science books in a certain supply room, 50 are on botany, 65 are on zoology, 90 are on
physics. 50 are on geology, and 110 are on chemistry. If science books are removed randomly
from the supply room, how many must be removed to ensure that 80 of the books removed are
on the same science?
(A) 81
(B) 159
(C) 166
(D) 285
(E) 324

19. The president of a country and 4 other dignitaries are scheduled to sit in a row
on the 5 chairs represented above. If the president must sit in the center chair, how
many different seat ing arrangements are possible for the 5 people?
(A) 4
(B) 5
(C) 20
(D) 24
(E) 120

20. If a customer makes exactly 1 selection from each of the 5 categories listed below, what is
the greatest number of different ice cream sundaes that a customer can create?
12 ice cream flavors;
10 kinds of candy;

!

Your way to Top Business Schools _____________
201

8 liquid toppings;
5 kinds of nuts;
With or without whip cream;
(A)9600
(B)4800
(C)2400
(D)800
(E)400

21. Given a selected committee of 8, in how many ways, can the members of the committee
divide the responsibilities of a president, vice president, and secretary?
(A) 72
(B) 672
(C) 336
(D) 56
(E) 112

22. How many four-digit numbers can you form using ten numbers (0, 1, 2, 3, 4, 5, 6, 7, 8, 9) if
the numbers can be used only once?
(A) 9000
(B) 5852
(C) 4940
(D) 4536
(E) 10000

23. A three-person committee must be chosen from a group of 7 professors and 10 graduate
students. If at least one of the people on the committee must be a professor, how many different
groups of people could be chosen for the committee?
(A) 70
(B) 560
(C)630
(D) 1,260
(E) 1,980

24. There are 11 top managers that need to form a decision group. How many ways are there to
form a group of 5 if the President and Vice President are not to serve on the same team?
(A) 232
(B) 84
(C) 378
(D) 162
(E)196

!

Your way to Top Business Schools _____________
202

25. There are 9 books on a shelf, 7 hard cover and 2 soft cover. How many different
combinations exist in which you choose 4 books from the 9 and have at least one of them be a
soft cover book?
(A) 184
(B) 126
(C) 91
(D) 36
(E) 21

26. There are 5 married couples and a group of three is to be formed out of them; how many
arrangements are there if a husband and wife may not be in the same group?
(A) 60
(B) 120
(C) 40
(D) 80
(E) 20

27. How many different signals can be transmitted by hoisting 3 red, 4 yellow and 2 blue flags
on a pole, assuming that in transmitting a signal all nine flags are to be used?
(A)2520
(B) 1260
(C)4!3!2!
(D) 9!
(E) 24

28. From a group of 8 secretaries, select 3 persons for promotion. How many distinct selections
are there?
(A) 366
(B) 3
(C) 8
(D)112
(E) 56

29. On how many ways can the letters of the word "COMPUTER" be arranged?
1) Without any restrictions.
2) M must always occur at the third place.
3) All the vowels are together.
4) All the vowels are never together.
5) Vowels occupy the even positions.

30. In how many ways can one choose 6 cards from a normal deck of cards so as to have all suits
present?
(A) (13
4
) x 24 x 47

!

Your way to Top Business Schools _____________
203

(B) (13
4
) x 27 x 47
(C) C
48


(D) 13
4

(E) (13
4
) x C
48



31. Each of the integers from 0 to 9, inclusive, is written on a separate slip of blank paper and the
ten slips are dropped into a hat. If the slips are then drawn one at a time without replacement,
how many must be drawn to ensure that the numbers on two of the slips drawn will have a sum
of 10?
(A) 3
(B) 4
(C) 5
(D)6
(E) 7

32. Seven men and seven women have to sit around a circular table so that no 2 women are
together. In how many different ways can this be done?
(A) 6!7!
(B) 7!7!
(C) 6!6!
(D) 14!
(E) 7!2

33. If Bob and Jen are two of 5 participants in a race, how many different ways can the race
finish where Jen always finishes in front of Bob?
(A) 120
(B) 60
(C) 30
(D) 15
(E) 5

34. How many number of odd 3 digit numbers can be formed with the digits 0,1,2,3,4,5 if no
digit is repeated in any number?
(A) 48
(B) 72
(C) 216
(D) 192
(E) 243




!

Your way to Top Business Schools _____________
204


Probability

1. (OG) If x is to be chosen at random from the set {1, 2, 3, 4} and y is to be chosen at random
from the set {5, 6, 7}, what is the probability that x y will be even?
(A) 1/6
(B) 1/3
(C)
(D) 2/3
(E) 5/6

2. Out of a box that contains 4 black and 6 white mice, three are randomly chosen. What is the
probability that all three will be black?
(A) 8/125
(B) 1/30
(C) 2/5
(D) 1/720
(E) 3/10

3. (GC) A new Machine is bought by a factory to produce 2 parts, A and B. A and B when
assembled produce C. During the testing stages of the machine, 1 out of 10 in A is defective and
2 out of 10 in B are defective. What is the probability of C being defective?
(A) 0.02
(B) 0.20
(C) 0.28
(D) 0.32
(E) 0.72

4. If a bottle is to be selected at random from a certain collection of bottles, what is the probability
that the bottle will be defective?
(1) The ratio of the number of bottles in the collection that are defective to the number that are
not defective is 3:500.
(2) The collection contains 3,521 bottles.

5. (OG) A committee is composed of w women and m men. If 3 women and 2 men are added to the
committee, and if one person is selected at random from the enlarged committee, then the
probability that a women is selected can be represented by
(A) w/m
(B) w/(w + m)
(C) (w + 3)/(m + 2)
(D) (w + 3)/(w+ m + 3)
(E) (w + 3)/(w+ m + 5)

6. Of the 10 employees in a department, 4 have a master's degree. If an action group with three
members is to be selected, at random, what is the probability that the group will include at least
one employee who has a master's degree?

!

Your way to Top Business Schools _____________
205

(A)
5
2

(B)
5
3

(C)
3
1

(D)
6
1

(E)
6
5


7. A box contains 100 balls, numbered from 1 to 100. If three balls are selected at random and with
replacement from the box, what is the probability that the sum of the three numbers on the balls
selected from the box will be odd?
(A)
4
1

(B)
8
3

(C)
2
1

(D)
8
5

(E)
4
3


8. Each of the 350 students either is graduate or postgraduate, what is the probability that one
student selected at random is a female postgraduate?
(1) The number of postgraduates is 187.
(2) The number of the female students is 247.

9. When a coin is tossed for 6 times, what is the probability that after the first tossing, every
outcome will be different from the previous one?
(A)
16
1

(B)
24
1

(C)
32
1

(D)
48
1

(E)
64
1


10. There are 8 books lying on a table: 4 are hardback book and the other 4 are paperback books. If 3
books are to be selected at random from the 8 books, what is the probability that at least one of
the paperback books will be selected?
(A)
2
1

!

Your way to Top Business Schools _____________
206

(B)
3
2

(C)
12
11

(D)
14
13

(E)
35
32


11. There are 20 books in a bookcase. If one book is selected at random, what is the probability that
the book is either a hardback or a novel?
(1) 8 books in the bookcase are novel books and 10 books are hardbacks.
(2) 3 books in the bookcase are hardback novels.

12. A jar contains only x black balls and y white balls. One ball is drawn randomly from the jar and
is not replaced. A second ball is then drawn randomly from the jar. What is the probability that
the first ball drawn is black and the second ball drawn is white?
(A)
|
|
.
|

\
|
+
|
|
.
|

\
|
+ y x
y
y x
x

(B)
|
|
.
|

\
|
+

|
|
.
|

\
|
+ 1
1
y x
x
y x
x

(C)
y x
xy
+

(D)
|
|
.
|

\
|
+

|
|
.
|

\
|
+

y x
y
y x
x 1 1

(E)
|
|
.
|

\
|
+
|
|
.
|

\
|
+ 1 y x
y
y x
x


13. Xavier, Yvonne, and Zelda each try independently to solve a problem. If their individual
probabilities for success are
4
1
,
2
1
, and
8
5
, respectively, what is the probability that Xavier and
Yvonne, but not Zelda, will solve the problem ?
(A)
8
11

(B)
8
7

(C)
64
9

(D)
64
5

(E)
64
3


14. Sixty percent of the members of a study group are women, and 45 percent of those women are
lawyers. If one member of the study group is to be selected at random, what is the probability
that the member selected is a woman lawyer?

!

Your way to Top Business Schools _____________
207

(A) 0.10
(B) 0.15
(C) 0.27
(D) 0.33
(E) 0.45

15. There is a total of 120 marbles in a box, each of which is red, green, blue, or white. If one
marble is drawn from the box at random, the probability that it will be white is
4
1
and the
probability that it will be green is
3
1
. What is the probability that the marble will be either red or
blue ?
(A)
6
1

(B)
4
1

(C)
7
2

(D)
3
1

(E)
12
5


16. Each of the eggs in a bowl is dyed red, or green, or blue. If one egg is to be removed at random.
what is the probability that the egg will be green?
(1) There are 5 red eggs in the bowl.
(2) The probability that the egg will be blue is
3
1


17. A machine is made up of two components, A and B. Each component either works or fails. The
failure or nonfailure of one component is independent of the failure or nonfailure of the other
component. The machine works if at least one of the components works. If the probability that
each component works is
3
2
, what is the probability that the machine works?
(A)
9
1

(B)
9
4

(C)
2
1

(D)
3
2

(E)
9
8

18. There are 18 balls in a jar. You take out 3 blue balls without putting them back inside, and now
the probability of pulling out a blue ball is 1/5. How many blue balls were there in the beginning?

!

Your way to Top Business Schools _____________
208

(A) 9
(B) 8
(C) 7
(D) 12
(E) 6

19. The probability of Sam passing the exam is 1/4. The probability of Sam passing the exam and
Michael passing the driving test is 1/6. What is the probability of Michael passing his driving test?
(A) 1/24.
(B) 1/2.
(C) 1/3.
(D) 2/3.
(E) 2/5

20. In a jar there are 3 red balls and 2 blue balls. What is the probability of drawing at least one red
ball when drawing two consecutive balls randomly?
(A) 9/10
(B) 16/20
(C) 2/5
(D) 3/5
(E)

21. Chef Gundy is creating a new dessert that will be made from 3 ingredients. If he has 8 cookies
and one flavor of sorbet to choose from, what fraction of the possible arrangements will contain
the sorbet?
(A) 2/3
(B) 1/3
(C) 1/56
(D) 1/9
(E) 1/84

22. The probability that the typist will make a mistake on a page is 1/100. What is the probability
that the typist will type 100 pages without making a single mistake?
(A)1
(B) 1/10
(C) 99/100
(D) (99/100)
100

(E) (1/100)
100


23. The probability that a visitor at the mall buys a pack of candy is 30%. If three visitors come to
the mall today, what is the probability that exactly two will buy a pack of candy?
(A) 0.343
(B) 0.027
(C) 0.063
(D) 0.189
(E) 0.147

!

Your way to Top Business Schools _____________
209


24. If four whole numbers taken at random are multiplied together, what is the probability that the
last digit in the product is 1, 3, 7 or 9?
(A) 0.4
(B) u.4
4

(C) u.S
(D) u.S
4

(E) u.6

25. If n is an integer from 1 to 96, what is the probability for n (n + 1) (n +2) being divisible
by 8?
(A) 25%
(B) 50%
(C) 62.5%
(D) 72.5%
(E) 75%

26. If the probability of rain on any given day in city x is 50% what is the probability it with rain on
exactly 3 days in a five day period?
(A) 8/125
(B) 2/25
(C) 5/16
(D) 8/25
(E) 3/4

!

Your way to Top Business Schools _____________
210

Test 7 Combinatorics, probability
1. If w < x < y < z, is the product of four numbers negative?
(1) z > 0.
(2) wx < 0.

2. If m and n are positive integers, and 80 = 22m + n, what is the value of n?
(1) n < 22.
(2) m = 3.

3. What is the remainder when positive integer n is divided by 3?
(1) When n l is divided by 3, the remainder is 2.
(2) When n + l is divided by 3, the remainder is 1.

4. If m and n are positive integers, what is the value of mn?
(1)
(2) The largest common factor of m and n is 3.

5. What is the value of m
2
n
2
?
(1) m + n = 4.
(2) (m n)
2
= 16.

6. If x and y are none zero, is ?
(1) x > 0.
(2) y > 0.

7. There are 33 students in a certain class, 18 are girls. If 23 of the students joined a math club,
how many boys did not joined the club?
(1) 11 girls joined the club.
(2) There are fewer girls than boys in the club.

8. Last year in sales of single-home houses, 60% of already-existing houses was sold while 76%
of new-established houses sold. If there are three times as many new-established houses as
already-existing houses, what percent of single-home houses has been sold in last year?

(A) 60 %
(B) 64 %
(C) 72 %
(D) 78 %
(E) 80 %

9. In a certain company, if 75% of the employees usually use a laptop, what percent of the
employees usually use a PDA (Personal Digital Assistant)?
(1) 60% of the employees who usually use a laptop also use a PDA.
(2) 90% of the employees who usually use a PDA also use a laptop.
15
7
=
n
m
0
3 2
5 3
>
y x
y x

!

Your way to Top Business Schools _____________
211


10. During a certain trading day, the opening price for stock X is $40 and the opening price
for stock Y is $25; the closing price for Stock X is $41, and the closing price for stock Y is
$28.5. The percentage increase of stock Y is how much greater than the percentage increase
of stock X?
(A) 115
(B) 11.5
(C) 12
(D) 12.5
(E) 125

11. If the units digit of integer x is greater than 1, what is the units digit?
(1) The units digit of x
3
and the units digit of x are the same.
(2) The units digit of x
2
is 6.

12. If point P is inside a circle with center at origin and radius of 2, is point inside the same
circle?
(1) OP = 1.
(2) PK = 3.5.

13. As the figure above shows, four identical circles are inscribed in a square. If the distance
between centers of circle A and is , what is the area of the shaded region?
(A) 256 722
(B) 256 642
(C) 256 322
(D) 128 322
(E) 64 162

14. Each of the 350 students either is graduate or postgraduate, what is the probability that
one student selected at random is a female postgraduate?
(1) The number of postgraduates is 187.
(2) The number of the female students is 247.

15. An analyst will recommend a combination of 3 industrial stocks, 2 transportation stocks, and
2 utility stocks. If the analyst can choose from 5 industrial stocks, 4 transportation stocks, and
3 utility stocks, how many different combinations of 7 stocks are possible?
(A) 12
(B) 19
(C) 60
(D) 180
2 4
A
B

!

Your way to Top Business Schools _____________
212

(E) 720

16. A club with a total membership of 30 has formed 3 committees, M, S, and R, which have 8, 12,
and 5 members, respectively. If no member of committee M is on either of the other 2
committees, what is the greatest possible number of members in the club who are on none of the
committees?
(A) 5
(B) 7
(C) 8
(D) 10
(E) 12

17. There are two kinds of books on the bookshelf: Spanish and English. If two books are randomly
chosen from the bookshelf without replacement, what is the possibility that at least one of the
books chosen is English book?
(1) The ratio of Spanish books to English book is 3:1.
(2) There are less than 20 books on the shelf.


18. When a coin is tossed for 6 times, what is the probability that after the first tossing, every
outcome will be different from the previous one?
(A)
(B)
(C)
(D)
(E)

19. There are 4 red marbles and 2 blue marbles in jar A, and there are 3 red marbles and 2 blue
marbles in jar B. If one marble is to be selected at random from each jar, what is the probability
that exactly one red marble and one blue marble will be selected?

(A)
(B)
(C)
(D)
(E)

16
1
24
1
32
1
48
1
64
1
5
1
15
4
3
1
15
7
11
7

!

Your way to Top Business Schools _____________
213

20. In how many arrangements can a photographer seat 3 girls and 3 boys in a row of 6 seats if the
boys are to have the first, third, and fifth seats?
(A) 6
(B) 9
(C) 12
(D) 36
(E) 720

21. A machine is made up of two components, A and B. Each component either works or fails. The
failure or nonfailure of one component is independent of the failure or nonfailure of the other
component. The machine works if at least one of the components works. If the probability that
each component works is
3
2
, what is the probability that the machine works?
(A)
9
1

(B)
9
4

(C)
2
1

(D)
3
2

(E)
9
8


22. Twelve jurors must be picked from a pool of n potential jurors. If m of the potential jurors are
rejected by the defense counsel and the prosecuting attorney, how many different possible juries
could be picked from the remaining potential jurors?
(1) If one less potential juror had been rejected, it would be possible to create 13 different juries.
(2) n = m + 12
23. What is the value of x?
(1)
9
4
= x

(2) The average (arithmetic mean) of x
2
, 6x, and 3 is 2.


24. The sum of the fourth and twelfth term of an arithmetic progression is 20. What is the sum of the
first 15 terms of the arithmetic progression?
(A) 300
(B) 120
(C) 150
(D) 170
(E) 270


!

Your way to Top Business Schools _____________
214

Answers:
1. B
2. D
3. D
4. C
5. E
6. A
7. A
8. C
9. C
10. B
11. E
12. C
13. E
14. E
15. D
16. D
17. E
18. C
19. D
20. D
21. E
22. D
23. B
24. C

!

Your way to Top Business Schools _____________
215





Practice tests

!

Your way to Top Business Schools _____________
216

Practice test 1
1. (Easy) Driving 1.5 times slower, Bill was late for school today. What is the usual time it
takes Bill to drive to school? (Assume that each day Bill takes the same route).
(1) It took Bill 15 more minutes to drive to school today than usually
(2) The distance between home and school is 15 miles
(A) Statement (1) ALONE is sufficient, but Statement (2) alone is not sufficient.
(B) Statement (2) ALONE is sufficient, but Statement (1) alone is not sufficient.
(C) BOTH statements TOGETHER are sufficient, but NEITHER statement ALONE is
sufficient.
(D) EACH statement ALONE is sufficient.
(E) Statements (1) and (2) TOGETHER are NOT sufficient.

2. (Medium) Is integer N even?
(1) N N = N
(2) N = N
3

(A) Statement (1) ALONE is sufficient, but Statement (2) alone is not sufficient.
(B) Statement (2) ALONE is sufficient, but Statement (1) alone is not sufficient.
(C) BOTH statements TOGETHER are sufficient, but NEITHER statement ALONE is
sufficient.
(D) EACH statement ALONE is sufficient.
(E) Statements (1) and (2) TOGETHER are NOT sufficient.

3. (Medium) m and n are two consecutive positive integers. Is m > n?
(1) m - 1 and n + 1 are consecutive positive integers
(2) m is an even integer
(A) Statement (1) ALONE is sufficient, but Statement (2) alone is not sufficient.
(B) Statement (2) ALONE is sufficient, but Statement (1) alone is not sufficient.
(C) BOTH statements TOGETHER are sufficient, but NEITHER statement ALONE is
sufficient.
(D) EACH statement ALONE is sufficient.

!

Your way to Top Business Schools _____________
217

(E) Statements (1) and (2) TOGETHER are NOT sufficient.

4. (Medium) What percent of the MIS students enrolled at Wisconsin University are female
students?
(1) Of the female students in Wisconsin University, 5 percent are studying MIS.
(2) Of the female students in Wisconsin University, 12 percent are studying MIS.
(A) Statement (1) ALONE is sufficient, but Statement (2) alone is not sufficient.
(B) Statement (2) ALONE is sufficient, but Statement (1) alone is not sufficient.
(C) BOTH statements TOGETHER are sufficient, but NEITHER statement ALONE is
sufficient.
(D) EACH statement ALONE is sufficient.
(E) Statements (1) and (2) TOGETHER are NOT sufficient.

5. (Medium) Robert can eat a 20 kg. bag of potatoes in 10 days. He and his wife Jane can
eat the same bag in 7 days. How many days does it take Jane to eat a 20 kg. bag of
potatoes by herself?
(A) 46 + 2/3
(B) 23 + 1/3
(C) 17
(D) 12
(E) 11 + 2/3

6. (Medium) Is integer R even?
(1) R
3
+ R
2
is even
(2) 3R is divisible by 6
(A) Statement (1) ALONE is sufficient, but Statement (2) alone is not sufficient.
(B) Statement (2) ALONE is sufficient, but Statement (1) alone is not sufficient.
(C) BOTH statements TOGETHER are sufficient, but NEITHER statement ALONE is
sufficient.
(D) EACH statement ALONE is sufficient.

!

Your way to Top Business Schools _____________
218

(E) Statements (1) and (2) TOGETHER are NOT sufficient.

7. (Hard) If quadrilateral ABCD is inscribed into a circumference. What is the value of
angle A?
(1) AC = CD
(2) Measure of angle D is 70 degrees
(A) Statement (1) ALONE is sufficient, but Statement (2) alone is not sufficient.
(B) Statement (2) ALONE is sufficient, but Statement (1) alone is not sufficient.
(C) BOTH statements TOGETHER are sufficient, but NEITHER statement ALONE is
sufficient.
(D) EACH statement ALONE is sufficient.
(E) Statements (1) and (2) TOGETHER are NOT sufficient.

8. (Hard) A 10 liter mixture of cranberry juice and water contains juice and water in the
ratio of 3:2. 5 liters of the mixture are removed and replaced with pure juice and the
operation is repeated once more. At the end of the two removals and replacements, what
is the ratio of juice to water in the resulting mixture?
(A) 5:3
(B) 6:4
(C) 8:2
(D) 17:3
(E) 9:1

9. (Hard) The probability that a visitor at the mall buys a pack of candy is 30%. If three
visitors come to the mall today, what is the probability that exactly two will buy a pack of
candy?
(A) 0.343
(B) 0.147
(C) 0.189
(D) 0.063
(E) 0.027

!

Your way to Top Business Schools _____________
219

10. (Medium) What is the value of X?
(1) X! is odd
(2) X is even
(A) Statement (1) ALONE is sufficient, but Statement (2) alone is not sufficient.
(B) Statement (2) ALONE is sufficient, but Statement (1) alone is not sufficient.
(C) BOTH statements TOGETHER are sufficient, but NEITHER statement ALONE is
sufficient.
(D) EACH statement ALONE is sufficient.
(E) Statements (1) and (2) TOGETHER are NOT sufficient.

11. (Medium) What is the value of integer J?
(1) |J| = J
- 1

(2) J
J
= 1

(A) Statement (1) ALONE is sufficient, but Statement (2) alone is not sufficient.
(B) Statement (2) ALONE is sufficient, but Statement (1) alone is not sufficient.
(C) BOTH statements TOGETHER are sufficient, but NEITHER statement ALONE is
sufficient.
(D) EACH statement ALONE is sufficient.
(E) Statements (1) and (2) TOGETHER are NOT sufficient.

12. (Hard) Right triangle ABC has a height BD. What is the value of AB times BC?
(1) AB is equal to 6
(2) The product of the non - hypotenuse sides is equal to 24
(A) Statement (1) ALONE is sufficient, but Statement (2) alone is not sufficient.
(B) Statement (2) ALONE is sufficient, but Statement (1) alone is not sufficient.
(C) BOTH statements TOGETHER are sufficient, but NEITHER statement ALONE is
sufficient.
(D) EACH statement ALONE is sufficient.

!

Your way to Top Business Schools _____________
220

(E) Statements (1) and (2) TOGETHER are NOT sufficient.

13. (Easy) If a and b are integers, is x even?
(1) x = a/b, where the term leaves no reminder and b is even.
(2) x = a/b, where the term leaves no reminder and both a and b are odd.
(A) Statement (1) ALONE is sufficient, but Statement (2) alone is not sufficient.
(B) Statement (2) ALONE is sufficient, but Statement (1) alone is not sufficient.
(C) BOTH statements TOGETHER are sufficient, but NEITHER statement ALONE is
sufficient.
(D) EACH statement ALONE is sufficient.
(E) Statements (1) and (2) TOGETHER are NOT sufficient.

14. (Medium) The choir consists of 5 boys and 6 girls. In how many ways can the singers be
arranged in a row, so that all the boys are together? (assume all members of the group are
uniform and combinations within the group do not matter).
(A) 120
(B) 30
(C) 24
(D) 11
(E) 7

15. (Medium) If n = p/q (p and q are nonzero integers) is n an integer?
(1) n
2
is an integer
(2) (2n + 4)/2 is an integer
(A) Statement (1) ALONE is sufficient, but Statement (2) alone is not sufficient.
(B) Statement (2) ALONE is sufficient, but Statement (1) alone is not sufficient.
(C) BOTH statements TOGETHER are sufficient, but NEITHER statement ALONE is
sufficient.
(D) EACH statement ALONE is sufficient.
(E) Statements (1) and (2) TOGETHER are NOT sufficient.

!

Your way to Top Business Schools _____________
221


16. (Hard) 60 pieces of candy are in a box. Each piece of candy is red, green, blue, or white.
If one candy is drawn from the box at random, the probability that it will be white is 1/5
and the probability that it will be green is 1/4. What is the probability that the candy piece
will be either red or blue?
(A) 1/5
(B) 1/4
(C) 9/20
(D) 11/20
(E) 19/20

17. (Medium) Is integer R positive?
(1) R
3
= R
(2) |R| = R
(A) Statement (1) ALONE is sufficient, but Statement (2) alone is not sufficient.
(B) Statement (2) ALONE is sufficient, but Statement (1) alone is not sufficient.
(C) BOTH statements TOGETHER are sufficient, but NEITHER statement ALONE is
sufficient.
(D) EACH statement ALONE is sufficient.
(E) Statements (1) and (2) TOGETHER are NOT sufficient.

18. (Hard) Aunt Marge is giving candy to each of her nephews and nieces. She has 20 candy
pieces and she gives candy to children according to her wish. Thus, Robert gets 2 more
candy pieces than Kate. Bill gets 6 less than Mary. Mary, in turn, gets 2 more candy than
Robert. Kate gets 2 more candy than Bill. How many candy pieces does Kate get?
(A) 2
(B) 4
(C) 6
(D) 8
(E) 10

19. (Medium) 6 students in the group study different languages as specified:

!

Your way to Top Business Schools _____________
222

Russian 4
Ukrainian 3
Hebrew 2
Each student studies at least 1 language. It is also known that exactly 3 students learn
exactly 2 languages. How many students are studying all languages?
(A) 0
(B) 1
(C) 2
(D) 3
(E) 4

20. (Hard) What is the last digit of the following number [2
22
] [3
15
] [5
16
] [7
1
]?
(A) 6
(B) 5
(C) 2
(D) 1
(E) 0

21. (Hard) A soccer coach riding his bike at 24 km/h reaches his office 5 minutes late. Had
he traveled 25% faster, he would have reached the office 4 minutes earlier than the
scheduled time. How far is his office from his house?
(A) 18 km
(B) 24 km
(C) 36 km
(D) 40 km
(E) 72 km

22. (Hard) Johnny wants to buy some candy with some nickels and quarters that his mother
gave him. How many quarters does he have?
(1) The total value of Johnnys coins is 85 cents.

!

Your way to Top Business Schools _____________
223

(2) Johnny has more quarters than nickels in his pocket.
(A) Statement (1) ALONE is sufficient, but Statement (2) alone is not sufficient.
(B) Statement (2) ALONE is sufficient, but Statement (1) alone is not sufficient.
(C) BOTH statements TOGETHER are sufficient, but NEITHER statement ALONE is
sufficient.
(D) EACH statement ALONE is sufficient.
(E) Statements (1) and (2) TOGETHER are NOT sufficient.

23. (Medium) Is Z an odd integer?
(1) Z/3 is odd
(2) 3Z is odd
(A) Statement (1) ALONE is sufficient, but Statement (2) alone is not sufficient.
(B) Statement (2) ALONE is sufficient, but Statement (1) alone is not sufficient.
(C) BOTH statements TOGETHER are sufficient, but NEITHER statement ALONE is
sufficient.
(D) EACH statement ALONE is sufficient.
(E) Statements (1) and (2) TOGETHER are NOT sufficient.

24. (Medium) Is x = 0?
(1) x y = x
(2) x + y = y
(A) Statement (1) ALONE is sufficient, but Statement (2) alone is not sufficient.
(B) Statement (2) ALONE is sufficient, but Statement (1) alone is not sufficient.
(C) BOTH statements TOGETHER are sufficient, but NEITHER statement ALONE is
sufficient.
(D) EACH statement ALONE is sufficient.
(E) Statements (1) and (2) TOGETHER are NOT sufficient.

!

Your way to Top Business Schools _____________
224

25. (Hard) Bill can buy 3 pairs of jeans and 2 shirts for a price of $69 or he can buy 2 pairs of
jeans and 3 shirts for the price of $66. How much does one shirt cost?
(A) $10
(B) $12
(C) $13.20
(D) $15
(E) $16.80

26. (Hard) Is 7
7
/7
Q
an integer?
(1) 0 Q 7
(2) |Q| = Q
2

(A) Statement (1) ALONE is sufficient, but Statement (2) alone is not sufficient.
(B) Statement (2) ALONE is sufficient, but Statement (1) alone is not sufficient.
(C) BOTH statements TOGETHER are sufficient, but NEITHER statement ALONE is
sufficient.
(D) EACH statement ALONE is sufficient.
(E) Statements (1) and (2) TOGETHER are NOT sufficient.

27. (Hard) S is the set of all fractions defined by the formula n/(n + 1), where n is a positive
integer less than 20. What is the product of all the fractions that are in S?
(A) 1/20
(B) 1/21
(C) 1/2
(D) 19/40
(E) 19/20

28. (Hard) 10 applicants are interviewed for a position; among them are Paul and Jen. We
invite a person, conduct an interview and ask them to leave. What is the probability to
have neither Paul nor Jen at the first interview?
(A) 1/10
(B) 1/9

!

Your way to Top Business Schools _____________
225

(C) 1/2
(D) 8/10
(E) 9/10

29. (Hard) John has 12 clients and he wants to use color coding to identify each client. If
either a single color or a pair of two different colors can represent a client code, what is
the minimum number of colors needed for the coding? (the order of a pair of colors does
not matter)
(A) 24
(B) 12
(C) 7
(D) 6
(E) 5

30. (Hard) Mary bought some kiwis, bananas, and lemons at the grocery store in proportion
of 1:4:7 accordingly. How many lemons did Mary buy?
(1) The total number of fruits Mary bought is 24.
(2) Mary bought 8 bananas.
(A) Statement (1) ALONE is sufficient, but Statement (2) alone is not sufficient.
(B) Statement (2) ALONE is sufficient, but Statement (1) alone is not sufficient.
(C) BOTH statements TOGETHER are sufficient, but NEITHER statement ALONE is
sufficient.
(D) EACH statement ALONE is sufficient.
(E) Statements (1) and (2) TOGETHER are NOT sufficient.

31. (Hard) A perfect number is defined as one for which the sum of all the unique factors less
the number itself equals to the number. For instance, 6 is a perfect number, because the
factors of 6 (apart from 6 itself) are 1, 2 and 3, and 1 + 2 + 3 = 6. Which of the following
is a perfect number?
(A) 12
(B) 20
(C) 28

!

Your way to Top Business Schools _____________
226

(D) 48
(E) 60

32. (Hard) How many five digit numbers can be formed using the digits 0, 1, 2, 3, 4 and 5
which are divisible by 3, without repeating the digits?
(A) 15
(B) 96
(C) 120
(D) 181
(E) 216

33. (Hard) In the office supply store Brittany bought 17 pens of three kinds: ball pens, gel
pens, and ink pens, which cost $0.05, $0.10 and $0.25 each. The total that Brittany paid
was $2.05. If Brittany bought twice as many gel pens as ink pens, how many ball pens
did she buy?
(A) 4
(B) 5
(C) 7
(D) 8
(E) 10

34. (Medium) There is a certain triangle with sides 7, 10, X. It is known that X is an integer.
How many different X can there be?
(A) 8
(B) 10
(C) 13
(D) 14
(E) 16

35. (Medium) Which of the following numbers can not be divided by the square of an integer
other than 1?
(A) 75

!

Your way to Top Business Schools _____________
227

(B) 42
(C) 32
(D) 25
(E) 12

36. (Hard) One fisherman was telling his friends that he caught a fish that had a 60 foot long
head. It also had a tail that was the size of the fish's head and a half of its body, and the
body was half the size of the whole fish. What is the length of this fish?
(A) 120
(B) 200
(C) 240
(D) 400
(E) 480

37. (Hard) If Samson is filling a bathtub with COLD water, it will take him 6 minutes and 40
seconds, and if he fills it with HOT water, it will take him 8 minutes. If draining the tub
takes 13 minutes 20 seconds, how many minutes will it take to fill up the bath tub with
both HOT and COLD water running while the plug is out, so the water is constantly
draining?
(A) 16
(B) 12
(C) 8.6
(D) 5
(E) 4.75
Practice test 2
1. (Easy) If a, b, and c are 3 different integers and a b c = 55, what is the value of c?
(1) a = 5
(2) b = 11
(A) Statement (1) ALONE is sufficient, but Statement (2) alone is not sufficient.
(B) Statement (2) ALONE is sufficient, but Statement (1) alone is not sufficient.

!

Your way to Top Business Schools _____________
228

(C) BOTH statements TOGETHER are sufficient, but NEITHER statement ALONE is
sufficient.
(D) EACH statement ALONE is sufficient.
(E) Statements (1) and (2) TOGETHER are NOT sufficient.

2. (Easy) What is the product of A and B?
(1) A = 2/xm
(2) B = 24xm
(A) Statement (1) ALONE is sufficient, but Statement (2) alone is not sufficient.
(B) Statement (2) ALONE is sufficient, but Statement (1) alone is not sufficient.
(C) BOTH statements TOGETHER are sufficient, but NEITHER statement ALONE is
sufficient.
(D) EACH statement ALONE is sufficient.

3. (Easy) (0.2 14 - 15/3 + 15 3/90)/(8 0.0125) = ?
(A) 2
(B) 1
(C) 0
(D) - 1
(E) - 17

4. (Medium) Two Watermelons of the same sort, A and B, are on sale. Watermelon A has
circumference of 60 cm, watermelon B, of 50 cm. If Watermelon A is 1.5 times more
expensive than Watermelon B, which watermelon is a better buy?
(A) A
(B) B
(C) Either
(D) Neither
(E) Impossible to determine

!

Your way to Top Business Schools _____________
229

5. (Medium) If P# = P/(P - 1), what is the value of P##?
(A) P/(P - 1)
(B) 1/P
(C) P
(D) 2 - P
(E) P - 1

6. (Medium) What is the product of 6 consecutive integers?
(1) The last integer is 4
(2) The sequence has both positive and negative integers

(A) Statement (1) ALONE is sufficient, but Statement (2) alone is not sufficient.
(B) Statement (2) ALONE is sufficient, but Statement (1) alone is not sufficient.
(C) BOTH statements TOGETHER are sufficient, but NEITHER statement ALONE is
sufficient.
(D) EACH statement ALONE is sufficient.
(E) Statements (1) and (2) TOGETHER are NOT sufficient.

7. (Hard) Which set has the greatest standard deviation?
I. 1, 3, 5, 7, 9
II. 2, 4, 6, 8, 10
III. 1, - 1, - 3, - 5, - 7
(A) I
(B) II
(C) III
(D) I and II
(E) none

8. (Easy) What is the volume of a cardboard box with sides a, b, and c?

!

Your way to Top Business Schools _____________
230

(1) a = 12/bc
(2) b = 3, c = 2
(A) Statement (1) ALONE is sufficient, but Statement (2) alone is not sufficient.
(B) Statement (2) ALONE is sufficient, but Statement (1) alone is not sufficient.
(C) BOTH statements TOGETHER are sufficient, but NEITHER statement ALONE is
sufficient.
(D) EACH statement ALONE is sufficient.
(E) Statements (1) and (2) TOGETHER are NOT sufficient.

9. (Hard) Water flows into an empty tank of 54 liters via 12 small pipes. The rate of each
pipe is 1 liter per hour. However, water flows out of the tank via several big pipes at the
rate of 1.5 liter per hour. If after 12 hours, the tank is completely full, how many big
pipes are there?
(A) 2.5
(B) 3
(C) 4
(D) 5
(E) 6

10. (Hard) There are 6 cards numbered from 1 to 6. They are placed into a box, and then one
is drawn and put back and then another card is drawn and put back. What is the
probability that one of the cards was 5 if the sum of the two was 8?
(A) 1
(B) 2/3
(C) 1/2
(D) 2/5
(E) 1/3

11. (Hard) If x = 1/6, which expression has the greatest value?
(A) x
1/3

(B) x

!

Your way to Top Business Schools _____________
231

(C) 1/2x
(D) x/0.03
(E) x
- 3


12. (Medium) 4% of population of country L has military experience and supports a war
against Country G. If 20% of those who support the war have no military experience
what percentage of the country's population actually supports war against G?
(A) 5%
(B) 7.5%
(C) 10%
(D) 12.5%
(E) 15%

13. (Hard) In the Cartesian coordinate system, Point A has coordinates (6, - 7) and Point B
has coordinates (4, 5). If a line is drawn to connect Point A and Point B, does point C lie
on line AB?
(1) Coordinates of Point C are ( 5, - 1)
(2) Point C equidistant from Point A and Point B.
(A) Statement (1) ALONE is sufficient, but Statement (2) alone is not sufficient.
(B) Statement (2) ALONE is sufficient, but Statement (1) alone is not sufficient.
(C) BOTH statements TOGETHER are sufficient, but NEITHER statement ALONE is
sufficient.
(D) EACH statement ALONE is sufficient.
(E) Statements (1) and (2) TOGETHER are NOT sufficient.

14. (Hard) Two cities, Kensington MD and Reston VA are 30 km apart. From both of these
cities, simultaneously, two hikers start their journeys towards each other. They are
walking at a constant speed of 5 km/hour each. Simultaneously, a fly leaves the city of
Kensington. It flies at the speed of 10 km/hour and passes the hiker from its city. When it
reaches the Reston hiker, it turns around and flies back to the Kensington hiker. It keeps
doing so until the hikers meet. At the moment they do meet, the fly lands on the shoulder
of the Kensington hiker as he continues his journey to Reston. How many kilometers has
the fly flown?
(A) 25

!

Your way to Top Business Schools _____________
232

(B) 30
(C) 37.5
(D) 45
(E) 60

15. (Hard) If x is a positive integer, is x > 2.5x - 5?
(1) x < 2
(2) x is a prime number
(A) Statement (1) ALONE is sufficient, but Statement (2) alone is not sufficient.
(B) Statement (2) ALONE is sufficient, but Statement (1) alone is not sufficient.
(C) BOTH statements TOGETHER are sufficient, but NEITHER statement ALONE is
sufficient.
(D) EACH statement ALONE is sufficient.
(E) Statements (1) and (2) TOGETHER are NOT sufficient.

16. (Medium) (40 x
2
y
2
)/z is divisible by prime number Q. Is Q an even prime number?
(1) Product of x
2
y
2
is even
(2) z Q = 6
(A) Statement (1) ALONE is sufficient, but Statement (2) alone is not sufficient.
(B) Statement (2) ALONE is sufficient, but Statement (1) alone is not sufficient.
(C) BOTH statements TOGETHER are sufficient, but NEITHER statement ALONE is
sufficient.
(D) EACH statement ALONE is sufficient.
(E) Statements (1) and (2) TOGETHER are NOT sufficient.

17. (Hard) Kate and David each have $10. Together they flip a coin 5 times. Every time the
coin lands on heads, Kate gives David $1. Every time the coin lands on tails, David gives
Kate $1. After the coin is flipped 5 times, what is the probability that Kate has more than
$10 but less than $15
(A) 5/16

!

Your way to Top Business Schools _____________
233

(B) 15/32
(C) 1/2
(D) 21/32
(E) 11/16

18. (Easy) To build a rectangular chicken pen, Mike has 40 meters of net. If Mike wants to
maximize the area of the pan, what will be the most favorable dimensions?
(A) 12 8
(B) 15 8
(C) 10 10
(D) 15 15
(E) 15 5

19. (Medium) Is T/S > F/G?
(1) T < S
(2) F > G
(A) Statement (1) ALONE is sufficient, but Statement (2) alone is not sufficient.
(B) Statement (2) ALONE is sufficient, but Statement (1) alone is not sufficient.
(C) BOTH statements TOGETHER are sufficient, but NEITHER statement ALONE is
sufficient.
(D) EACH statement ALONE is sufficient.
(E) Statements (1) and (2) TOGETHER are NOT sufficient.

20. (Medium) When integer p is divided by 7, the remainder is 2. Is p divisible by 8?
(1) p is divisible by 2 and 3
(2) p < 100
(A) Statement (1) ALONE is sufficient, but Statement (2) alone is not sufficient.
(B) Statement (2) ALONE is sufficient, but Statement (1) alone is not sufficient.
(C) BOTH statements TOGETHER are sufficient, but NEITHER statement ALONE is
sufficient.

!

Your way to Top Business Schools _____________
234

(D) EACH statement ALONE is sufficient.
(E) Statements (1) and (2) TOGETHER are NOT sufficient.

21. (Hard) Otto takes 20 days to empty a 50 - liter barrel of beer and, together with Hanna, he
can empty it in 14 days, how many days will it take Hanna to finish a barrel of beer by
herself?
(A) 17
(B) 33
(C) 39
(D) 47
(E) 58

22. (Hard) Is X divisible by 15?
(1) When X is divided by 10, the result is an integer
(2) X
2
is a multiple of 30
(A) Statement (1) ALONE is sufficient, but Statement (2) alone is not sufficient.
(B) Statement (2) ALONE is sufficient, but Statement (1) alone is not sufficient.
(C) BOTH statements TOGETHER are sufficient, but NEITHER statement ALONE is
sufficient.
(D) EACH statement ALONE is sufficient.
(E) Statements (1) and (2) TOGETHER are NOT sufficient.

23. (Hard) A machine consisting of 3 connected wheels is mixing concrete. The engine is
running at the speed of 60 rpm and the mixer is mixing at the rate of 8 rpm. What is the
diameter of the mixing wheel if the radius of the smaller (drive) wheel is 2 inches?
(A) 12
(B) 15
(C) 25
(D) 30
(E) 32

!

Your way to Top Business Schools _____________
235

24. (Easy) What is the value of x?
(1) x
2
- 12x + 32 = 0
(2) x > = 0
(A) Statement (1) ALONE is sufficient, but Statement (2) alone is not sufficient.
(B) Statement (2) ALONE is sufficient, but Statement (1) alone is not sufficient.
(C) BOTH statements TOGETHER are sufficient, but NEITHER statement ALONE is
sufficient.
(D) EACH statement ALONE is sufficient.
(E) Statements (1) and (2) TOGETHER are NOT sufficient.

25. (Medium) A drugstore has 80 different kinds of drugs on its easy access shelves: 30% are
for allergy, 25% are laxative, 20% are for cold, 5% are for headaches, and the rest are
pain relief medicines. A blind person walks into the store to get some medicine for her
cold. Assuming there are 20 bottles of each individual drug on the shelves and the blind
customer does not ask for assistance, how many bottles will she need to purchase to
ensure she has at least one bottle of cold medicine?
(A) 61
(B) 120
(C) 800
(D) 1281
(E) 1321

26. (Medium) What is the value of R?
(1) R
R
= R
(2) (1/R)
R
= R
(A) Statement (1) ALONE is sufficient, but Statement (2) alone is not sufficient.
(B) Statement (2) ALONE is sufficient, but Statement (1) alone is not sufficient.
(C) BOTH statements TOGETHER are sufficient, but NEITHER statement ALONE is
sufficient.
(D) EACH statement ALONE is sufficient.
(E) Statements (1) and (2) TOGETHER are NOT sufficient.

!

Your way to Top Business Schools _____________
236


27. (Hard) Carly has 3 movies that she can watch during the weekend: 1 Action movies, 1
Comedy, and 1 Drama. However, she needs to watch the Drama 3 times. Assuming Carly
has time for 5 movies and intends to watch all of them, in how many ways can she do so?
(A) 6
(B) 20
(C) 24
(D) 60
(E) 120

28. (Medium) If x
2
= y + 5 , y = z - 2 and z = 2x , is x
3
+ y
2
+ z divisible by 7?
(1) x > 0
(2) y = 4
(A) Statement (1) ALONE is sufficient, but Statement (2) alone is not sufficient.
(B) Statement (2) ALONE is sufficient, but Statement (1) alone is not sufficient.
(C) BOTH statements TOGETHER are sufficient, but NEITHER statement ALONE is
sufficient.
(D) EACH statement ALONE is sufficient.
(E) Statements (1) and (2) TOGETHER are NOT sufficient.

29. (Medium) Is 4Q/11 a positive integer?
(1) Q is a prime number
(2) 2Q is divisible by 11
(A) Statement (1) ALONE is sufficient, but Statement (2) alone is not sufficient.
(B) Statement (2) ALONE is sufficient, but Statement (1) alone is not sufficient.
(C) BOTH statements TOGETHER are sufficient, but NEITHER statement ALONE is
sufficient.
(D) EACH statement ALONE is sufficient.
(E) Statements (1) and (2) TOGETHER are NOT sufficient.

!

Your way to Top Business Schools _____________
237

30. (Hard) On any given Saturday, flights arrive at XYZ airport every hour, for 24 hours. The
average number of flights arriving at XYZ airport on any Saturday is a multiple of 12. Is
the total number of flights that arrived on a Saturday > 180?
(1) On the same Saturday, the median number of flight arrivals every hour is 17
(2) On the same Saturday, Highest number of flight arrivals in an hour was 30.
(A) Statement (1) ALONE is sufficient, but Statement (2) alone is not sufficient.
(B) Statement (2) ALONE is sufficient, but Statement (1) alone is not sufficient.
(C) BOTH statements TOGETHER are sufficient, but NEITHER statement ALONE is
sufficient.
(D) EACH statement ALONE is sufficient.
(E) Statements (1) and (2) TOGETHER are NOT sufficient.

31. (Hard) A flower shop has 2 tulips, 2 roses, 2 daisies, and 2 lilies. If two flowers are sold
at random, what is the probability that both are not tulips?
(A) 1/8
(B) 1/7
(C) 1/2
(D) 7/8
(E) 27/28

32. (Easy) What is the value of X?
(1) X is a mode of [3, 0, 1, - 1, 0, 5, 1]
(2) X is neither positive nor negative
(A) Statement (1) ALONE is sufficient, but Statement (2) alone is not sufficient.
(B) Statement (2) ALONE is sufficient, but Statement (1) alone is not sufficient.
(C) BOTH statements TOGETHER are sufficient, but NEITHER statement ALONE is
sufficient.
(D) EACH statement ALONE is sufficient.
(E) Statements (1) and (2) TOGETHER are NOT sufficient.

!

Your way to Top Business Schools _____________
238

33. (Medium) Is 4t
3
- 2t
2
- 8t + 16 divisible by t
2
?
(1) t > 1
(2) t is an even prime number
(A) Statement (1) ALONE is sufficient, but Statement (2) alone is not sufficient.
(B) Statement (2) ALONE is sufficient, but Statement (1) alone is not sufficient.
(C) BOTH statements TOGETHER are sufficient, but NEITHER statement ALONE is
sufficient.
(D) EACH statement ALONE is sufficient.
(E) Statements (1) and (2) TOGETHER are NOT sufficient.

34. (Hard) A fair coin marked 1 and 2, and a fair die are rolled together. What is a probability
to have the sum even?
(A) 1/8
(B) 1/4
(C) 1/2
(D) 3/4
(E) 7/8

35. (Hard) Three Xeroxes are making copies of the same document. Machine A makes 12
copies per minute, Machine B - 7, and Machine C - 19. It costs 8c/copy for Copier A,
5c/copy for Copier B, and 11c/copy for Copier C. If a separate attendant has to be hired
for each copier and be paid $30 per hour (you have to pay $60 if the attendant works for
one hour and one minute), which copier alone will be the most efficient choice for 1200
copies?
(A) Copier A
(B) Copier B
(C) Copier C
(D) All
(E) A and B

36. (Medium) If a, b, and c are positive distinct integers, is (a/b)/c and integer?
(1) c = 2

!

Your way to Top Business Schools _____________
239

(2) a = b + c
(A) Statement (1) ALONE is sufficient, but Statement (2) alone is not sufficient.
(B) Statement (2) ALONE is sufficient, but Statement (1) alone is not sufficient.
(C) BOTH statements TOGETHER are sufficient, but NEITHER statement ALONE is
sufficient.
(D) EACH statement ALONE is sufficient.
(E) Statements (1) and (2) TOGETHER are NOT sufficient.

37. (Medium) {a, b, 4, 6, 2}
Is the average of a set of 5 distinct positive integers less than the median?
(1) the highest number in the set is 6
(2) the lowest number in the set is 2
(A) Statement (1) ALONE is sufficient, but Statement (2) alone is not sufficient.
(B) Statement (2) ALONE is sufficient, but Statement (1) alone is not sufficient.
(C) BOTH statements TOGETHER are sufficient, but NEITHER statement ALONE is
sufficient.
(D) EACH statement ALONE is sufficient.
(E) Statements (1) and (2) TOGETHER are NOT sufficient.
Practice test 3
1. (Easy) (4.5 - 2 3/6 + 1/4
2
)/0.75
(A) 1/5
(B) 15/4
(C) 15/3
(D) 19/4
(E) 57/16

2. (Easy) Is 18K/L an integer?
(1) K
2
/L
2
is an integer
(2) K - L = L

!

Your way to Top Business Schools _____________
240

(A) Statement (1) ALONE is sufficient, but Statement (2) alone is not sufficient.
(B) Statement (2) ALONE is sufficient, but Statement (1) alone is not sufficient.
(C) BOTH statements TOGETHER are sufficient, but NEITHER statement ALONE is
sufficient.
(D) EACH statement ALONE is sufficient.
(E) Statements (1) and (2) TOGETHER are NOT sufficient.

3. (Hard) Two assistants can type up the report in 12.5 hours and edit it in 7.5 hours. If
Mary will take 30 hours to type the report and Jim will take 12 hours to edit it alone, how
many hours will it take if Jim types the report and Mary edits it immediately after he is
done?
(A) 41.4
(B) 34.1
(C) 13.4
(D) 12.4
(E) 10.8

4. (Medium) At a dog competition, a dog can be awarded 10 points if it runs through 4
pipes, makes 10 jumps, and walks on 2 beams. If Roofy gets 9 points after missing a Pipe
and Ralph gets 7 points after missing a pipe and a beam, how many points does Butch get
if he missed 4 jumps but went through the rest of the exercise perfectly?
(A) 8.2
(B) 8.4
(C) 8.8
(D) 9.0
(E) 9.2

5. (Medium) A circle is inscribed in a square. If a diagonal running through the center of the
circle is 4 cm long, what is the area of the square that is not occupied by the circle?

(A) 1.7
(B) 2.7

!

Your way to Top Business Schools _____________
241

(C) 12
(D) 24
(E) 25

6. (Hard) A contractor estimated that his 10-men crew could complete the construction in
110 days if there was no rain. (Assume the crew does not work on any rainy day and rain
is the only factor that can deter the crew from working). However, after 5 days of rain, on
the 61
st
day, he hired 6 more people to finish the project early. If the job was done in 100
days, on how many days was it raining?
(A) 4
(B) 5
(C) 6
(D) 7
(E) 8

7. (Easy) After 6 games, team B had an average of 61.5 points per game. If it got only 47
points in game 7, how many more points does it need to score to get its total above 500?
(A) 85
(B) 74
(C) 67
(D) 53
(E) 28

8. (Medium) If 12 ounces of strong solution of vinegar is mixed with 50 ounces of water to
form three percent vinegar, what was the original strength of the vinegar solution?
(A) 19.3
(B) 17
(C) 16.67
(D) 15.5
(E) 12.5

!

Your way to Top Business Schools _____________
242

9. (Hard) A conference room is equipped with a total of 45 leather and upholstery chairs. If
there are 14 reclining chairs and 20% of the leather chairs are reclining and 10 of the
reclining chairs are upholstery, how many upholstery chairs are non-reclining?
(A) 31
(B) 25
(C) 15
(D) 14
(E) 10

10. (Easy) A smith received an order to make a 1 million liter cube-shaped tank. If he has
only 4x2 sheets of metal that can be cut, how many metal sheets will be required for this
order if the smith has to make the sides first and then weld them?
(A) 92
(B) 90
(C) 82
(D) 78
(E) 75

11. (Easy) A bus traveling between two towns, 100 km apart, makes 2 stops between them. If
all stops are 15 minutes each, what is the average speed of the bus (excluding stops) if it
takes the driver four hours and five minutes from Leaving Terminal in town A to arriving
to the same terminal?
(A) 83
(B) 76
(C) 71
(D) 66
(E) 61

12. (Hard) If the probability that Mike can win a championship is 1/4, and that of Rob
winning is 1/3 and that of Ben winning is 1/6, what is the probability that Mike or Rob
win the championship but not Ben?
(A) 1/6
(B) 5/12

!

Your way to Top Business Schools _____________
243

(C) 7/12
(D) 5/6
(E) 1

13. (Easy) If M is three time as great as N which is 5 times smaller than 25 less 3 divided by
2. What is the value of M?
(A) 1
(B) 2
(C) 3
(D) 6
(E) 12

14. (Easy) What is the number of integers from 1 to 1000 (inclusive) that are not divisible by
11 nor by 35?
(A) 884
(B) 890
(C) 892
(D) 910
(E) 945

15. (Easy) At Springfield High, three fourths of the male and one half of the female students
speak a foreign language. If there are two-thirds as many male as female students at the
high school, what fraction of the students speak a foreign language?
(A) 1/3
(B) 2/3
(C) 1/2
(D) 4/3
(E) 3/5

16. (Easy) Which expression is the greatest?
(A) 1876452/1876455

!

Your way to Top Business Schools _____________
244

(B) 1883456/1883459
(C) 1883491/1883494
(D) 1883446/1883449
(E) 1883453/1883456

17. (Easy) Company policies require that all projects have only even number of employees, a
representative of at least 3 ethnic groups has to be present, at least 33% women, and one
supervisor per 5 employees. What will be the smallest team size for the lithium panel
project if it is to meet the company requirements in 100% of cases and requires 3 top
scientists and 2 assistants all of whom are Asians?
(A) 12
(B) 11
(C) 10
(D) 9
(E) 8

18. (Easy) A lumberjack crew needs to secure a safety zone for cutting down the trees. How
far from the first tree should the safety barrier be moved so that it is at least 5 meters
away from a fallen tree?
(1) The height of the first tree is 50m
(2) Currently the distance from the barrier to the top of the tree is 70 meters
(A) Statement (1) ALONE is sufficient, but Statement (2) alone is not sufficient.
(B) Statement (2) ALONE is sufficient, but Statement (1) alone is not sufficient.
(C) BOTH statements TOGETHER are sufficient, but NEITHER statement ALONE is
sufficient.
(D) EACH statement ALONE is sufficient.
(E) Statements (1) and (2) TOGETHER are NOT sufficient.

19. (Easy) If 69% of K is 23, how much is 23% of K?
(A) 11.11
(B) 10.59
(C) 8.33

!

Your way to Top Business Schools _____________
245

(D) 7.67
(E) 6.67

20. (Easy) $Y$ = (Y
Y
)/(Y Y 2). What is the last digit of $($4$)$?
(A) 8
(B) 6
(C) 4
(D) 2
(E) 1

21. (Easy) If a 5-year project is structured by doubling the original investment every year,
what percentage of the total cost was invested in the first year?
(A) 12.5%
(B) 10%
(C) 8.25%
(D) 5%
(E) 3%

22. (Easy) Is area of triangle ABC greater than area of triangle DEF?
(1) Area of ABC < Perimeter of DEF
(2) Angles of ABC = Angles of DEF
(A) Statement (1) ALONE is sufficient, but Statement (2) alone is not sufficient.
(B) Statement (2) ALONE is sufficient, but Statement (1) alone is not sufficient.
(C) BOTH statements TOGETHER are sufficient, but NEITHER statement ALONE is
sufficient.
(D) EACH statement ALONE is sufficient.
(E) Statements (1) and (2) TOGETHER are NOT sufficient.

23. (Medium) Which of the following expressions has the greatest value?
(A) 0.456

!

Your way to Top Business Schools _____________
246

(B) 1/2 - (1/2)
4

(C) 300/650
(D) 3 (3/19)
(E) 0.17

24. (Easy) A plane flying from Europe to South America takes 12.4 hours. The return trip
takes 10.25 hours. If the average ground speed of a plane is 700 km/h, what is the
distance between European and South American cities?
(A) 8680
(B) 8273
(C) 7931
(D) 7245
(E) 7131

25. (Hard) If 40% of unsuccessful suicide attempts are repeated within 6 months and 25% of
suicides are successful, what percentage of suicides in any 6 months period is the first
attempt?
(A) 30
(B) 40
(C) 50
(D) 60
(E) 70

26. (Medium) Water evaporates at the rate of two liters per hour per one square meter of
surface. How long will it take to evaporate 30 liters of water from a 56 ton swimming
pool?
(1) The depth of the pool is 2 meters
(2) The radius of the circular pool is 3 meters
(A) Statement (1) ALONE is sufficient, but Statement (2) alone is not sufficient.
(B) Statement (2) ALONE is sufficient, but Statement (1) alone is not sufficient.
(C) BOTH statements TOGETHER are sufficient, but NEITHER statement ALONE is
sufficient.

!

Your way to Top Business Schools _____________
247

(D) EACH statement ALONE is sufficient.
(E) Statements (1) and (2) TOGETHER are NOT sufficient.

27. (Hard) 10 sergeants and 7 lieutenants meet at the training camp. If each sergeant salutes
other sergeants and lieutenants once and each lieutenant salutes each of the sergeants
back but not each other, how many salutes took place?
(A) 144
(B) 131
(C) 115
(D) 90
(E) 45

28. (Hard) To make lemonade, a recipe requires 4 lemons, 12 ounces of sugar, and 2 gallons
of water. If each lemon contains 0.4 cups of 75% lemon acid and we use 2 cups of 45%
lemon acid, how much water will be needed?
(A) 1
(B) 1.33
(C) 1.5
(D) 1.75
(E) 2.12

29. (Hard) Which of the following lines is parallel to line x = 2.66 - 2y?
(A) 2x + 25
(B) 2.66x - 2
(C) x + 3y
(D) - 1/2x - 32
(E) 1/2x + 3

30. (Easy) Is x/12 an integer?
(1) x is a multiple of 60
(2) x x x > 0

!

Your way to Top Business Schools _____________
248

(A) Statement (1) ALONE is sufficient, but Statement (2) alone is not sufficient.
(B) Statement (2) ALONE is sufficient, but Statement (1) alone is not sufficient.
(C) BOTH statements TOGETHER are sufficient, but NEITHER statement ALONE is
sufficient.
(D) EACH statement ALONE is sufficient.
(E) Statements (1) and (2) TOGETHER are NOT sufficient.

31. (Medium) If x
2
- 100 < 300, how many integers x satisfy this condition?
(A) 42
(B) 39
(C) 38
(D) 37
(E) 19

32. (Medium) (1/2)
2
+ (1/4)
- 2
+ 1/8
2/3
+ 1/4
1/2
= ?
(A) 20(3/4)
(B) 17
(C) 9(1/2)
(D) 6
(E) 4(3/4)

33. (Medium) If a company sells s tons of product A annually and charges b dollars per ton,
what is profit if it costs g dollars to manufacture a kilogram of product A and p dollars to
ship it to the customer?
(A) b - g 1000 + p 1000
(B) b s - (g + p)/1000
(C) [b - (g + p)] s
(D) [g/1000 - (b + p)] s
(E) [b/1000 - (g + p)] s

!

Your way to Top Business Schools _____________
249

34. (Medium) If p is a prime number such that 2 < p < 15 what is the remainder when p
2
is
divided by 4?
(A) 0
(B) 1
(C) 2
(D) 3
(E) 4

35. (Medium) A car has been sold by a dealership for a profit of 25%. If the dealership
bought it from a manufacturer that in turn made a profit of 12.5%, what was the end price
to the consumer if the cost to manufacture a car is $12,000?
(A) 12,750
(B) 13,750
(C) 15,500
(D) 16,150
(E) 16,875

36. (Medium) If the blades of a windmill are 10 meters long each and the edges of the blades
rotate at the speed of 315 meters per minute, how many rounds per minute do the blades
make?
(A) 2
(B) 3
(C) 4
(D) 5
(E) 6

37. (Hard) A construction company wants to number new houses using digit plates only. If
the company puts an order for 1212 plates how many houses are to be given numbers?
(The numbers of houses are consecutive and the number of the first house is 1).
(A) 260
(B) 440
(C) 556

!

Your way to Top Business Schools _____________
250

(D) 792
(E) 1200
Practice test 4
1. (Medium) The price of a certain commodity increased at a rate of X% per year between
2000 and 2004. If the price was M dollars in 2001 and N dollars in 2003, what was the
price in 2002 in terms of M and N?
(A) (M N)
(B) N (N/M)
(C) N M
(D) N M/N
(E) N M
3/2


2. (Medium) It took the bus three hours to get from town A to town B. What was the
average speed of the bus for the entire trip?
(1) After one hour the bus finished 1/3 of the distance going at 60 km/h
(2) During the second hour the average speed of the bus was 120 km/h, twice its speed
during the third hour
(A) Statement (1) ALONE is sufficient, but Statement (2) alone is not sufficient
(B) Statement (2) ALONE is sufficient, but Statement (1) alone is not sufficient
(C) BOTH statements TOGETHER are sufficient, but NEITHER statement ALONE is
sufficient
(D) EACH statement ALONE is sufficient
(E) Statements (1) and (2) TOGETHER are NOT sufficient

3. (Hard) If the circus were to sell all of its 220 tickets for this month's performance at their
usual price, the revenue from sales would be 10% greater than that collected last month.
However, the circus raised the ticket price by 5% and sold only 200 tickets as a result.
What percent less was last month's revenue than that of this month?
(A) 2
(B) 5
(C) 100/21
(D) 110/20

!

Your way to Top Business Schools _____________
251

(E) 9/4

4. (Medium) 4 women and 6 men work in the accounting department. In how many ways
can a committee of 3 be formed if it has to include at least one woman?
(A) 36
(B) 60
(C) 72
(D) 80
(E) 100

5. (Hard) Two sets are defined as follows:
A = {2, 3, 4, 4, 4}
B = {0, 1, 2}.

If a number is taken from set A at random and another number is taken from set B at
random, what is the probability that the sum of these numbers is a prime integer?
(A) 1/15
(B) 2/15
(C) 5/15
(D) 7/15
(E) 9/15

6. (Medium) The vertices of a triangle have coordinates (x, 1), (5, 1), and (5, y) where x < 5
and y > 1. What is the area of the triangle?
(1) x = y
(2) Angle at the vertex (x, 1) = angle at the vertex (5, y)
(A) Statement (1) ALONE is sufficient, but Statement (2) alone is not sufficient
(B) Statement (2) ALONE is sufficient, but Statement (1) alone is not sufficient
(C) BOTH statements TOGETHER are sufficient, but NEITHER statement ALONE is
sufficient

!

Your way to Top Business Schools _____________
252

(D) EACH statement ALONE is sufficient
(E) Statements (1) and (2) TOGETHER are NOT sufficient

7. (Hard) Among 200 people 56% like strawberry jam, 44% like apple jam, and 40% like
raspberry jam. If 30% of people like both strawberry and apple jam, what is the largest
possible number of people who like raspberry jam but do not like either strawberry or
apple jam?
(A) 20
(B) 60
(C) 80
(D) 86
(E) 92

8. (Easy) The sequence is defined as follows:
A(1) = 1; A(2) = - 1
A(n + 1) = A(n) + 2 A(n - 1).
What is the sum A(1) + A(2) + ... + A(1001) ?
(A) - 2
(B) - 1
(C) 0
(D) 1
(E) 2

9. (Easy) Set T consists of odd integers divisible by 5. Is standard deviation of T positive?
(1) All members of T are positive
(2) T consists of only one member
(A) Statement (1) ALONE is sufficient, but Statement (2) alone is not sufficient
(B) Statement (2) ALONE is sufficient, but Statement (1) alone is not sufficient
(C) BOTH statements TOGETHER are sufficient, but NEITHER statement ALONE is
sufficient

!

Your way to Top Business Schools _____________
253

(D) EACH statement ALONE is sufficient
(E) Statements (1) and (2) TOGETHER are NOT sufficient

10. (Medium) Machine A can produce 50 components a day while machine B only 40. The
monthly maintenance cost for machine A is $1500 while that for machine B is $550. If
each component generates an income of $10, what is the least number of days per month
that the plant has to work to justify the usage of machine A instead of machine B?
(A) 6
(B) 7
(C) 9
(D) 10
(E) 11

11. (Hard) Is |x - 1| < 1?
(1) (x - 1)
2
1
(2) x
2
- 1 > 0
(A) Statement (1) ALONE is sufficient, but Statement (2) alone is not sufficient
(B) Statement (2) ALONE is sufficient, but Statement (1) alone is not sufficient
(C) BOTH statements TOGETHER are sufficient, but NEITHER statement ALONE is
sufficient
(D) EACH statement ALONE is sufficient
(E) Statements (1) and (2) TOGETHER are NOT sufficient

12. (Medium) @ represents the tens digit in integer N = 1947@6. What is @?
(1) N is divisible by 4
(2) N is divisible by 3
(A) Statement (1) ALONE is sufficient, but Statement (2) alone is not sufficient
(B) Statement (2) ALONE is sufficient, but Statement (1) alone is not sufficient
(C) BOTH statements TOGETHER are sufficient, but NEITHER statement ALONE is
sufficient

!

Your way to Top Business Schools _____________
254

(D) EACH statement ALONE is sufficient
(E) Statements (1) and (2) TOGETHER are NOT sufficient

13. (Hard) If the farmer sells 75 of his chickens, his stock of feed will last for 20 more days
than planned, but if he buys 100 more chickens, he will run out of feed 15 days earlier
than planned. If no chickens are sold or bought, the farmer will be exactly on schedule.
How many chickens does the farmer have?
(A) 60
(B) 120
(C) 240
(D) 275
(E) 300

14. (Medium) The 19th of September 1987 was Saturday. What day was the 21st of
September 1990 if 1988 was a leap - year?
(A) Monday
(B) Tuesday
(C) Wednesday
(D) Thursday
(E) Friday

15. (Medium) At what angle do the lines y = K x + L and x = y + K L intersect?
(1) K = 2
(2) K = L
(A) Statement (1) ALONE is sufficient, but Statement (2) alone is not sufficient
(B) Statement (2) ALONE is sufficient, but Statement (1) alone is not sufficient
(C) BOTH statements TOGETHER are sufficient, but NEITHER statement ALONE is
sufficient
(D) EACH statement ALONE is sufficient
(E) Statements (1) and (2) TOGETHER are NOT sufficient

!

Your way to Top Business Schools _____________
255

16. (Hard) What is (x
2
- 6 x + 9) + (2 - x) + (x - 3) if each term in this expression is well
defined?
(A) (2 - x)
(B) 2 x - 6 + (2 - x)
(C) (2 - x) + (x - 3)
(D) 2 x - 6 + (x - 2)
(E) x + (x - 2)

17. (Medium) Two payment schemes are available for customers in the N&K store. The first
scheme implies a down payment of 20% of the purchase price and 10 monthly payments
of 10% each. The second implies a down payment of 10% and 20 monthly payments of
8% each. If a customer buys a TV for $216, by what percent will he find the first scheme
cheaper than the second (approximately)?
(A) 14%
(B) 27%
(C) 30%
(D) 34%
(E) 35%

18. (Medium) A man cycling along the road noticed that every 12 minutes a bus overtakes
him while every 4 minutes he meets an oncoming bus. If all buses and the cyclist move at
constant speed, what is the time interval between consecutive buses?
(A) 5 minutes
(B) 6 minutes
(C) 8 minutes
(D) 9 minutes
(E) 10 minutes

19. (Easy) What is the hundreds digit of 2
10
+ 2
5
+ 1?
(A) 0
(B) 1
(C) 2

!

Your way to Top Business Schools _____________
256

(D) 5
(E) 8

20. (Medium) Richard is 3 years younger than his sister. How old will Richard be in 5 years?
(1) Two years ago Richard was twice as young as his sister
(2) If Richard's sister were born 2 years earlier, she would now be twice as old as Richard
(A) Statement (1) ALONE is sufficient, but Statement (2) alone is not sufficient
(B) Statement (2) ALONE is sufficient, but Statement (1) alone is not sufficient
(C) BOTH statements TOGETHER are sufficient, but NEITHER statement ALONE is
sufficient
(D) EACH statement ALONE is sufficient
(E) Statements (1) and (2) TOGETHER are NOT sufficient

21. (Hard) Mary and Joe are to throw three dice each. The score is the sum of points on all
three dice. If Mary scores 10 in her attempt what is the probability that Joe will outscore
Mary in his?
(A) 24/64
(B) 32/64
(C) 36/64
(D) 40/64
(E) 42/64

22. (Medium) M, N, and O are midpoints of sides AB, BC, and AC of triangle ABC. What is
the area of triangle MON?
(1) The area of ABC is 3/4
(2) ABC is an equilateral triangle with height 3/2
(A) Statement (1) ALONE is sufficient, but Statement (2) alone is not sufficient
(B) Statement (2) ALONE is sufficient, but Statement (1) alone is not sufficient
(C) BOTH statements TOGETHER are sufficient, but NEITHER statement ALONE is
sufficient

!

Your way to Top Business Schools _____________
257

(D) EACH statement ALONE is sufficient
(E) Statements (1) and (2) TOGETHER are NOT sufficient

23. (Medium) Is the mean of a non - empty set S bigger than its median?
(1) All members of S are consecutive multiples of 3
(2) The sum of all members of S equals 75
(A) Statement (1) ALONE is sufficient, but Statement (2) alone is not sufficient
(B) Statement (2) ALONE is sufficient, but Statement (1) alone is not sufficient
(C) BOTH statements TOGETHER are sufficient, but NEITHER statement ALONE is
sufficient
(D) EACH statement ALONE is sufficient
(E) Statements (1) and (2) TOGETHER are NOT sufficient

24. (Easy) If X is an odd integer, which of the following numbers must be even?
(A) X
3
/3
(B) (X
2
- 1)/2
(C) X p + 1 where p is a prime number
(D) (X + 7) (X - 2)/2
(E) X
X - 1


25. (Hard) The diameter of each of the car's wheels is 20 inches. What was the average speed
of the car in km/h if after the two-hour journey each wheel made 75,000 revolutions? (1
inch = 0.0252 meter)
(A) 32
(B) 41
(C) 59
(D) 74
(E) 88

26. (Easy) If X Y is divisible by 4, which of the following must be true?

!

Your way to Top Business Schools _____________
258

(A) If X is even then Y is odd
(B) If X = 2 then Y is not an integer
(C) If X is 0 then X + Y is not 0
(D) X
Y
is even
(E) X/Y is not an integer

27. (Medium) Comet A is seen near the Earth every 12 years while comet B every 20 years.
If both comets were observed in 1979, for how many years do we have to wait to see the
two comets together again? (Assume now is 2004)
(A) 16
(B) 20
(C) 25
(D) 32
(E) 35

28. (Medium) The average of four distinct positive integers is 60. How many integers of
these four are smaller than 50?
(1) One of the integers is 200
(2) The median of the four integers is 50
(A) Statement (1) ALONE is sufficient, but Statement (2) alone is not sufficient
(B) Statement (2) ALONE is sufficient, but Statement (1) alone is not sufficient
(C) BOTH statements TOGETHER are sufficient, but NEITHER statement ALONE is
sufficient
(D) EACH statement ALONE is sufficient
(E) Statements (1) and (2) TOGETHER are NOT sufficient

29. (Medium) 20% of employees are women with fair hair. 40% of fair-haired employees are
women. What percent of employees have fair hair?
(A) 25
(B) 30
(C) 45

!

Your way to Top Business Schools _____________
259

(D) 50
(E) 60

30. (Hard) Among 5 children there are 2 siblings. In how many ways can the children be
seated in a row so that the siblings do not sit together?
(A) 38
(B) 46
(C) 72
(D) 86
(E) 102

31. (Medium) A, B, and C are points on the plane. Is AB > 15?
(1) BC + AC > 14
(2) Area of triangle ABC < 1
(A) Statement (1) ALONE is sufficient, but Statement (2) alone is not sufficient
(B) Statement (2) ALONE is sufficient, but Statement (1) alone is not sufficient
(C) BOTH statements TOGETHER are sufficient, but NEITHER statement ALONE is
sufficient
(D) EACH statement ALONE is sufficient
(E) Statements (1) and (2) TOGETHER are NOT sufficient

32. (Easy) What is (1/16)
- 1/4
?
(A) - 1/2
(B) 2
(C) 256
2

(D) 1/2
(E) 4

33. (Medium) In what proportion should the 30% - solution and the 3% - solution be mixed
to obtain 12% - solution?

!

Your way to Top Business Schools _____________
260

(A) 1:3
(B) 1:2
(C) 2:3
(D) 3:4
(E) 4:5

34. (Medium) A plane takes off from the hill at 750 meters above the sea level and lands
some time later in a town located at 50 meters below the sea level. During the first part of
its flight the plane gained height at a rate of 50 meters per minute but then it started to
descend at a rate of 20 meters per minute. The duration of the first part of the flight was
what percent of the total flight time?
(1) The duration of the descent is known
(2) The total flight time is known

(A) Statement (1) ALONE is sufficient, but Statement (2) alone is not sufficient
(B) Statement (2) ALONE is sufficient, but Statement (1) alone is not sufficient
(C) BOTH statements TOGETHER are sufficient, but NEITHER statement ALONE is
sufficient
(D) EACH statement ALONE is sufficient
(E) Statements (1) and (2) TOGETHER are NOT sufficient

35. (Medium) If operation $ is defined as
$X = X + 2 if X is even,
$X = X - 1 if X is odd,
What is $(...$($($(15)))...) 99 times?
(A) 120
(B) 180
(C) 210
(D) 225
(E) 250

!

Your way to Top Business Schools _____________
261


36. (Easy) What is X?
(1) X
2
- 1 = X + 1
(2) X + 3 is a prime number
(A) Statement (1) ALONE is sufficient, but Statement (2) alone is not sufficient
(B) Statement (2) ALONE is sufficient, but Statement (1) alone is not sufficient
(C) BOTH statements TOGETHER are sufficient, but NEITHER statement ALONE is
sufficient
(D) EACH statement ALONE is sufficient
(E) Statements (1) and (2) TOGETHER are NOT sufficient

37. (Medium) 160 kg of dry wood pulp is needed to produce 50 sheets of paper. How much
raw pulp will be needed to produce 200 sheets of paper if raw pulp loses 20 percent of its
weight during desiccation?
(A) 400
(B) 600
(C) 640
(D) 800
(E) 850

!

Your way to Top Business Schools _____________
262

Key Glossary
Arithmetic
Absolute Value
Addend
Addition
Additive Inverse
Approximation
Braces
Brackets
Canceling
Cardinal number
Carry
Circulating decimal
Common Denominator
Common Factors
Common Multiples
Complex Fraction
Composite Number
Consecutive Numbers
Cube
Cube Root
Decimal Fraction
Decimal Point
Denominator
Difference
Distributive property
Divisibility
Even Number
Expanded Notation
Factor
Factorization
Finite
Fraction
Greatest Common Factor T
Hundredth
Imaginary Numbers
Improper Fraction
Infinite
Integer
Interval
Invert
Irrational
Least Common Multiple
Lowest Common Denominator
Minuend
Mixed Number
Multiples
Multiplicand
Multiplicative Inverse
Multiplier





{ }
[]
()


































()





!

Your way to Top Business Schools _____________
263

Natural
Negative Number
Number Line
Numerator
Odd Number
Operation
Operand
Order of Operations
Ordinal number
Parentheses
Percentage
Periodic
Place
Positive Number
Prime Number
Product
Proper Fraction
Proportion
Quotient
Ratio
Rational Number
Real Number
Reciprocal
Reduce
Rounding
Scientific Notation
Subtrahend
Sum
Tenth
Terminating Decimal
Unknown
Value
Whole Number










()












()
()


















!

Your way to Top Business Schools _____________
264








Algebra

Algebra
Algebraic Fractions
Ascending Order
Axiom
Binomial
Binomial Coefficient
Binomial Equation
Closed Interval
Coefficient
Constant
Cross-multiplication
Degree of Polynomial
Descending Order
Equation
Evaluate
Exponent
Extremes
Extract a Root
Factorization of Algebraic Equations
Fractional Equation
Fractional Exponent
Half-Open Interval
Incomplete Quadratic Equation
Leading Coefficient
Like Term
Linear Equation
Literal Equation
Monomial
Nonlinear Equation
Open Interval
Polynomial
Power
Proper Fraction
Quadratic Equation
















()

( )







()







!

Your way to Top Business Schools _____________
265

Radical Sign
Radicand
Reduced Equation
Solution Set or Solution
Square
Square Root
System of Equations
Term
Theorem
Trinomial
Variable



,












Geometry
Acute Angle
Acute Triangle
Adjacent Angles
Alternate Angle
Alternate Exterior Angle
Alternate Interior Angle
Altitude
Altitude of a Triangle
Angle
Angle of Depression
Angle of Elevation
Angular Bisector
Ar
Area
Axis of Symmetry
Bar ()
Base
Bisect
Bisector of an Angle
Central Angle
Center of a Circle

!

Your way to Top Business Schools _____________
266

Chord
Circle
Circumcenter
Circumcircle
Circumference
Circumradius
Circumscribed
Circumscribed circle
Complementary Angles 90
Concave
Concentric Circles
Cone
Congruent
Congruent Squares
Congruent Triangles
Consecutive Angles
Convex
Corresponding
Cube
Decagon
Degree
Diagonal of Polygon
Diameter
Edge
Ellipse
Equiangular Polygon
Equilateral Polygon
Equilateral Triangle
Exterior Angle
Face
Height
Heptagon
Hexagon
Hypotenuse
Incongruent
Inscribed Angle
Interior Angles
Intersecting Lines
Isosceles Right Triangle
Isosceles Triangle
Legs
Line Segment
Median
Midpoint
Minute
Nonagon
Oblique
Oblong
Obtuse Angle
Obtuse Triangle
Octagon

!

Your way to Top Business Schools _____________
267

Oval
Parallel Lines
Parallelogram
Parallelepiped
Pentagon
Perimeter
Perpendicular Lines
(~3.14)
Plane
Plane Figure
Plane Geometry
Point
Polygon
Prism
Pythagorean Theorem
Quadrilateral
Radius
Ray
Rectangle
Regular Polygon
Rhombus
Right Angle
Right Circular Cylinder
Right Triangle
Scalene Triangle
Segment
Similar
Solid Geometry ()
Square
Straight Angle
Straight Line
Supplementary Angles
Surface Area
Tangent to A Circle
Trapezoid
Triangle
Vertex
Vertical Angles
Vertices
Volume



Coordinate Geometry
Abscissa (-)
Analytic Geometry
Cartesian Coordinates
Coordinate Axes
Coordinate Plane

!

Your way to Top Business Schools _____________
268

Cross Point
Hyperbola
Intercept
Open Ray
Ordered Pair
Ordinate (-)
Origin
Parabola
Quadrants ()
Rectangular Coordinates ()
Slope
Transversal
X-Axis
X-Coordinate
Y-Axis
Y-Coordinate

Word Problems
Balance
Biannual
Compound Interest
Dividend
Percentage Change
Quarterly
Rate
Semiannual
Simple Interest
Velocity


Statistics
Element
Empty Set
Equal Sets
Equivalent Sets
Mean (Arithmetic)
Median
Mode
Null Set
Range
Roster
Set
Subset
Universal Set


!

Your way to Top Business Schools _____________
269

Combinatorics & Probability
Combinations
Dependent Events
Independent Events
Intersection of Sets
Permutations
Probability
Union of Sets
Venn Diagram











!

Your way to Top Business Schools _____________
270

List for comments
We will appreciate if you leave here comments about misprints, mistakes and any other problems
with the book you find. Thank you in advance.

!

Your way to Top Business Schools _____________
271

List of Mistakes
# of question Topic of question (type) Your answer Correct Answer
ARITHMETIC
Numbers
1 A
2 C
3 E
4 E
5 D
6 C
7 C
8 C
9 B
10 B
11 A
12 B
13 E
14 C
15 C
Divisibility
1 A
2 C
3 A
4 B
5 B
6 E
Decimals and fractions
1 D
2 E
3 B
4 E
5 E
6 B
7 C
8 D
9 A
10 B
11 B
12 C
13 B
14 A
15 E

!

Your way to Top Business Schools _____________
272

16 C
17 A
18 D
19 E
20 A
21 A
22 A
23 B
Digits
1 A
2 B
3 A
4 A
5 D
6 D
7 C
8 E
9 A
10 E
11 B
12 E
13 D
Powers and Roots
1 E
2 B
3 A
4 C
5 C
6 A
7 C
8 B
9 D
10 A
11 B
12 A
13 B
Progressions and Statistics
1 C
2 E
3 A
4 D
5 C

!

Your way to Top Business Schools _____________
273

6 E
7 E
8 B
9 E
10 E
11 C
12 A
13 A
14 B
15 B
16 C
17 C
18 E
19 E
20 C
21 B
22 E
23 E
24 B
25 C
Factorizaton
1 E
2 A
3 B
4 C
5 D
6 C
7 A
LCM and GCD
1 B
2 A
3 C
4 C
5 B
6 C
7 B
8 A
9 D
10 C
11 D
12 B

!

Your way to Top Business Schools _____________
274

Division with remainder
1 A
2 E
3 C
4 B
5 D
6 D
7 A
8 B
9 C
10 B
11 E
12 B
13 E
14 C

# of question Topic of question (type) Your answer Correct Answer
ALGEBRA
1 A
2 C
3 C
4 D
5 E
6 A
7 C
8 D
9 E
10 A
11 B
12 D
13 E
14 C
15 D
16 C
17 B
18 D
19 E
20 A
21 B
22 A
23 D
24 C

!

Your way to Top Business Schools _____________
275

25 D
26 E
27 A
28 E
29 B
30 D
31 B
32 C
33 B
34 D
35 C
36 D
37 B
38 A
39 A
40 B
41 E
42 D
43 B
44 E
45 E
46 D
47 E
48 C
49 A

# of question Topic of question (type) Your answer Correct Answer
GEOMETRY
1 B
2 C
3 A
4 E
5 A
6 E
7 C
8 E
9 A
10 A
11 D
12 D
13 B

!

Your way to Top Business Schools _____________
276

14 C
15 D
16 C
17 B
18 A
19 D
20 C
21 E
22 E
23 D
24 B
25 C
26 E
27 D
28 E
29 B
30 A
31 C
32 D
33 C
34 B
35 B
36 A
37 B
38 E
39 A
40 D
41 D
42 D
43 B
44 A
45 E
46 D
47 E
48 C
49 B
50 B





!

Your way to Top Business Schools _____________
277

# of question Topic of question (type) Your answer Correct Answer
WORD PROBLEMS
Rate Problems
1 D
2 C
3 C
4 E
5 E
6 E
7 C
8 C
9 C
10 D
Work Problems
1 D
2 D
3 C
4 D
5 E
6 B
7 B
8 B
9 B
10 C
Systems
1 D
2 C
3 D
4 D
5 B
6 B
7 C
8 D
9 D
10 E
11 C
12 C
13 D
14 D
15 A
16 E
17 B

!

Your way to Top Business Schools _____________
278

18 D
19 E
20 E
Mixtures
1 A
2 B
3 A
4 D
5 C
6 A
7 E
8 C
9 C
10 D
Percentage & Profit
1 D
2 B
3 A
4 D
5 E
6 D
7 A
8 A
9 D
10 E
#of question Topic of question (type) Your answer Correct Answer
COMBINATORICS
1 A
2 A
3 B
4 C
5 A
6 A
7 A
8 D
9 B
10 A
11 A
12 D
13 D

!

Your way to Top Business Schools _____________
279


14 A
15 C
16 C
17 D
18 E
19 D
20 A
21 C
22 D
23 B
24 C
25 C
26 D
27 B
28 E
29 1. 8!
2. 7!
3. 3!6!
4. 8!-3!6!
5. 3!5!
30 E
31 E
32 A
33 B
34 A
# of question Topic of question (type) Your answer Correct Answer
PROBABILITY
1 D
2 B
3 C
4 A
5 E
6 E
7 C
8 E
9 C
10 D
11 C
12 E

!

Your way to Top Business Schools _____________
280



13 E
14 C
15 E
16 E
17 E
18 E
19 D
20 A
21 B
22 D
23 D
24 B
25 C
26 C

!

Your way to Top Business Schools _____________
281

Appendix
Operation Symbol Result Example
Addition + Sum 16 is the sum of 12 and 4
16=12+4
Subtraction - Difference 8 is the difference of 12 and 4 8=12-4
Multiplication

Product 48 is the product of 12 and 4 48=124
Division + Quotient 3 is the quotient of 12 and 4 3=12:4

English words Mathematical meaning Symbol
Is, was, will be, had, has, will
have, is equal to, is the same
as
Equals =
Plus, more than, sum,
increased by, added to,
exceeds, received, got, older
than, farther than, greater than
Addition +
Minus, fewer, less than,
difference, decreased by,
subtracted from, younger than,
gave, lost
Subtraction -
Times, of, product, multiplied
by
Multiplication

Divided by, per, for, quotient Division
: ,
u
b

More than, greater than Inequality >
At least Inequality
Fewer than, less than Inequality <
At most Inequality
What, how many, etc Unknown quantity X (or some other variable)

Multiplication table
1 2 3 4 5 6 7 8 9 10
2 4 6 8 10 12 14 16 18 20
3 6 9 12 15 18 21 24 27 30
4 8 12 16 20 24 28 32 36 40
5 10 15 20 25 30 35 40 45 50
6 12 18 24 30 36 42 48 54 60
7 14 21 28 35 42 49 56 63 70
8 16 24 32 40 48 56 64 72 80
9 18 27 36 45 54 63 72 81 90
10 20 30 40 50 60 70 80 90 100


!

Your way to Top Business Schools _____________
282

Fractions to decimals




Powers of integers
2
1
= 2 3
1
= 3 4
1
= 4 5
1
= 5
2
2
= 4 3
2
= 9 4
2
= 16 5
2
= 25
2
3
= 8 3
3
= 27 4
3
= 64 5
3
= 125
2
4
= 16 3
4
= 81 4
4
= 256 5
4
= 625
2
5
= 32 3
5
= 243 4
5
= 1024 5
5
= 3125
2
6
= 64
2
7
= 128
2
8
= 256
2
9
= 512
2
10
= 1024

Squares of integers
1
2
= 1
2
2
= 4
S
2
= 9
4
2
= 16
S
2
= 2S
6
2
= S6
7
2
= 49
8
2
= 64
9
2
= 81
1u
2
= 1uu
11
2
= 121
12
2
= 144
1S
2
= 169
14
2
= 196
1S
2
= 22S
16
2
= 2S6
17
2
= 289
18
2
= S24
19
2
= S61
2u
2
= 4uu

1
S
= u.2
S
S
= u.6
1
4
= u.2S
S
4
= u.7S
1
8
= u.12S
S
8
= u.S7S
2
S
= u.4
4
S
= u.8
S
8
= u.62S
S
S
= u.6
1
S
= u. (S)
2
S
= u. (6)

Vous aimerez peut-être aussi